460 Comments

My family moved in the middle of the school year when I was in elementary school, and because of that I never actually learned long division or long multiplication (it was later in the year in the first school and earlier in the year in the second school). I also missed a whole host of lessons about things like animal classification, American folk tales, and who knows what else. I never managed to pick up either long division or long multiplication, but it didn't seem to have any impact on my future academic career (I was the top of my class for math in high school and got good grades on my math classes in college).

Potentially missing something in Kindergarten could have hurt more, but those are also the skills that will be obviously missing and that the teacher would know to help with. If the whole class next year can't do long division, the teacher can throw in an extra lesson or two on long division and even everyone out.

Expand full comment

The fact that American schools seem to spend so much time on things like long multiplication and division is probably part of the reason why missing school doesn't matter much. At the end of the day, these are just fairly simple algorithms that you need to learn to execute without making mistakes. I'm pretty sure that if you wanted to learn these, you could pick them up as an adult in a few hours.

Now multiplication tables on the other hand. That requires a fair bit of memorization and I think an adult who didn't know them would have to spend some time before they were able to reproduce them quickly.

Expand full comment

On the other hand, modern memorization tools are really good. I'm using Anki to memorize Norwegian and I'm getting through about ten words per day, using maybe ten minutes per day. So that's, what, 2.5 hours of total effort for the 12x12 times table, assuming it's exactly as difficult as Norwegian and ignoring symmetry?

Expand full comment

I mean I’m not sure memorization tools are that great of a way to learn a language. You might learn direct word meanings without more subtle important parts. Same for times tables.

Expand full comment

In maths, drilling the rote bits is so that you have more brain left over when doing more complicated problems (real or academic).

Eg even proofs often involve some calculations.

Expand full comment

I’d say drilling is probably of significantly more value in math than in language. But we definitely do way too much of it in school, over a way too long time, and not enough other stuff.

Expand full comment

Drilling is tremendously helpful in learning a language. An expanded vocabulary is critical.

Expand full comment

I don't think anyone really does long division or multiplication. That task doesn't serve a purpose because a calculator is faster and more accurate. It also doesn't reinforce an intuition. I don't think that I can long division but I know higher mathematics. I also forgot all the integration tricks from calculus.

Expand full comment

Long division is useful as a precursor to polynomial division, imo, which is used a fair bit in more mathy subjects later on. Sure, it's basically retaught from scratch at that point as most people have forgotten how long division works by then, but it's not completely useless.

Expand full comment

I think I taught undergraduates polynomial division. But yeah, people would've forgotten. And my point is not so much that you won't need to use long division again in school but that you don't have to in real life. Do people long divide polynomials outside of classrooms? They probably just use wolfram.

Expand full comment

I teach polynomial division to 11th- and 12th-graders science majors (in France) every year. Only practice division by x-a, but hint at more complicated cases. It is not only useful as a technique, it helps understand and connect key concepts about polynomials, like why a being a root implies x-a can be factored

Expand full comment

But you're again relating it to something in school. My point is that people don't need to understand polynomial division to operate in the real world.

Expand full comment

> It helps understand and connect key concepts about polynomials

Surely the key-most concept is that numbers are polynomials? [Typically evaluated at x=10, i.e. in base 10.]

The correctness of the polynomial division algorithm then shows that long division also works in base b (for all integers b >= 2).

Expand full comment

I was only ever taught long division of polynomials, not numbers. (but we did do short division of numbers)

Expand full comment

I really do long division or multiplication all the time, maybe not every day, but certainly every week. If you only do arithmetic when you have time to pull out a calculator, then you'll never use arithmetic routinely in observing and understanding the world. You'll never get into the habit of computing probabilities or estimating quantities in everyday life, and so you'll never be able to understand the world empirically, and will probably fall back on something like Aristotelian metaphysics without realizing it.

Expand full comment

I tend to do that stuff in my head. I always have a phone with a calculator, but sometimes my hands are busy, and sometimes I'm walking around and don't want to stop so I can poke the phone without missing keys. While a lot of mental artithmetic is about finding ways NOT to have to carry all the intermediates of the "long" methods in your mind, I do think that the "long" methods are a useful starting point for developing your bag of mental tricks.

Expand full comment

What benefit do you get from doing long division/multiplication over rounding to three significant figures total (e.g. 6543 x 34567 to 6500 x 30000) which I think is much easier and faster for most people.

Expand full comment

I usually round off to about 2 digits. I didn't mean that I do the entire computation. I was responding to a sub-thread begun by someone who said he/she didn't know how to do long division, which I think implies being able to do long division even with rounding off.

Expand full comment

I regularly do long multiplication in my head (if by that you mean calculating a bunch of partial products and adding them up), and occasionally long division too. Last week I was sitting outside a restaurant waiting for my food and did some long division on paper while I waited. That kind of thing isn't useful for math, of course, but I wasn't doing math, I was doing engineering. It's true that a calculator is faster, but I didn't have one with me.

Expand full comment

It is a useful precursor to concepts such as 'algorithm' or 'recursion' that appear if you go on to study Computer Science. Even longhand addition/subtraction is helpful for explaining binary arithmetic and things like half- and full-adders by analogy.

Expand full comment

You clearly haven't had to deal with the typical smart phone calculator. By the time you've gotten past security, found and fired up the app, waded through the usual piles of crap smart phones bombard one with, you could have done the damned arithmetic in one's head. Maybe a dedicated calculator might make sense, but I dropped my faithful HP as a fashion accessory years ago.

Expand full comment

Really? My Android has a calculator app that came with it, and it doesn't have any ads at all. I have it on my home screen and use it all the time.

I'm reminded of a Facebook meme a while back: "Remember when we were told in school 'You won't have a calculator everywhere you go'? We sure showed them."

Expand full comment

Yea, the calculator app is pretty decent. And there are other apps freely available with absolutely no ads, too, because "writing a calculator app" is one of the easiest things one can do after learning how to program.

People still need to know what to put *into* an app, and if someone simply *cannot* add three 4-digit numbers on paper, it's a sign of something else seriously wrong that needs addressed, so we might as well make sure the kids can do it.

Expand full comment

Same with me but for fractions. And kind of funny, every other part of elementary math is ingrained in my head. I can still do those basic math facts and algorithms instantly and accurately, my kids need help with any of it and I can still do it all (I suspect this will change when they hit High School), except fractions. I have no idea how to do those.

Except, I can quickly derive the rules. So I show my kids how to do that. What's "23/50 + 12/25" I have no idea how to tell you off the top of my head.

But 1/2 + 1/2 = 1

Or 1/2 + 1/2 = 2/2

or 2/4 + 2/4 = 4/4

OK, I see what to do now. You get a common denominator then add the numerator.

I can see how going through that process might be more helpful for my kids then if I just remembered the rule and told them. So ironically, the one part of elementary math I missed so still can't recall how to do, might be the one part I'm best at teaching my kids.

Expand full comment

No irony there. The thing you had to figure out how to do yourself is the thing you actually understand. The thing you actually understand is the thing you can teach.

I think both those things are very general principles from maths teaching to sports coaching.

Expand full comment

Sorry, plenty of irony. No paradox.

Expand full comment

Not being able to do long multiplication and division and especially adding fractions so actually bad for math. You should know how to do that stuff and it’s important for higher math. But I mean that in a specific way - you need to understand the principles / properties well enough to do them for numbers given enough time. You won’t really need to “long multiply” or “long divide” or “fraction add” large numbers ever (although you would a few hundred years ago before computers), but it’s important to understand the principles that lead to them because without that you can’t do more complicated tricks. (I last long divided maybe eight years ago, but picked some 4 digit numbers in my head and managed to, based on principled knowledge, figure it out) - (abc) * (def) = (a00) (def) + (b0)def + c(def) = (ad*100 + (ae)10+(af))100 + (bd*100 + be*10 + bf)*10 + (cd 100 + ce10 + cf) or (a/b) + (c/d) = ad/bd + bc/bd =(ad + bc)/bd so 1/3+1/6 = 2/6 + 1/6 = 3/6 (from the formula (3+6)/18).

So while not being able to do long division or multiplication is bad, missing school for it probably doesn’t matter that much because of how shit they are. Don’t schools spend like five years worth of math on adding multiplying and dividing increasingly long numbers anyway? Missing the 4-digit year can’t just that much.

Expand full comment

I'm curious about your claim that you never learned long division or long multiplication. Do you mean "if asked to multiply 837 x 964 or divide 3943 / 73 without a calculator, you wouldn't know how to proceed"? Or that you never memorized which was the standard convention for keeping track of what goes where? Or do you mean "you learned it despite not being taught it by the school"?

I also switched schools in a way that would've prevented the school from teaching me long multiplication or division, but the second school expected me to know it, so my mother taught me. It took more than two lessons, although certainly less than 20. It would've gone much faster when I was older and had more mathematical background, but I'm not at all confident that not learning multiplication and division wouldn't have slowed me down when acquiring the "more mathematical background".

Expand full comment

For most of middle school I just had no idea, but it never really came up in situations where I didn't have access to a calculator. Nowadays I know enough algebra that I can figure out how to calculate it if I really need to, but I literally never need to do it on paper since I always have a calculator on me.

> not learning multiplication and division wouldn't have slowed me down when acquiring the "more mathematical background".

Kinda my point - for most of the individual lessons in elementary school, missing one of them might mean that you don't learn the thing, but it probably doesn't matter. If it does matter, then you'll learn it later.

Expand full comment

Mm. I think we potentially disagree; I was saying that I'm NOT SURE that not learning (things) wouldn't have hurt me later.

You say that you now know enough algebra that you can figure out some way to multiply out (8 * 100 + 3 * 10 + 7) * (9 * 100 + 6 * 10 + 4); I agree with that (although in passing, I find the point less convincing for long division).

However, I think that knowing the algorithm initially, before I had the concepts in my head, possibly helped me learn the relevant algebra later on, if only by providing an example of "oh so that's what I was doing!". I can't prove it one way or the other, in particular because I probably had plenty of other examples, and you can almost certainly skip any one example; all I know for sure is that in the limit where you end up skipping them all, it's much harder to learn.

Expand full comment

By "never managed to pick up [long division]", do you mean that to this day you cannot divide e.g. 46 by 11 without a machine? Or you can produce the correct result, just not showing your steps in rigid conformity to some narrow standard of how long division is supposed to be done?

Expand full comment

Dividing 46 by 11 is easy enough to do in my head, but if you asked me to do 40689 by 356 I wouldn't really know what to do. Hmm, I suppose I would want to first figure out how many times 356 * 10^n can fit into 40689, then decrease n's until I'm left with a number smaller than 356 and that's the remainder? But that's just from my general number sense, not because I was ever taught it.

Expand full comment

Right, 40600 - 35600 = 50000, so it's at least 100. 5080 is in between 3560 and 7120, and 5080 - 3560 = 1520, and 1529 is something like three or four 356es? Four 356es is 1424, and 1529 - 1424 = 105, so in integer division 40689/356 = 114, remainder 105. You can carry it further; 105 is almost but not quite three 35.6es.

Expand full comment

I think that test scores are the wrong thing to look at. I suspect the main benefits of grade school come from getting improved socialization and developing better strategies for general learning/problem solving. Like in your Spanish example, I also don't remember the majority of my second-language education from grade school, but I feel like the experience alone let me explore a lot of new ways of thinking.

Expand full comment

You also learn a great deal of categorization and existence knowledge. For example, while Scott does not remember Guassian elimination, he knows something called that actually exists, that it's relevant in math, probably algebra. That means if he hears "Gaussian elimination" in some context later on, he knows enough to roughly place the idea -- it has something to do with algebra -- which means he's about 80% of the way to being able to use the idea, all he needs now is some decent google-fu.

This is a very important part of education, which naive people often overlook when they contemplate how much factual knowledge has evaporated from memory over time. Learning sets of facts brings with it some additional meta-factual knowledge, including about the existence and categorization of facts, which usually sticks around long after the facts have evaporated, and which allows the human mind to recover the knowledge much faster than someone who never learned the facts at all (especially in this era of search-at-your-fingertips). You may not remember exactly in what years Ulysses Grant was President, or his specific policies with respect to Reconstruction, but you will often recognize that "Grant" is the name of a US President, and that he is associated with the Civil War, and that alone gets you 80% of the way to full knowledge, which you can easily complete with a little searching.

Naturally, a lot of people look at this and think "well, why don't we teach the metafactual stuff *only* and leave out the unnecessary facts? We'll be able to get more in when we dispense with the useless memorization of facts, and students will be less stressed and more capable." This is where "teachings students to think instead of memorize" comes in. Very catchy, and it appeals to everybody.

Unfortunately, so far as I can tell, it doesn't work. It appears the human mind simply doesn't absorb the metafactual knowledge *unless* it is digested along with some minimal set of facts, sort of the way Nutrient A is sometimes not absorbed well unless accompanied by Nutrient B. When you attempt to teach only the "thinking/analysis" stuff and punt on all the facts, the metafactual stuff doesn't stick. I'm not sure why this should be so, but empirically it does seem to be true. Maybe the human mind only really integrates the metafactual stuff when it has to, e.g. when it's a way of organizing facts in order to recall them better and faster.

Expand full comment

Agreed! I think this applies especially at higher education levels. For me, the biggest advantage of university over self-directed learning was having a nice structure for all the different concepts, even though I don't remember how to do many of those things now.

Expand full comment

Probably why "whole word" spelling works for some very bright kids (who probably already know some of it) and didn't generalize.

Expand full comment

Interesting point re metafactual knowledge. But im not sure that it requires all that schooling (just lazy research on wikipedia has sufficed to give me some metafacts).

And im also not sure why the particular selection of metafacts learned in k12 classes is so important (why “gaussian elimination” as opposed to “Sulla’s proscription”?)

Expand full comment

Related to the point about teaching only the meta knowledge not working: The Monad Burrito Problem (https://byorgey.wordpress.com/2009/01/12/abstraction-intuition-and-the-monad-tutorial-fallacy/). When learning something abstract, you struggle for a while with the details, then understand in a flash of insight. Once you get it, it's easy, so you'd like to transmit just the insight and skip all the hard work, but you can't because the exact mental analogy you used doesn't actually matter, and sharing your description of the insight won't help anyone who hasn't struggled through the concrete examples on their own.

Expand full comment

If you presume the ability to Google things, the idea of having a mental taxonomy in which ‘Gaussian elimination’ is categorized is trivial. Skip the steps of remembering it has something to do with math and just Google the words verbatim, and you will immediately learn that “In mathematics, Gaussian elimination, also known as row reduction, is an algorithm for solving systems of linear equations.”

This ‘metafactual knowledge’ shorn of Google is important if you’re a dilettante who needs badly to pretend to be conversant in subjects where you’re actually out of your depth. That way if somebody says “U.S. Grant” in a policy debate you can nod sagely and say “Ah yes, the American President, Grant,” and appear to be following the conversation when in fact the only thing you have grasped is the dim memory of 5th grade civics—and, worse, you have failed in an opportunity to learn something.

What is really important, and is not taught by ‘education’ but rather by exposure to vast quantities of knowledge, is the ability to learn, reason, and research for oneself. The ability, really, to say, “Grant who?” and then answer your own question by taking the initiative to crack a book or, even better, a primary source. Whether this is actually taught in any schools I do not know; I certainly learned it among my books and through argument with other people, not by learning to take a whiz in between bells and temporarily memorize a list of badly mangled ‘facts’ in time for a pop quiz. And I went to pretty good schools! Grade school is probably harmless, though, and at least accomplishes the objective of making sure that the vast majority of the unwashed masses are literate and numerate; probably the most harm is done in college and higher education, where it is all too common that mediocrity is showered with praise and hidebound, narrow thinking is regarded as creative research.

In any case, having ‘metafactual’ knowledge is simply the sign of being insufficiently familiar with your subject to discuss it except in general terms, and might even be worse than ignorance, because hand in hand with this metafactual knowledge is often a ‘scientific consensus’ or a ‘sense of the literature,’—in other words a pernicious textbook prejudice towards the socially desirable opinions about a problem or concept.

Expand full comment

>having ‘metafactual’ knowledge is simply the sign of being insufficiently familiar with your subject to discuss it except in general terms

What about other subjects though? Nobody can have in-depth knowledge of them all, so would you say it's preferable to know nothing at all outside of your area of expertise?

Expand full comment

I don’t think that’s a good dichotomy. There are more choices than ‘using deep knowledge of subject’ and ‘using traces of categorization bestowed by educator.’

I would say it’s preferable *not* to express opinions based on ‘metafactual knowledge.’ Or make decisions using it. Simply *having* such traces is probably inevitable but confers you nothing but an unwarranted amount of confidence. “Drink deep or taste not the Pierian spring,” right?

If you must make a decision, educate yourself to at least an amateur’s understanding of actual facts and principles; if you can’t do that, use your reason to determine who the trustworthy experts are, if any, and examine their evidence; ideally, do both. This is a far cry from expertise, but it’s enough to improvise with.

Expand full comment

>I would say it’s preferable *not* to express opinions based on ‘metafactual knowledge.’ Or make decisions using it.

Yes, a world in which everybody follows this principle might very well be a much saner one. It's clearly not how this one works though, given that the biggest current "tech" giants have grown insanely rich off of advertisement, which provides even more tainted "knowledge" than an educator ostensibly following scientific consensus.

Expand full comment

Anyone who believes that ads are a source of world knowledge cannot be educated anyway, but I suspect those people are very few regardless.

Whether I am right or wrong about that is immaterial. What’s important for each person is that they don’t wildly exaggerate the validity and importance of barely remembered or unremembered categories. But nobody can stop you from overrating your own knowledge and I don’t presume to try.

Expand full comment

Regarding the digestion, I suspect part of it may be that it is easier to recall an 'experience' rather than pure facts. For example, I had some probability and statistics courses a few years back, but barely do statistics nowadays. I recently was asked by a friend to help interpret some claims about covid testing. We wanted to calculated certain conditional probabilities, and it seemed that we needed was to flip the events on a conditional probability. This situation seemed familiar, and I recalled that Bayes rule did something like that. So I looked it up, and was able to calculate what we wanted. Note that I did not recall anyone telling me "if you want to flip events, use Bayes". Perhaps someone did, but that is not very effective. After all, we don't (and shouldn't!) believe everything people say to us. Having to remember "My teacher said that X" seems more difficult than simply remember "X". Furthermore, the teacher can only teach a limited number of explanations, and these do not necessarily have to "fit" with my internal mental model. What I describe as "flipping events" is a natural way of describing the goal to me, but it may be very confusing for others with different internal mental models of conditional probabilities.

Expand full comment

If you had been home for that year you would still have been doing things, just different things. Learning to help your mother make dinner, or organizing and running a WoW guild, or arguing politics with your best friend or DMing a campaign might also have let you explore new ways of thinking.

Expand full comment

That's a fair point, although I imagine there are some benefits to being forced to explore outside your intellectual "comfort zone". Like, perhaps it gives more practice with convergent thinking as opposed to divergent thinking.

Expand full comment

Yes!! I grew up in India and went to mediocre schools. The education I got at home debating ideas within the family was far superior to that. And my parents had interesting friends. For example, an economics professor from Brown University who visited once a year and stayed with us. I learned more from conversations with him than maybe in all my K-12. In middle school, I'd ask him things like "Do you believe in astrology?" And, "But how come some predictions come true?" In a couple of sentences, he taught me so much.

Expand full comment

I certainly agree with this, but there is value in learning culturally relevant things that others will also learn. It amazes me when my kids come home from school having learned the same folk story cultural things (like Johnny Appleseed, but including songs, anecdotal stories, patriotism, etc.) that I did. There's a baseline for communal living that helps groups of people who have very little reason to ever meet still have shared values and understandings. If we all just do our own thing, then we might be better individuals in a worse society.

I consider this speculative at this point, but I've been thinking about it a lot more as I watch my own kids grow up.

Expand full comment

Yes, there is indeed huge value in that.

Expand full comment

It also means that they all learn the same false things, with less chance of someone who has learned different things pointing out evidence that they are false. A fair amount that I was taught in a very good high school wasn't true.

One of my standard examples is the claim in the driver's education course that two cars hitting head on at 50 mph was equivalent to each to running into a brick wall at 100 mph. It's quite easy to prove that that cannot be true, but it was what the book said, and when I questioned it and we took the question to the physics teacher, he said it was true and was uninterested in my proof that it was false.

For almost anything politically relevant, such as the causes of the Great Depression or, nowadays, climate change, the school will teach some orthodoxy, likely to be in part false, with which one depending on the political environment, both community and teachers.

Expand full comment

I can't disagree with you there. My mom liked to muse about how many moons Jupiter supposedly had when she was in school, verses how many when I was in school. The number is significantly higher now, too, even though the true number hasn't changed at all. It's also surprising how many times in just my lifetime the origin of mankind has shifted. Usually between east and west Africa, but at least once to Asia. Each time taught as definite fact.

Expand full comment

So, humans evolved in an environment in which they were socialized by interacting with many other humans of different ages, with everybody performing diverse social roles in diverse circumstances.

It'd be pretty odd if you got "better socialization" by shoving them into age-segregated groups performing stereotyped activities under highly formalized rules, all under the supervision of a limited number of adults who were themselves in very stereotyped roles.

And I don't think that you do.

Expand full comment

I think that's an interesting idea about reducing the amount of age segregation in educational environments, but I suspect even the highly segregated environments we have now are better for socialization than isolated homeschooling environments. Although, I guess not all homeschooling environments need to be isolated.

Expand full comment

I'd rather get rid of "educational environments" as a distinguished category. I just don't know how to get there.

I think a lot of pressure for their existence comes from the fact that all the *adults* are also shoved into weird confining institutional environments (you know, "jobs"). The adults' envronments have no room for the kids, and in fact actively demand that the kids be excluded. As long as that's true, even if you don't have "educational environments", you'll nonetheless have still-fairly-age-segregated kids doing God-knows-what, whether singly or in groups, with limited adult guidance and limited adult examples. Which is probably not ideal socialization either.

And of course if your goal in "socialization" is to TRAIN people to be able sit down, shut up, act like everybody else, and fit into an institution, well, then...

Expand full comment

As I've written at considerable length, generally children sort themselves into ability bands very early in life and then stay in those same bands, relative to peers, with remarkable fidelity. (See https://freddiedeboer.substack.com/p/education-doesnt-work) Presumably this is due to some kind of intrinsic ability asserting itself consistently throughout academic life, probably genetic in origin. My guess is that once students get back into school they will fall back into their old ability bands in dominant majorities and the hierarchy will have reasserted itself. What's less certain is how the schooling pause will influence how any given age cohort performs relative to another. Since the pause generally seems to be happening fairly uniformly across cohorts it's hard to see that it will make much difference, although I suppose people who miss their last year of formal schooling and never make it up could perform worse compared to those who finished right before the pauses. But in general I suspect that you're correct and that this just won't matter much, for the reasons you've laid out, save for those rare kids who just literally never return to formal schooling at all.

Expand full comment

You know a lot more about this than I do. My impression is that a much larger than usual fraction of kids have either dropped out or have had their grades crater. I guess the question is what fraction recover? Do we have any way to guess?

Expand full comment

Results probably correlate tightly with the family/home/neighborhood environment.

Expand full comment

Correlate, yes. But the kinship studies strongly suggest this is not a causal relationship. In the behavioral genetics/population genomics literature they call that stuff the "shared environment," and it is moderately influential early in life but that influence rapidly declines as children age due to the Wilson effect.

Expand full comment

Bear in mind that I'm talking about relative performance - a cohort of students that had to face Covid restrictions when others didn't would definitely be disadvantaged in short-term assessments, although for the reasons Scott says here I suspect that disadvantage would fade and ability would reassert itself over time. More to the point, though, this has been a pretty close to universal condition, with some exceptions, so the relative performance is not likely to be deeply affected. (I think!)

Expand full comment

I think the talkative kids did well on zoom and the quieter kids did not (separate from other confounders). Watching the class dynamics in my kids' zoom classes made me think that, and I had not thought that ahead of time. So many kids in those classes said nothing - partly because the teachers often did zoom like it was identical to being in front of a room, except with a camera. It is not; they did not realize they were now doing theater and so failed to account for the pros and cons of that. The teachers would get more and more frustrated, the kids in general would get quieter and quieter, and then one or two kids would get the mike and just run on; the teachers were so pleased someone was talking that they did nothing to rein those kids in, with the other kids sitting in increasingly uncomfortable silence while the theater went from bad to worse.

Also, the kids couldn't lock eyes with someone across the room and know they were not alone, recognizing the ridiculousness. They could not poke their neighbor when the teacher's anxiety ramped up. They could not walk out into the hall and share the rude names they had come up with for the teachers and other students, participating in a community. There was no personal defense possible; camera, cranky teacher, silent class, that one kid yammering away about how much they agree with the teacher. Fodder for many horror movies to come, I expect, as this generation grows up. At that point, some kids couldn't bring themselves to do the assignments, others could.

The angry, sarcastic, snarky parts of socialization, so necessary to school, were gone. People say it was bad because kids missed the friendly parts, and I think that is partly true but not the whole story.

Expand full comment

Anecdotally, I was far behind at the beginning of my education then rocketed to the top of my class for the rest of grade school. I was a middle to upper middle student in middle school and close to the top student at my high school. I then went to a somewhat elite university where I was bottom of the class the first two years and top of the class the last two. Now, some of this was due to a turbulent life, but I'm curious to hear your thoughts on it. (And to be clear: I'm not trying to pose this as evidence either way. I'm just not all that well informed on how to contextualize my experience.)

Expand full comment

You can think about someone like Ramanujan, who came from a position of near-total educational depravation and yet in very short order became one of the most gifted mathematicians of his era. Because he had a natural talent that's simply not something most people can access.

Expand full comment

Did Ramanujan have “near total education deprivation”? He was undereducated I think, but he regularly attended some sort of school. He was also born a Brahmin.

Expand full comment

Hmmm, fair question - this was all pretty dimly remembered on my part, I'm probably exaggerating. I think I got the impression from a Numberphile video?

Expand full comment

I just read the first 40 pages of a bio of him and he didn’t seem that undereducated. Started at age 5 in a “12 kids in someone’s house” school, moved around for a few years, did half a year at a school in a language he didn’t speak, scored 1st of class in arithmetic at age 10.

Expand full comment

5 as the normal age. He wasn’t totally deprived but it’s fair to say that really good education wasn’t one of the big causes there

Expand full comment

What does his being Brahmin have to do with this?

Expand full comment

Sorting into innate ish ability classes being partially mediated by ancestry!

Expand full comment

He seems to have been poor, as many Brahmins in Tamilnadu are, even today. Tamil Brahmins are just 1 to 2% of the Tamil population, by the way. Due to disproportionate success, they're seen as having stolen something and are the target in the dirty identity politics in the state. Until the British came and needed people to maintain ledgers (reading, writing, basic arithmetic, English..) to collect taxes, Tamil Brahmins were not making much money.

In free India, Tamil Brahmins often have to leave the state to escape the legal discrimination. Every politician in the state gets ahead only by proving his or her anti-Brahmin credentials.

Expand full comment

I read your article and was bracing for it to end with a polemic about this is why we should end the meritocracy, which I disagree with for similar reasons Scott did in 2017. But instead you made a very strong argument for wealth redistribution which I'd be hard pressed to argue against, except maybe over the precise method and amount. I'm general I've found myself agreeing with more of your writings in the last two years. I don't know if this is a sign of me becoming more socialist or you becoming better at persuading people on the right.

Either way, it's a shame that the only thing the right and left seem to agree on is the importance of education, when the studies suggest it's one of the least impactful levers to adjust.

Expand full comment
founding

I feel like we have not properly grokked the fundamentally flawed nature of every social science study and far too many people treat them as statements of simple fact about reality instead of generally a mixture of fraud, other kinds of fraud, and luck.

Expand full comment

It depends on what you mean. In terms of predictive validity, modern educational assessments are incredibly accurate; we know how to develop tests that predict future performance on all manner of academic tasks with remarkable precision. Not just assessments predicting assessments but predicting real-world behaviors like observed reading ability. Does that make them "statements of simple fact"? I don't know. But I do know that if a casino would take my action on which children will succeed in school even deep into the future, and I could give those children a battery of modern tests, I would become a very rich man.

Expand full comment

Perhaps you could become a very rich man if the casinos took your action, and you personally administered the tests -- but do you trust nationwide tests to be administered fairly, or in fact at all ?

Expand full comment

“ do you trust nationwide tests to be administered fairly, or in fact at all”

Why would they not be

Expand full comment

The data for “them being incredibly accurate at predicting future performance” are not administered by him, and they’re still predictive. And while people do cheat and study ... that’s not responsible for the effect.

Expand full comment

yes? of course some problems will arise but those should be rare enough that their effects disappear at the margin (this is proven by the very fact that the tests are predictive)

security measures in public schools are completely bonkers these days when it comes to tests. if you want to cheat as a teacher it's really quite difficult (I administer the SAT 4x a year)

Expand full comment

Is there a risk of circular logic here. Tests that predict performance on "academic tasks"? How is the performance at the academic test measured? Some sort of test?

Expand full comment

The actual behavior that the construct refers to. The ability to read as judged by literacy experts. I'm trying not to be short with you because this mysterianism has infected the conversation - "what does it really mean, to be able to read?" - when we know perfectly well the answer to those questions.

Expand full comment

Ah. So the educational tests are actually predicting performance as judged by the evaluation of experts. Which experts, I wonder? Academic experts?

I realize by the way, that this line of thinking doesn't really go anywhere. My point is only the same one sibling comments are making - Alexander is arguing that success as measured by metrics controlled outside the system don't give much support to school being super critical. You're arguing that success as measured by experts tells a very different story.

Expand full comment

Every social science study is flawed?

Expand full comment

I wonder if quality of memory significantly influences one's take on this question. I was a top student, but I remember even less than Scott purports to. So to me, the idea of worrying about having missed 5% of which I will forget 95% is quite absurd. But perhaps if I remembered all the classes and the content, then the value of that time would be much more real, even years later.

Expand full comment

Exactly. A crucial point. People forget everything anyway. I remember one year in December our school got snow days and I missed some finals. Some people were worried they would forget too much in 3 weeks. That seems reasonable if they do not study! So if someone goes from 90% to 70% because of three weeks, imagine 3 years or imagine 6 years. If the information is nearly totally lost by age 22 - 25 when many young adults are just starting their career, how useful can it be?

Expand full comment
author

So to some degree "people forget everything anyway". But places with zero school (eg third world countries) do end up with much less educated people. So somebody is getting education at some point. While it's possible that some of this is ambiently in the environment (eg a kid I know who learned to read by playing video games with lots of text in them), it wouldn't surprise me if smarter people could pick things up ambiently from the environment and duller people need their hands held and benefitted from school a bit. I just think a little school probably goes a long way.

Expand full comment

I think it's important to differentiate between "school", "education", and "learning". Also, I would introduce an "engagement" variable as a quality multiplier.

Let's say that the average kid gets 10,000 hours of school. This may include 4,000 hours of education, but at a low rate of enegagement. So obviously we do end up with some learning at the end. (I suspect that learning that results from low enegagement will also have much lower retention, as in your example about the Civil War).

If the above example is the status quo, we can play around with the formula. We can try to replace school with other education tools -tutoring, self-directed, online/adaptive, apprenticeships, etc. These will have various rates of engagement with various results.

Or we can cut out "education" entirely and replace "school" with "life". Obvisouly there will be some learning involved in 10,000 hours of life, but it will be entirely dependent on the specific life.

My point here is that when we consider school, or no school, we can be running the economics on the hours spent and the quality of the hours. We can also be optimizing these economics for "learning".

As a parent, If I can come up with a solution that can provide 10,000 hours of high engagement learning to my kid, then it's probably a much better option for my kid than school. Conversely, if I know that I'll be at work all day and my kid will spend the day alternating between staring at a blank wall and looking at porn, then school becomes a very attractive option.

Expand full comment

To some degree. People don't forget how to read, or at least they rarely do. Nor do they forget their failure to learn to read or the lessons drawn from such failure.

Expand full comment

The overwhelming majority of my early knowledge of English comes from computer games, RPGs (this does net you weird vocabulary, it must be admitted), reading SF/F, and talking on the Usenet in the 90's. I mean, yes I needed to be taught enough for that to even be possible, but once there? School was not even close as a factor.

Expand full comment

The more I think about this, the more I have the following problem with this line of argument:

Sure, I'm a molecular biologist and I don't use much beyond biology and general literacy. But to figure out I wanted to be a biologist I had to learn a lot of stuff. I figured out that I liked traveling but had no faculty for languages. I learned that I loved reading history but disliked doing historical research. I was good at math by normal human standards but not future mathematician/physicist standards. I liked devising chemical syntheses on paper but not doing them in the lab. But I really liked and had an aptitude for molecular bio, and that's what I do.

It's an inefficient system. They make us all do math until we hit our limits, and 0.3% are deemed good enough to do it for a living. We learn about all sorts of stuff so we can figure out we hate it. But try coming up with a system where you only learn exactly what you need.

Expand full comment

That's a very interesting comment, IMO.

b/c I suspect that "kids don't suffer from lack of schooling" depends a lot on what you think schooling is meant to achieve.

One, to provide an education. The limits Scott mentioned are more true than not, I think. "We forget everything".

Two, to test pupils in a long winded rat race. There, I believe lack of schooling would probably hurt more or less, depending on the way the rat race is designed. If every year is a "yay or nay" on a large amount of material with a non-recoverable-afterwards fork, every minute of schooling actually matters.

Three, to awaken curiosity as Ivan Fyodorovich suggests and give a bit of an initial basis in any subject you might want to pursue. For example, I too have forgotten all about massive amount of the Maths I was taught. I especially hated geometry and trigonometry. I'm told many types of engineers and physicists use these tools.

Now, contrary to Ivan Fyodorovich, I knew pretty early on I didn't want to work with lots of sciences and Maths and I still had to go through the pain of learning that stuff to satisfy the rat race setup of the French educational system.

It sure seems like a lot of waste but I wouldn't call the impact of e-schooling negligible in a country like France where we all compete for limited space (my understanding being that, in the US, if you're born in the middle of the country and don't want to leave your small town, education is basically not needed. You'll find a local job where a bit of discipline, work ethic and a modicum of decency will see you through. Education the French way is only necessary if you're planning to compete in the top 20 cities).

Expand full comment

A serious issue is that kids in much of the US didn't miss a year of school, they had a year of sad-ass Zoom school during which an abnormally large number of kids checked-out and/or flunked everything. I seriously wonder if that is ultimately worse for them than a year of no school.

Expand full comment

I suspect you’re right.

Expand full comment

This is a very good question.

Expand full comment

My mother, a teacher at a regional school in Germany (regional schools being the type of school that doesn't qualify you for university, it's grades 5-10) (in a bad neighbourhood, if it matters), has observed that about half the students did their homework assignments and tried to keep up (or their parents made them) whereas the other half didn't really do anything at all - some of the kids don't have (enough) access to the internet, no room for themselves, siblings they had to care for while the parents used the family computer for work etc, so this is not surprising. (I have heard from other classes in secondary schools where all kids attended zoom meetings - I expect these are socioeconomic differences).

The problem is that now both these groups are in the same class. This is frustrating for everyone. Students that kept up resent the repetition, students that didn't have to rush through everything etc.

There are also some teachers that gave lots of homework, called around regularly, held zoom classes for those able to attend them, and some that didn't make their students do much of anything, fully aware they'd need to teach all of it again anyway once school resumes regularly. I expect covid to have widely different effects depending on which of these groups you fall into and which sort of teachers you had.

Expand full comment

If zoom is so bad that it is not as beneficial as leisure time, wouldn't in-person lectures in a great deal of poorly run schools be not as beneficial as leisure time? I think so.

Expand full comment

Maybe, but my specific concern is:

- Far more kids are dropping out/failing than in a normal year

- Some would have flunked out eventually anyway, some will recover when school resumes as normal, but for some this might permanently derail their educations/life trajectories.

In other words, it's the failing, not the learning loss/opportunity cost I'm worried about.

Expand full comment

For some kids zoom school was better than in person as far as learning. I think that was definitely the case for my 11 year old. In regular school there are lots of disruptive kids that get in the way of learning. In zoom school those kids just completely checked out, which was bad for them but actually good for my daughter because they were no longer disruptive.

Expand full comment

I'm not sure the disruptive kids necessarily checked out completely. I've been thriving in online classes at uni because I can listen with one ear when bored, playing (simple) computer games and checking back in when I want, with recorded lectures I can go through them at my own pace, and when I zone out I can just pause and get back to it later (I take ~3h per lecture that way, but I retain a whole lot more than when I attend in person). I've never been too disruptive as a kid, but now I can do all sorts of things that would have been - sing to myself, make annoying noises, doodle, chat with friends, talk to my partner, lounge in whatever position I want to, rock and sway, even get up and walk through the room. Being disruptive is no longer disrupting! I expect a lot of ADHD kids to do better, not worse, under those circumstances. I attend way more classes now than I used to, because I don't have to commit to sit in a room (I can't sit for long for chronic pain reasons and attend classes mostly from the sofa now), bored out of my mind or jittery. All of this dependent on the option of disabling the webcam, of course.

Expand full comment

Remote kindergarten worked *startlingly* well. I don't know about the socialization side of things, but my kid learned to read and write (to a reasonable, age-appropriate extent) entirely from remote lessons. And it definitely wasn't the only thing they covered.

Expand full comment

I can't say that Zoom school was bad for all kids. On a population level though, there has been substantial measurable learning loss:

https://texas2036.org/posts/tracking-covid-learning-loss/

Now, for the reasons Scott gets into, I don't think this will matter greatly in the long run. What I worry about more is the kids who out and out failed or dropped out, and I worry some of them will be derailed forever. I can't find data quite as comprehensive for this, but this article has some links https://www.usatoday.com/story/news/education/2020/12/23/students-failing-grades-online-class-coronavirus/3967886001/

Expand full comment

Someone said the New Orleans results after Katrina were due to the new charter schools being able to better screen the new teacher cohort.

Expand full comment

There was a good EconTalk about this a few years back I think (I can't find it, sorry). Part of the issue seems to have been that the school boards and such were literally involved in massive criminal activities, such that the FBI had people permanently posted there. (Presumably the FBI were not the source of the criminal activities, but who know these days...) Bribes and kickbacks for jobs, money disappearing, political nastiness... The interviewee's position was that the system was just so horribly corrupt and broken that shattering it and starting entirely from scratch was needed, otherwise the system's immune response against reform was too strong.

Expand full comment

Also, NOLA lost about 1/2 its population after katrina. Also, "Case studies of neighborhood recovery show that more-advantaged neighborhoods before Katrina have higher rates of return, and even gain new residents, while disadvantaged neighborhoods remain sparsely populated (Elliott et al. 2009). "

Might want to look at the MS coast to see if charts with NOLA or not - much different before/after demographics

Expand full comment

The main reason was that a lot of the poor kids didn't come back.

Turns out if you get rid of a lot of the poor kids, your scores go up.

Expand full comment

> I’m inherently skeptical of these, because I’m suspicious that education researchers love finding that education has huge effects, that any disruption to education is a disaster, and that kids should be in school much more.

We should also expect education researchers to find that strikes are bad for students, since education researchers are incentivized to find results that school administrations approve of, and school administrations want reasons to shut down teacher strikes.

Expand full comment

I don't know that administrations do want to shut down teacher strikes. My sense has always been it's mostly "schools vs parents" not "administrations vs teachers" in these sorts of events. Finding that strikes are awful would be good for schools trying to get more resources given to them, because "If we can't make an agreement that gets these teachers back to work, all the children die!" sounds like a good way to get resources given to the schools.

Expand full comment

Administrators definitely want to end strikes, as they are judged on their ability to fulfill the purposes of the school. You are correct that they are even happier if the teachers get everything they want and the relationship is smooth. There's very little downside for administrators in getting more resources, unless it causes long term financial problems for the district.

Expand full comment

I agree that administrators have more costs associated with teachers' strikes than the teachers themselves, preferring to avoid them. I expect though that by the time contract negotiations get into strike territory it is not because of decisions the administrators are making but because of higher level issues, e.g. needing more money for pay or benefits, that the administrators don't pay the price of. So strikes tend to be against those holding the purse strings of both the teachers and administrators, and the result of a successful strike is both getting more money and resources. I haven't heard of any strikes where the end result was teachers getting more money by way of the administration getting pared down to make the cash available.

Then again, there are probably many strike outcomes I have never heard of :D Still, that seems like the sort of outcome that would show up in the literature a lot.

Expand full comment

Cuts to administration do sometimes happen, although even if the cause is higher teacher benefits it's unlikely that a school board would draw that line directly. The more likely scenario is one board negotiating higher wages/benefits and the next board making cuts where applicable to fix a shortfall. Unfortunately for the teacher's union, the administration is usually far too small to make administration cuts really relevant to the higher costs of teachers. If admin is 10% of the number of teachers (which is probably too high), then there simply isn't enough money on the admin side to cover teacher increases. A 5% teacher increase is equal to a 50% admin increase or more. Not to mention that each individual administrator is more likely to be filling a necessary role and in direct support of the school board. You're not likely to cut your building principal, even if you can't afford the teacher contract.

Expand full comment

Huh, I would have assumed that education researchers want to find things in favor of the teachers, and teachers want to show that education matters, but that strikes don't.

Expand full comment

It depends on whether education researchers are more aligned with teachers or administrators. I would guess administrators since it's usually administrators who pay the bills and give education researchers a platform, but it could also be teachers since education researchers generally come from a teaching background.

Expand full comment

Well, remember that the point of strikes is to inflict pain by not working, so that your bosses will accede to your demands to get you to come back to work. If the only bad thing about teacher strikes was the inconvenience of finding a place to put your kids 8 hours a day, someone might ask "Why do we pay teachers so much?" and just invite them to strike forever while they put their kids in daycare.

Expand full comment

Put another way, your biggest worry when taking a month off of work is that you will return and no one will have noticed you were gone. That looks bad for your continued employment.

Expand full comment

Teachers want the result to be both that education matters and also that strikes matter. That can align quite easily, and both incentivize the administration or school board to resolve things quickly (by giving in to the requests of the teachers).

If strikes don't matter, then administrators can just wait out the teachers.

Expand full comment

Based on conversations with K-12 teachers, I've come to think:

- there are roughly two skills taught, "number sense" and "reading comprehension"

- good curriculum + teaching can improve them, though practice and exposure is a big part of it

- it's possible to catch up quickly, though gets harder later b/c you're missing foundations

I think many schools are not good enough at teaching these for absence to matter. But especially for kids who don't get much exposure elsewhere, absence is indeed a missed opportunity. Just, one that would only show up in the stats if the schools were better.

Expand full comment

I think that the very low value add of schools does account for a lot of what we are seeing here. When I enroll my kids in piano class, I expect them to play piano better than what they could do if I just had one around the house. Absences there would likely show up. The really damning thing about schools is that they don't seem to be able to teach much more than kids pick up just by being around their families and doing normal human things. Most parents who manage to have jobs and run a household can, and probably do, teach their kids all they really learn during their school years, with the exception of some trivia and harder subjects. (Neither one of my parents was going to teach me math past algebra, but had everything else covered far better than school.)

Expand full comment

I was in a meeting with a third grade parent who admonished her daughter, with great seriousness, that the curriculum had gone beyond what she, the mother, could teach and that now the kid was on her own to learn in school what she could learn.

You might be surprised at how many parents don't know much of anything beyond second grade level.

Expand full comment

Oh no, just the opposite, I am often surprised at how many people make good with just basic math and reading. What people NEED to know to get by is really pretty low level. Three cheers for the division of labor.

Further, if the research on nature/nurture is anything to go by, it seems somewhat unlikely the daughter was going to pick up a great deal more, either.

I do wonder what the subject of the curriculum was, however. If it was history or science I could see a parent saying "I don't know this stuff, and I am not going to be reading the book for you to learn it, so you are on your own." Third grade math is what, maybe multiplication? Then again, the way they teach math now is bonkers... it takes me a while to figure out what the hell they actually want my kid to do on the way to the answer sometimes.

Expand full comment

If missing school affects graduation rates but not learning, maybe kids who miss a lot of school notice how they didn't miss much actual learning, so they realize school matters less than they thought it did, which makes them more likely to choose to drop out.

Expand full comment

I was wondering the same thing while reading that section.

Expand full comment

I wonder how much absences cause dropping out vs. how many kids drop out for the same reason they were absent, ie. their parents don't care about school anymore than they do.

Expand full comment

One note I'd like to observe: When it comes to "excused" vs "unexcused" absences, what we are generally looking at is, "Kids who have parents who care and are with-it enough to communicate with the school when the child doesn't attend vs children who don't." So it's still very much a correlational relationship that "unexcused" absences predict learning deficits.

Expand full comment

Most of my "unexcused" absences in high school were from before I figured out I could just forge an absence slip from my parents. Presumably that would muddle some of the correlation.

Expand full comment

I always had to ask a girl with much batter cursive than me to handle that task.

Expand full comment
author

I was thinking it was more "kids who are delinquent and go smoke pot instead of attending school" vs. "kids who get sick and have to go to the doctor a lot"

Expand full comment

That's another way of saying "parents who don't care" vs "parents that do care"

Expand full comment

Sure, that's part of it too, but again, the significant distinction is "kids whose parents see the Unexcused Absences and take steps to make sure their kids get to school" versus "kids whose parents aren't paying attention to their education."

Thinking logically, it has to be something along these lines. If missing school is what causes learning deficits, then there is no logical way to claim that missing school because you're sick doesn't have that impact but missing school because you're skipping does. You're missing school either way. But the kid who is skipping doesn't like school and even when he is attending is probably not really engaged, and of course he's going to learn less. It's not the absence that has the impact, but the absence is indicative of either low parental involvement or low personal motivation.

Expand full comment

Some parents don't care whether the absences are excused or not, which will naturally result in a higher proportion of unexcused absences. Those parents tend to also not care much about what their kids do or making sure they are actually learning. They usually do miss a lot more school as well, but that's not a given. Many of the higher earning and more conscientious parents where my kids go to school take multiple vacations during the school year and otherwise get their kids out of school. The kids do great, and as many absences are excused as can be allowed.

Expand full comment

Something that was missing from almost all of those studies is a measurement of resources required for catching the students back up. Almost all systems have some stabilizing mechanisms to help bring up students who e.g. got cancer back up to grade level (extra attention by classroom teachers, special ed, social workers, etc). Even disasters come with extra resources in the US to help support the students. Just because a system kept results within it's normal parameters during normal times doesn't imply it can do so during abnormal times. Looked at from this lens the Pakistan study looks much worse.

Expand full comment
author

That's a good point, thanks.

Expand full comment

I've found systems thinking to be the most powerful tool in my toolbox for reasoning about how organizations function. If you haven't read it, Thinking in Systems by Donella Meadows is a really good primer and has the single best chapter in a book I've read (the one about leverage points).

Expand full comment

My mom's an elementary school teacher. This year she's teaching second grade with a class of students with mixed reading skills from third grade to pre-k. Note this is NOT a special ed class. There is a single systemic resource to catch the kids up: for a few hours a week they take the kids out of the classes and regroup them roughly by skill instead of by age. This practice has, of course, been suspended during Covid. But even in normal times I'm not sure how effective it is, as the older kids would get teaching at the same speed as the younger kids.

Then again her principal usually places her a grade level after a bad teacher and a grade level before a rookie teacher, so perhaps that's an example of a resource at work.

N=1.

Perhaps in the Pakistan situation, a bunch of western counselors swooped in and told them about how they should have PTSD or something?

Expand full comment

A some of the stuff I'm thinking about is of the form of how her principal behaves. Preferentially spending a minute extra per class session with a kid that's behind, the administration putting a kid that likes to help the teacher in a class that has a slightly higher number of behind students, things like that.

Other stuff, like extra help, either through an IEP (which technically makes it special ed) or non-IEP support from the same people is also built into the system at most places. My mom is a retired speech language pathologist and she spent a lot of time with elementary kids who were behind for various reasons but didn't technically qualify for special ed. Her school had the lowest scores in the district, so the state kicked in extra money to add more staff to support those efforts. Those are structural corrective actions versus the adhoc ones above.

Expand full comment

This is a very interesting point with an anecdotal resonance for me. In senior year in HS, I came down with a liver bug and had to be in bed-rest for nearly 2 months. It was awful because it was boring, but I also missed a lot of classes.

Because India has board exams (https://en.wikipedia.org/wiki/All_India_Senior_School_Certificate_Examination), schools start early in summer and finish teaching a year's worth of material in 2 months, and then spend the rest of the year in practice tests and so on. Sadly for me, I was sick for most of the classes. I taught myself most of everything, but Chemistry was my Achilles Heel - I couldn't make sense of it. I noticed most of my classmates were struggling with it just as much as I was and organized them to demand that the classes be taught all over again, and the plan worked - our teacher was young and filled with ideas about being a good teacher and she decided to teach everything again ;-)

At the end of the year, I got a certificate that looked like this for Chemistry (image not mine) - https://qph.fs.quoracdn.net/main-qimg-ae9cedf353e6c0414ae4024776130d95, which made my life very hard. Everybody wanted me major in Chemistry or Chemical Engineering or something like that - all I wanted to do was put Chemistry behind me (I didn't like it very much) and move on, which is what I did ;-) - I've now forgotten nearly everything I learned in that class 20 years or so ago!)

Expand full comment

While I think school is definitely overrated, and missing school underrated, a few comments:

- Presumably not only the amount of knoweldge retained, but also the "damage" from missing school varies quite heavily from child to child? Some children have positive effects from missing school, some children have negative effects from missing school. If "missing school" means "missing the science fair project you were going to win" then that's different to "missing maths class you were kind of average at".

- School is more than learning, missing school is probably less bad than missing "time to meet with friends".

- If everyone misses school, the social effects of a single child "missing out" can't be compared to the social effects of a snow day. If you used to be cool but your parents then stopped you from joining meetings with your friends maybe you become less cool? idk, I never understood how being cool worked in school, and the little I knew I have forgot.

- Lots of things vary from school to school, maybe in Argentina they have school figured out fully but not in the US (I kid, but this kind of thing makes it hard to run good studies).

Expand full comment

I would just quibble that social time with friends is definitely not one of the "Things that make school super important" when policy makers or whomever are discussing closing schools for one reason or another, or determining how much money schools should get. If the studies that find "Eh, missing school entirely doesn't mean kids don't get educated" are finding a true effect, that suggests that kids could spend even more time hanging out with friends.

Just hollow out the school into a giant warehouse, put in some comfortable chairs and couches, some tables, maybe a small library, you know, all the fancy dormitory rec areas from colleges. Fire all the teachers and just hire some reasonably functional adults to act as supervisors to keep the monsters at bay, and just have the kids hang out from 9-5. When they are older, maybe they could go and get part time jobs with their friends somewhere.

Boom, all the "more than learning" social friends time happens, money is saved, and kids have a much more pleasant and perhaps even more productive use of their time.

Expand full comment

I have no opinion on this, my thoughts on the relevance of social time with friends were in the context of "should I send my immunocompromised kid back to school?"

Expand full comment

If I may ask, what have you been doing with the kid over the past year and a half?

Expand full comment

I think it's a quotation from Scott's OP, not Matty's own kid.

Expand full comment

Derp, you are right. Sorry.

I am curious about what parents with immunocompromised kids have been doing with them the past year, though. COVID has been miserable for my three kids with not being able to go out and do things and see friends (less so since the oldest is just 7) but I am not worried about the health risks so the trouble is mostly things being closed etc. If I were worried about the health risks due to immune problems, that would be a nightmare.

Expand full comment

We sent our kids back to school as soon as it was an option mainly because they both really wanted the interaction with peers in person. The learning was going fine with online school. And I would say both of them were much happier after they went back and we were glad we faced our fears and sent them.

Expand full comment

I am glad our school was open last year, albeit with masks. I really hope they don't start doing remote this fall, especially since our second is starting kindergarten this year. Here's hoping.

Expand full comment

I suspect that you are entirely correct about grades and test scores as such, but what about harms from prolonged social isolation? I know of no studies on that, but my prior is that it will be a significant factor, especially for teenagers, and especially if they have less than ideal family support.

Expand full comment

There are probably better ways to fix social isolation for young people other than sending them to schools. Day camp or something.

Expand full comment

I was unschooled until I was 15, I'm pursuing a PhD now. Catching up on the basics wasn't easy but only took a few months. There are still a bunch of random general knowledge things I don't know, but at most it's caused a moment of embarrassment in social situations (e.g., when I genuinely thought dinosaurs were mythical creatures).

BUT I was motivated to catch up, which I think makes a big difference. I'd say most kids probably don't care too much about their education, so for them, missing school might matter more

Expand full comment

>when I genuinely thought dinosaurs were mythical creatures

Well I've never seen one!

Expand full comment

Wow, shame on your parents for not telling you about dinosaurs. If there's one thing about history that's universally interesting to kids, it's them!

Expand full comment

Not sure if you want to elaborate, but I'm curious what you mean by unschooled? No classroom or home education until age 15?

Expand full comment

NM, I read the comments before the article!

Expand full comment

Okay, this is tangential. Skip if you don’t want to read someone trumpet their liberal arts and science education.

You may be right about deleterious effects of missing school. But as a case study of the value of school (college), I got a liberal arts education with a double major in chemistry and philosophy. The chem helped me do my environmental chem work right out of school but the classes that I FEEL were most valuable include a course on Kant, a course on modernist epistemology, a course where we read Conjectures and Refutation, Against Method, and Lakstos’s Methodology of Scientific Research Programs. For whatever reason I also feel very influenced and enriched by a history and aesthetics of film class and a film comedy class. I later went to grad school in a different scientific field and then a career in science and was often told that my strength was critical thinking. Of course, we can’t draw causal conclusions. Maybe a talent for critical thinking drew me to my experiences. One thing I feel confident in is I encounter a lot of younger colleagues who can’t think. I cherish those four years. I know I am off point, but you mentioned not being a fan of school and I feel very different. Don’t know how it paid off economically (not too bad it seems) but in terms of what it feels like to be me, it was worth a lot to me.

Expand full comment

I think this is pretty clearly looking at K-12; college is a different beast (I can’t recall Scott’s writings or perspectives on post-secondary at the moment). What’s your perspective on your pre-college education?

Expand full comment

It sucked but that was a mix of me and the school

Expand full comment

Maybe they should talk about Kant, Feyerabend and Lakatos in K-12?

Expand full comment

I feel the... irony, I suppose might be a good word for it, of this post is that I took 0 classes on philosophy, read no philosophers, and yet my homeschooling gave me gratuitous amounts of critical thinking. Now this was because we argued for fun at the dinner table, I read books like Godel Escher Bach, and other such learning opportunities I had growing up. I presume that my aptitude for critical thinking is part inherent and part environmental, but none of it was related to a classroom.

Expand full comment

I am not sure there is irony here. I am sure there are homes and home schooling that can effectively instill critical reasoning. My father taught me a bit about Plato, Aristotle and, his favorite John Dewey and he liked to sit outside and read poetry (The One Hoss Shay was a favorite) or talk about the stars at night. Lucky to have that. Realistically, a lot of people can’t get that at home.

Expand full comment

I suspect that most people don't get that from school either, given my own experiences interacting with those outside my bubble. Which suggests that "a lot of people won't get it ever". I suppose the irony I was getting at is that you're touting that as a "great result of school" but it seems more likely that it was a result of "you as an individual being exposed to ideas, which you then took and run with" and that doesn't require school, or family, but an interest in learning and access to information (e.g. the internet). Formal training or no, those who have an aptitude for critical thinking will pursue testing it, those without are unlikely to pursue it.

On SSC, Plumber would often write about "simple work for simple folks" and I feel as though your position is falling into something of a typical mind fallacy. It's less "this wasn't good for you" but more "What did work for you is not the same as what works for others" and even more "Kids tend to be information sponges, but aptitude shouldn't be overlooked". For the year and a half or so I did attend public school (charter school), I felt as though everything was done at a snail's pace. As several people point out: lots of information is repeated year after year because kids forget. Perhaps there is a way to do school that would make it truly beneficial, but my experience indicates that we'd probably be better off if we didn't reiterate things so much.

Expand full comment

I am not trying to be dogmatic either way, but believe I benefitted tremendously from the back and forth with teachers and fellow students in many of my classes, philosophy but also chemistry and even calculus. The other thing that was super beneficial in my view was the socialization part - this from someone who grew up in small town Appalachia. Now I will also say I got a lot of beneficial socialization working at a Fortune 100 company with people from all around the world.

Expand full comment

In my case, I got a little from back and forth with a few teachers but more from extended political/philosophical arguments with a friend and still more from interaction with my father.

Expand full comment

One would hope so.

Expand full comment

Some of that forgetting, I think, is a feature not a bug. "Education" and "learning" seem obvious to adults but what the kids think they're doing may be something completely different. "See friends, sit quietly, do all the problems correctly, sit around, hear a story, eat with friends, watch someone get in trouble, sit with friends, write a sentence correctly, sit with friends, ride the bus home, see older kids yelling" is a somewhat accurate kid's-eye view of a school day and you'll notice that acquiring, retaining and analyzing information is not in the top activities; sit quietly and see friends are the top two. Math and writing occurred as many times as watching someone get in trouble and older kids yelling (once). This isn't a great model but I think it hits some main points - and this is why apprenticeship can be a good idea, the kids see what the skills are for and are motivated to continue by a desire to level up in regards to the mentor. Otherwise, "learning" is the word grownups say when they mean sitting quietly, seeing friends and people-watching. Part of the transition to zoom during Covid was the abrupt and peculiar discovery that "learning" had other meanings. I think a lot of kids felt cheated.

Expand full comment

I've experienced a lot of different kinds of school. Public school, private schools, charter schools, home schools, self-education, distance learning, family trade education. And none of them for more than a few years. I'm the product of a bunch of incomplete educations.

It definitely handicapped me academically in some ways. I don't feel it's handicapped me all that much in life other than getting through credentialed gates. I feel that the education I had in common family knowledge/trade was the most valuable, for what it's worth. Self-education probably second place. Third was on the job training and vocational stuff.

This makes me think the old apprentice and community school model might be better than what we currently have. Though I'm not particularly certain. Especially in a generalizable sense: it might just be it would have been better for me.

Expand full comment

I've attended ten different schools in my youth, followed by a bunch of different vocational education, distance learning, some odd jobs, the faint start of a career and now university, so I think I can relate. I think the main takeaway for me is that what matters is what I know and can do, as these will help me find my way in another environment, help me beyond the institution I am at right now. The ability to catch up by myself is also important - I do have odd gaps here and there.

I've been saying for years (tongue in cheek) that I'm in favor of more child labor. A lot of things only make sense to learn when you have need of them, yet school offers basically no opportunity for self-efficacy, and only distant promises of what it's for.

I learned lots of stuff I missed in school in a fraction of the time when I had use for it (this might not generalize if I am as smart as I think I am, but I think it applies to people less smart, too). There's also the fact that a lot of kids pass tests without any useful knowledge about the subject at all, purely by memorizing data meaningless to them, and guessing the teachers' passwords.

Expand full comment

Very much this. Learning is fun and easy when you plan to use it for something, and tends to be a boring grind when you don't even know what it is for. Even though I really like learning for learning's sake, the things I like to learn about, and how much actually sticks with me, suggest that just learning for learning's sake is not terribly effective.

Expand full comment

My experience with "Learning for learning's sake" is that there is a lot of value in "oh I've seen something like this before", but if that's the case, than the way we teach is completely wrong. School should be a smorgasbord of diverse experiences, with very little emphasis on any particular topic. Spend a week on carpentry, a week on plumbing, etc etc. I imagine that if K-8 was just exposing kids to various activities, we could then have 9-BS be actual training, and that training would be orders of magnitude more effective because the kids could sort into things they already found interest in.

Expand full comment

Oh, and it's probably worth mentioning that New Orleans improved relative to a Louisiana median that was itself a basket case, and so it's hard to draw meaningful conclusions from what happened there even independent of the large demographic changes in the city post-Katrina. Half of New Orleans's charter schools got the worst possible grades in the state's grading system in 2019, which again is relative to a terrible state baseline. Whatever "miracle" occurred post-Katrina it still left a huge portion of the system's students unable to meet basic proficiency standards.

Expand full comment

One problem with verifying your predictions is that standardized test standards keep getting lowered, or even outright eliminated (in favor of e.g. more diverse and inclusive metrics). Thus, comparing test scores today vs. five years from now might be comparing apples and oranges.

Expand full comment
founding

You wrote: "I think some of these points are stronger for poor/minority/at-risk kids, and weaker for comfortable/Eurasian/not-at-risk kids."

For the at risk kids, school closing is a disaster. Imagine a kid living in a homeless shelter or a kid whose parents are terrible role models or, worse, abusive, or a kid who doesn't have enough to eat. For those kids, school may be their only oasis of normalcy. Without school, they will at minimum suffer and might permanently lose a chance at a decent life. This has little to do with test scores and everything to do with socialization and having adults in their lives who care.

You probably leaned the 80/20 rule AFTER you left school, but I would say that 80% of the damage from school closings will be felt by 20% of the kids.

Expand full comment

Yeah, came here to say this. I remember articles from last spring talking about how all the rich kids would be fine, and the high school kids would survive (education-wise), but things could turn out really bad in the long term for elementary school kids from families with non-helicopter parents.

(Of course, to the extent that this article is “Contra Helicopter Parents on Educational Outcomes” it would still be correct, but it might be missing other real problems while disproving there fake ones.)

Expand full comment

The vast majority of kids living in those kinds of family situations do not attend a public school that you would calssify as an oasis of normalcy. Not a snark, my wife and my mother work at two of them.

Expand full comment

A lot of kids in DC missed nearly a year and a half of in-person school, and we don't know how many of them also had very little involvement with virtual school. We have some data on learning loss already, and the real story is probably worse because we're not tracking anything for the kids who weren't present enough to take tests. I agree with the conclusion of this, which is that if you're the kind of parent who is an SSC reader, your young kids are probably not going to permanently academically affected in any way by not having school for awhile. But from the perspective of 'should we keep schools closed this year' - which is the perspective I am personally terrified of right now - this is a disaster on a number of levels. We see the kids who were already the worst-off losing the most learning. We don't know how many kids just aren't going to come back at all. (Chronic truancy already having been a major issue, it's not like we're great at making teenagers go to school if they don't want to.) Our teen (and tween) carjacking sprees certainly aren't being helped by not having kids in school. And the message that our city government has sent parents who do think it's important kids be in school is so negative for trust in institutions.

Expand full comment

Hear, hear. I had serious medical problems in grade 5, needed a major surgery in grade 6, and was told I'd have to miss a year. My parents tried homeschooling, rigorously followed a bunch of curricula, and discovered I could finish *all* the assigned coursework in 2 hours/day and spend the rest of the time reading my favorite books. We were so unimpressed by the time wasted in "regular school" that we kept homeschooling another 2 years. I now have a PhD, but those were among the best days of my life.

Expand full comment

> I could finish *all* the assigned coursework in 2 hours/day

Yep. This is the origin story of basically every homeschooler.

Expand full comment

C.S. Lewis has a great comment near the end of The Lion, the Witch, and the Wardrobe, about the kings and queens of Narnia making "good laws" to stop trees from being cut down and to stop children from being sent to school.

Expand full comment

In "Prince Caspian" when Aslan goes around freeing the kingdom from Telmarine tyranny one of the first places he stops is a school building, which he magically turns into a forest glade. C. S. Lewis didn't like school, and for good reason. He was tutored by his mother until she died of cancer while he was young. Then he was sent to an awful boarding school in Britain, where the headmaster was suffering from some kind of mental disorder and only taught geometry: the rest of the "lessons" consisted of him randomly choosing kids to answer questions on various topics and beating them with a cane if they got it wrong. After the school folded (due to not teaching anything and having a crazy headmaster) he was sent to a much better boarding school where he didn't have a great time. Quoting from his autobiography: "Never, except in the front line trenches (and not always there) do I remember such aching and continuous weariness as at (school). Oh, the implacable day, the horror of waking, the endless desert of hours that separated one from bed-time! And remember...a school day contains hardly any leisure for a boy who does not like games. For him, to pass from the form-room to the playing field is simply to exchange work in which he can take some interest for work in which he can take none, in which failure is more severely punished, and in which (worst of all) he must feign an interest...Consciousness itself was becoming the supreme evil; sleep, the prime good. To lie down, to be out of the sound of voices, to pretend and grimace and evade and slink no more, that was the object of all desire--if only there were not another morning ahead--if only sleep could last for ever!"

Then after a couple years of this his father took him out of school to be educated by a private tutor. He writes about how his father tried to prepare him for the change: "He did his best to put all the risks before me: the dangers of solitude, the sudden change from the life and bustle of a great school (which change I might not like so much as I anticipated), the possibly deadening effect of living with only an old man and his old wife for company. Should I really be happy with no companions of my own age? I tried to look very grave at these questions. But it was all imposture. My heart laughed. Happy without other boys? Happy without toothache, without chilblains, happy without pebbles in my shoes? And so the arrangement was made. If it had had nothing else to recommend it, the mere thought, 'Never, never, never, shall I have to play games again,' was enough to transport me. If you want to know how I felt, imagine your own feelings on waking one morning to find that income tax or unrequited love had somehow vanished from the world."

So yeah, C. S. Lewis hated school big time.

Expand full comment

Hey, it's great to see another Lewis fan here! I'm not well-read on his autobiography, but I've been looking through the recent literature on boarding schools in Britain & Canada. It seems everyone hated them, despite being so posh and upper-class. It's a real puzzle that each generation sent their kids back, despite remembering very well what an awful time they'd had.

Expand full comment

The autobiography of Roald Dahl (the Charlie and the Chocolate Factory guy) had similar recollections of the misery of British boarding school, and he was there around the same time as C.S. Lewis (about a generation after). One thing that stood out though was that Dahl's parents were Norwegian, though his father had a successful business in Wales. Roald recounts why their family stayed in Wales even after their father died when he was just a child:

"Here she was, (my mother), a young Norwegian in a foreign land, suddenly having to face all alone the very gravest problems and responsibilities. She had five children to look after, three of her own and two by her husband's first wife, and to make matters worse, she herself was expecting another baby in two months' time. A less courageous woman would almost certainly have sold the house and packed her bags and headed straight back to Norway with the children. Over there in her own country she had her mother and father willing and waiting to help her, as well as her two unmarried sisters. But she refused to take the easy way out. Her husband had always stated most emphatically that he wished all his children to be educated in English schools. They were the best in the world, he used to say. Better by far than the Norwegian ones. Better even than the Welsh ones, despite the fact that he lived in Wales and had his business there. He maintained that there was some kind of magic about English schooling and that the education it provided had caused the inhabitants of a small island to become a great nation and a great Empire and to produce the world's greatest literature. 'No child of mine', he kept saying, 'is going to school anywhere else but in England.' My mother was determined to carry out the wishes of her dead husband."

Later he recounts how, miserable at boarding school, he faked appendicitis so he would be sent home. Once home the local doctor checked him out and the following occured:

"'You're faking, aren't you?" he said.

"How do you know?" I blurted out.

"Because your stomach is soft and perfectly normal," he answered. "If you had had an inflammation down there, the stomach would have been hard and rigid. It's quite easy to tell."

I kept silent.

'I expect you're homesick,' he said.

I nodded miserably.

'Everyone is at first,' he said. 'You have to stick it out. And don't blame your mother for sending you away to boarding-school. She insisted you were too young to go, but it was I who persuaded her it was the right thing to do. Life is tough, and the sooner you learn how to cope with it the better for you.'

'What will you tell the school?' I asked him, trembling.

'I'll say you had a very severe infection of the stomach which I am curing with pills,' he answered smiling. 'It will mean that you must stay home for three more days. But promise me you won't try anything like this again. Your mother has enough on her hands without having to rush over to fetch you out of school.'

'I promise,' I said. 'I'll never do it again'"

So it seems that people kept sending their kids out of a combination of the superiority of British schools (were they superior? They might well have been, compared to the rest of Europe at the time.), and the old British "stiff upper lip" ideal, where the the fact that schools were awful was a selling point since they would toughen kids up.

It should be noted that while C. S. Lewis hated the boarding school he went to (the good one, not the one run by a madman) his older brother Warnie loved it. As C. S. pointed out himself, he wasn't a very sociable kid. He might have had a better time if he was better at sports and made friends.

Expand full comment

I really want to read this C. S. Lewis autobiography, but I can't figure out which book it is. Can you help, FLWAB? (The question sounds dumb but he has so many books.)

Expand full comment

Hi Tam, I know "Surprised by Joy" is the title of the main autobiography. FLWAB, is that the only one? Is "A Grief Observed" also autobiographical? (After his wife died?)

Expand full comment

"Surprised by Joy" is his autobiography, although it's at least as much about his personal conversion experience as it is about his childhood and life in general. "A Grief Observed" is autobiographical I suppose but it reads much more like a series of journal entries while he was trying to come to terms with his loss. It is very short and I think not intended to be read by others, at least not while he was writing it. I believe he was convinced to publish it only much later, though I could be misremembering that.

Expand full comment

I’m taking all these passages from “Suprised by Joy.” It’s in the public domain (in Canada at least) so you can read it on Project Gutenberg here:

https://gutenberg.ca/ebooks/lewiscs-surprisedbyjoy/lewiscs-surprisedbyjoy-01-h.html

Expand full comment

I really like the section flow here, especially the straightforward statements of confidence levels at the end. Falsification might be the ideal and all, but once you've demonstrated calibration even the harder-to-check propositions go a long way towards clarifying what points you're trying to get across.

Expand full comment

This article cites a number of studies about various incidents that disrupted education, including Hurricane Katrina, the bombing of German cities during WWII, the Blitz, and the closing of the public school system in Prince Edward County, Virginia after Brown v. Board, that claim to have found long-lasting effects though I’m sure some of them suffer from the problems you mentioned: https://www.propublica.org/article/the-students-left-behind-by-remote-learning

Expand full comment

Kind of unimpressed with this analysis. You carefully look for confounders when you contemplate studies that show ill effects from missing school -- but don't do the same when you contemplate studies that show no difference. That's illogically asymmetric, but consistent with the normal human tendency to have a hard time seeing and testing hypotheses other than the preferred.

For example, if almost everyone right now *assumes* missing a year of school is A Very Bad Thing it follows all the parents/students/teachers/schools that can do so will work extra hard (like your burned-out friend) to make up for it, which would certainly do much to compensate for any intrinsic harm. But if people did *not* assume missing a year of school is bad, which is what you would like them to conclude, they might no longer do so, and hey presto ill effects will emerge as if by magic.

The inherent problem with this kind of study is that the real human beings underlying it change their behavior in response to the experimental conditions, so it is exceedingly hard to have genuinely comparable experiment and control groups -- the people in each group will *not* respond the same, even if they were hypothetically identical, meaning even if you could clone Parent A and Child A1, and put one copy of each in your experimental and control group, they would not respond the same, that being human nature. That's one good reason social science research is infinity times harder than doing research on cells or drugs.

Expand full comment

This is exactly why I try to even avoid this topic. I know I'm hopelessly biased against school because I personally found it pointless, so any study claiming to confirm this is something I'm going to silently and subconsciously file away as "evidence" without subjecting it to any kind of scrutiny, and anything that finds school matter is going to tempt me to tear apart the methodology and analysis to find every possible study design and statistical flaw I can.

Avoiding this trap takes a serious commitment to formal process. Subject every single study to exactly the same analysis, whether or not it passes a first smell test or not. Don't even apply a smell test.

But I am definitely not willing to do that myself because I already have a job and no one is paying me to write a blog.

Expand full comment
founding

>You carefully look for confounders when you contemplate studies that show ill effects from missing school -- but don't do the same when you contemplate studies that show no difference.

In most real-works situations, differences go down when you adjust for extra confounders, rather than growing. So Scott's analysis ok on that issue.

Expand full comment
author

"For example, if almost everyone right now *assumes* missing a year of school is A Very Bad Thing it follows all the parents/students/teachers/schools that can do so will work extra hard (like your burned-out friend) to make up for it, which would certainly do much to compensate for any intrinsic harm. But if people did *not* assume missing a year of school is bad, which is what you would like them to conclude, they might no longer do so, and hey presto ill effects will emerge as if by magic."

I'm not really sure why you think this is a confounder - most of the studies I looked at looked at kids who actually did miss school, not kids who were at risk of missing school.

Expand full comment

While I disagree with the argument, I believe the argument is that if a kid did miss school and everyone thought that was not a big deal, they would not spend the effort to take compensate for the absence. If a kid missed school and everyone thought it was a bad thing, they would do extra studying, homework, assistance, etc.

In both cases the kid actually did miss school, but their support network would react differently. I just discount the effectiveness of the support network in fixing the problem even if they did think it was a problem (even if it was a problem).

Expand full comment

Ugh, clarifying thought after I hit post:

And the reason I discount the effectiveness is because there are schools that are known to be bad schools, and if knowing something is bad would cause people to effectively compensate for it, those schools that are thought to be bad would be better than ones that are thought to be neutral.

Expand full comment

Yeah, I was shocked particularly by ignoring changes in student composition post Katrina, which was an incredibly obvious confounder to look for.

Personally, my prior is that almost all social science studies of this sort are just so much as noise (C.f. Replication Crisis) and that it's not worth drawing much of a conclusion from them either way.

Expand full comment

My daughter would have missed first grade last year. Instead we hired a teacher and gave her in-person lessons with a few other kids. I think the effect of missing that year would have been potentially very bad for her social development, *given her personality and how far along she was socially at the time the pandemic hit.* She of course would have made up some of this gap in the future, but I also suspect there would be some lasting consequences.

I of course can't be sure about any of this. And I'm not making the same claim for all kids, some (likely minority) of whom benefited socially from the school closure. Point is: school is more than just memorizing a bunch of facts.

Also, maybe worth point out that Scott doesn't just have an "anti-school bias." He is extreme on this issue ("SCHOOL IS JAIL") based on his own own idiosyncratic personal history, and I think we should massively discount claims that kids will just go learn what they need to anyway. *Scott* would do this, but he isn't the typical human.

I will also mention that I also harbor a fair degree of skepticism about the value of school, so I'm not coming at this as a diehard defender of education. But I think that a lot of critiques of the form that Scott makes are extremely glib, in the form of, "La di da, the effects might be worse for the socioeconomically disadvantaged." Yeah, no shit. It's easy for me to be skeptical of the value of school -- I went to prep school and Ivy League and have a masters. It's "obvious" to me that this was a form of credentialism that may or may not justify the tremendous expenditure of resources. But this is a very rarified perspective.

Expand full comment

I couldn't agree more with every point.

Expand full comment

To which I would reply: what points? We know Scott is biased against school: he has admitted as much not just elsewhere but in this very post. One’s ‘suspicion’ that there would have been lasting effects presents no actual evidence, and reminding everybody that educated people have privilege is… well, basically everything here is either obvious or already said in the post or both.

Expand full comment

I wish we could decouple the breakfast/lunches, social workers, support structure from the information/skill acquisition. I agree that the support, food and social workers are lifelines for many kids. But having it happen where the information is imparted is (to me) too many jobs for the same building, same teachers, same administration. Those functions are combined because that's where kids spend their time, other than home; it's convenience, not because those tasks are fundamentally complementary to each other. When "If they don't see their math teacher, how will we investigate the abuse?" is a reasonable question, there's an issue. A system redesign is unlikely, but if this family support is so necessary and important (and it appears to be) there should be a way to provide it in a more granular way outside of the classroom.

Expand full comment

I don't understand the argument. Are you saying that to less privileged students, an Ivy League education would be more than credentialism and they would actually learn something?

Also, if the effects are only bad for the socioeconomically disadvantaged, why don't we put an income limit on free public school? Then we could design the school to specifically help them. There are plenty of welfare programs with income caps.

Expand full comment

I think you're not trying very hard to understand the argument.

I'm saying "I don't remember the Spanish I learned in high school" isn't an insightful criticism of schooling, and the people who think it is aren't engaging this question seriously. I'm also saying that people who write somewhat unhinged screeds against school and then jot off a few paragraphs in which they claim to "steelman" the other side of the debate are engaging in a form of rationalist theater that characterizes a lot of this community.

I'm also saying that I'm really, really not interested in your "ideas" about schooling. I think it is a complicated topic that I'm pretty sure you know almost nothing about but still have strong opinions about.

Now do you understand?

Expand full comment

Why are you even here? You seem angry and defensive.

Remember, Scott is the guy who wrote a long paean to UBI and un/alternative schooling in SSC Gives A Graduation Speech. Do you expect every single article he writes to be about psychiatry?

What would you expect to see when someone steelmans an argument that with all the facts and reasoning they have in front of them they still disagree with? If he's missing something by him laying out the best case for the other side it is easier for someone else to know where to correct him.

Expand full comment

No, I'm not angry or defensive. I am, as I said, *uninterested* based on my strong priors about the expertise and biases of folks like Scott and probably yourself. I'm not sure what Scott's thoughts on UBI or alternative schooling have to do with his bad takes on school closures, but I am allowed to point out his bad takes.

By the way, we already place income limits on free public school, because schools are funded through property taxes, so parents pay for public school through housing prices.

This is a topic that attracts a shit-ton of dumb, uninformed opinions. Which honestly is not that big deal -- most opinions on most topics are dumb and uninformed, including my own -- but for the fact that everyone here seems to think that their opinions on schooling are super clever because they say the word "priors" a lot and probably had high SAT scores.

You claimed not to understand my argument, and I believe you, because my argument has to do with the ideological blind spots of this community. My argument is simply that this topic is extremely complex, that people like Scott have extremely biased views based on their extremely unrepresentative personal histories, and saying "blah blah steelmanning" doesn't change this at all. I also offered a counter-narrative based on personal experience that cuts against Scott's anecdotes about high school math. You don't have to agree with any of this, but it's just not that hard to understand if you want to.

Expand full comment

It’s not just him. You are obviously angry (categorizing a post that’s got scads more evidence than you have presented, which is zero, as ‘dumb and uninformed’ or an ‘unhinged screed’ is evidence enough) and your insistence on being insulting towards the community at large rather than engaging is obviously defensive. If you’re uninterested in these opinions, you don’t have to participate. If you are interested, which you apparently are, then you’ll have to endure challenges.

So my challenge is: what makes you think that your intuitions overcome this evidence? You have said that it’s dumb and uninformed, how is it? What is being missed?

Also, it seems plain that you misunderstood what is meant by an ‘income limit.’ He was talking about capping entry to public schools based on parental income. Rich kids aren’t barred from going to public school currently. I’m not sure what function you think property tax plays in entry to school, but it’s not relevant to the hypo.

Expand full comment

Homeschooling is living proof that missing even significant amounts of school has relative modest effects to most of the population.

Note the word “most”. I’d argue it’s brutal to those from dysfunctional homes and actually net negative for bright kids with neuroatypical backgrounds.

Certainly the slicing of homeschooled kids’ standardized test results prove that they generally excel vs public schools even if their parents are poorer, completely uneducated and motivated by very different things (religion vs anti-bullying vs educational quality)

Expand full comment

Homeschooling is not typically the same as just missing school. Even parents who aren't great at it typically still come up with some kind of plan for teaching their kids. While some parents certainly engaged in some kind of additional instruction with their kids during school closures, not a lot were finding or creating entire curricula or otherwise trying to act as a substitute for schools. (The substitute for in-person school was supposed to be virtual school, and in some cases it was and in others it wasn't.)

Expand full comment

I would say about 2/3 of homeschooling families have formalized or at least semi-formal curricula. But plenty have sparse or none - see the unschooling Scott referenced - and those kids do statistically the same as those of us who have their kids doing math workbooks at age 5.

Expand full comment

Yeah, so that unschooling subset is the more relevant subset -- to the extent that we think either of them is relevant because of the selection issues.

Expand full comment
author

I agree with Emily here - the unschooling study found that unschooled kids did worse than public schooled kids and homeschooled kids did better. These are two very different situations, even if they do kind of bleed into each other.

Expand full comment

I’ll track them down but I had seen multiple studies showing lower standardized test outcomes for unschooling vs classical or hybrid homeschooling curricula but not at a statistically significant level. If I can dig them out I’ll post here.

Expand full comment

I had friends who unschooled their daughters. Both were totally illiterate at age 8 and 9 respectively. They finally asked me how to teach the girls to read. They did learn after that. But really I think they could have spent their lives as illiterates if this hadn't happened as their parents had absolutely no idea what they were doing and the mother just spent the time playing with them, as she herself said. I don't think either of them had the slightest sense. But it was fun while it lasted.

Now, many years later, the daughters will not speak to the father but are passionate about social justice.

Expand full comment
founding

> What I would really like is some consistent absolute test (ie your score is how many questions you get right, not how well you do relative to others) that all schoolchildren take yearly.

This sounds like it would be really useful. Would also be nice to use that give that test at the beginning of the year to decide what to teach kids. One of the elementary schools I went to did this, and I was able to skip over the topics I already knew. Other kids benefitted from covering topics they still hadn't mastered or had forgotten over the summer. Seems like the kind of thing you would want to do if you thought what you were teaching in classes was important enough for students to actually know.

Expand full comment

The unschooling study probably doesn’t prove anything since it self-selects for parents of above average intelligence/conscientiousness.

Expand full comment

Citation? I would have predicted the opposite based on my own unschooling experiences in the South and Midwest. Maybe on the coasts it’s more of a social status marker?

Expand full comment

The MLA citation format stuff still makes me want to smash things. I remember getting crap from teachers and/or professors about incorrect citations and having . "Oh, you italicized the publication's name, instead of the article's title, like you're supposed to? Haha, dummy. Here's a half letter grade off your ten page paper about ancient Persopolis. Better score well on the GMAT son, because your grades aren't getting you anywhere in life."

They turn you into a monster, then they call you one.

Expand full comment

Reason # 478392745 to homeschool tbh

Expand full comment

I swear the whole way we teach how to write essays is a psyop. When I learned it in middle school and again in highschool it was so boiled down to a "science" that it left no room for individual expression. Not just what each paragraph had to do, but what each sentence WITHIN the paragraph had to do. This kills prose and creates really choppy, stilted papers.

Also, there was an extreme overstatement about how important the first sentence was. Constantly told by teachers that the first sentence should make them want to read it. You're the TEACHER, you are OBLIGATED to read the damn paper! This make the game of writing a paper always start with the hidden rule of "put an extremely hot take as your first sentence."

Citations were the ultimate rule mongering. Teachers played fast and loose with what they want cited, even in college. Supposedly, we don't need citations for things we just already knew, so I used that to put something about Ceasar in a paper that was primarily about the Jewish War and Josephus's respective account. I get the paper back, teacher asks where my citation is. My brain, I guess?

Also infuriating that putting citations at the end of the essay but not also having in-text citations somehow still counts as plagiarism. Call it bad form, but saying that it's plagiarism equates not having in text citations with wholesale copying, and that's just not fair.

Expand full comment

I believe this is because the algorithm they use to detect cheating flags it as plagiarism, because actually catching plagiarism is really really hard. Unless you're copying something that the teacher is already familiar with, take #257 from obscure author on a given topic is unlikely to raise any concerns with the teacher.

Expand full comment

I remember learning about paraphrasing vs plagiarism in high school and my takeaway was "oh, so we CAN copy stuff if we change it around enough?" I wasn't referring to citation websites, although they gave me a heart attack sometimes because it would see your citations are the same as another paper's and add that to the plagiarism score lol. I remember strictly being taught about citations and how plagiarism still occurs when there are end of paper citations but no in-text citations. I have my doubts that high school teachers even checked every citation.

Expand full comment

>Also, there was an extreme overstatement about how important the first sentence was. Constantly told by teachers that the first sentence should make them want to read it. You're the TEACHER, you are OBLIGATED to read the damn paper! This make the game of writing a paper always start with the hidden rule of "put an extremely hot take as your first sentence."

I know someone who did this well: "The Industrial Revolution and its consequences have been a disaster for the human race."

Expand full comment

Then the professional organizations change the citation methods every few years! My favorite is when the teacher insists the students use correct citation methods but then they themselves get it wrong on the syllabus.

Expand full comment
author

I want to stand up for having good topic sentences.

I think there's a lot of writing where this doesn't matter, because if you're reading eg a scientific paper you don't expect it to be well-written, and if you're reading a book you've probably committed to reading at least a chapter before giving up.

But for something like a blog post, I think being able to write a good first few sentences is pretty helpful. Maybe this is because there are a lot of people who are horrible at this, so being even a bit okay at it goes a long way.

Also, I don't know, there's something that's just a lot of fun about writing a good topic sentence. When I'm able to get something I'm really happy with, like on https://astralcodexten.substack.com/p/ambidexterity-and-cognitive-closure , I feel good about it in a way that can make up for a lot of deficiencies in the rest of the essay.

Expand full comment

If I wrote anything resembling a SSC blogpost for an English class, I’d get an F for not following anything resembling their formula and personal preferences.

Expand full comment

Not in my class! Over my last fifteen years teaching high school English, the model I present to my students has evolved (or devolved?) from the five-paragraph specified-sentence nonsense we're all ranting about to literally this: "An essay is smart statements that flow into each other and add up to something." Oh, and maybe hint that your paper adds up to something by having a snappy title pointing in that direction.

And no, "smart" is not some silly acronym. We know interesting/supported/insightful ideas when we see them.

Am I doing students a disservice when it comes to their future classes? Maybe. But I try to tell them what to expect.

Expand full comment

Honestly I learned to write more by just talking to random people and reading than I ever did in those terrible English classes.

Expand full comment

The entirety of school taught essay writing is a massive joke.

Expand full comment

I believe this is mainly because we don't know how to teach people writing. Either kids pick it up through osmosis, or we give them a formalized pseudo-writing that they can limp by with as an adult.

I have a college degree in writing, and only one class I took even discussed how to write or structure (and it was a perfunctory lecture on three-act-structure and rising/falling action).

(Or, more generously, we know the only thing younger kids can do is practice, so we give them a formalized structure so they can start somewhere and don't just sit blankly at a piece of paper thinking they have no idea how to write.)

Expand full comment

"We" absolutely do. Those we people are people who wright for a living. I learned more in 1 year on the job than I did in 27 years of education from kindergarten through graduate school. But sending my report to my boss and dealing with his redlines made me a good writer because he was a great writer.

If you aren't a great writer, you probably can't teach people to be a great writer. If you are a mediocre writer, you can only teach someone to be a mediocre writer. If you don't spend a significant chunch of your time writing (for example, because you are standing up and lecturing to children), you are very, very unlikely to be a good writer.

Once upon a time, schools used to at least use great writers as exemplars, but old great writing is yucky now. So five paragraphs and five thesis statements it is!

Expand full comment

of course, all my posts are from my phone and you aren't paying my rate so you get shitty typos and such. Sorry!

Expand full comment

We know how to teach math with only mediocre mathematicians (or human calculators). We can teach art with mediocre artists. Kids start off as bad writers. I would assume a mediocre writer could improve kids to a mediocre level and ready to pass on to the next teacher.

And if writing professionals really know how to teach people how to write, I would think some of them would develop curriculums that they could resell at great profit instead of just having a handful of nationally recognized workshops.

Expand full comment

Make the first sentence about a bomb in school. They always kept on reading after that!

Expand full comment

They should teach kids to use BibTeX

Expand full comment

I think we're experiencing cognitive dissonance over this because we expect US public schools to do what they say they're supposed to do (i.e. provide the most education to eligible students within the allocated budget).

When I look at the institution of US public schools, they do not seem well-tailored to this problem. I'd be hard pressed to find a single problem they *are* suited for. As best I can tell, it's a mix between providing free daycare (so parents can work) keeping kids busy (off the streets and out of the labor market), inoculating kids with desirable values, and teaching them a bunch of facts. I think, despite the best intentions of their constituent personnel, the institution is pretty terrible at raising people and preparing them for life as adults.

So I can't be surprised that a year (or five) of missed school is easy to make up, under the current system. Would it be the same if schools were designed to engage students and teach them useful and relevant things for their lives? I hope it wouldn't be.

Expand full comment

In my childhood, the comic strip "Peanuts" would caricature schooling as memorizing long lists of Presidents and Civil War battles, of course portraying it as dreary and pointless. But I would have loved that as a kid! I had all the Presidents memorized by the time I was 12, and I taught them to my brother who was only FIVE. (We're both still big fans of Presidential trivia to this day, and have taken several trips together to visit Presidential historical sites.)

But I learned them all on my own, for fun. Never got anything like that in school.

Expand full comment

I learned them for fun at a similar age, forgot 2/3 of it within a year, had to learn it in school and was annoyed at the drudgery, immediately reverted to the existing 1/3, relearned the rest a few years back, and hopefully it'll stick without maintenance. Human interest is weird.

Expand full comment
author

I learned them in math class, because we had to bring our own rulers, and my parents got me one that said "Ruler Of The Free World" and had a list of all the presidents on it (along with the normal inches and centimeters)

...actually, this explains a lot about me, doesn't it.

Expand full comment

In regards to Katrina: while some kids may have missed a year or two of school, the vast majority were back in school somewhere not too long after the storm. A study from RAND says that 31% of displaced public school students didn't return to Louisiana public schools during the 2005-06 school year, but that doesn't take into account out-of-state schools (or private schools). Out of the people who did return to school, the median time before re-enrolling was five weeks (so early October 2005, as many of the school districts in the metro New Orleans area started reopening). I was a college student in New Orleans at the time and I transferred elsewhere for the semester; I have a sibling who was in school for a few weeks in Texas during September, then at a different school in the New Orleans area for the rest of the fall, then her original (private) school in January 2006. That experience was pretty common for middle- and upper-class people in the New Orleans area.

Responding more directly to the point of the graph: the population changes caused by Katrina are likely responsible for a lot of the increase. From 2000-2010 the population of the city itself (coterminous with the Orleans Parish school district) shrank by 140,000, mostly due to Katrina. The drop of the African-American population was 120,000 of that. I'm not sure if year-over-year ACT scores by race are available for New Orleans public schools, but I'm guessing that if you control for race, it wasn't as much of an improvement as it seems. I also suspect that among black New Orleanians, the ones who didn't come back were much more likely to be poor and have kids who performed poorly in school. If you were a homeowner during Katrina, flood insurance payments and Road Home money helped. If you were a renter, your place may very well have been completely destroyed, your job was probably gone, and you might as well start over in Houston or Atlanta or wherever else you ended up.

On a broader note, I think it's pretty clear that the various disruptions of COVID have negatively impacted at least some measurements of student learning. The 2021 AP exams returned to a more typical format (they were shortened and changed in 2020), and scores were way down across almost all subject areas. The AP tests are designed to be absolute and not relative. True, people probably end up forgetting about Teapot Dome or King Lear or Aeneas' trip to the underworld; but insofar as those tests do accurately depict what students are learning in class, the 20-21 school year was not good for those students.

Expand full comment

Really informative comment, thanks!

Expand full comment
author

Thanks. While 2021 AP tests going badly fits with me "learning loss has short-term but not long-term effects", it sounds like AP tests are the thing to watch in a few years to see if there were long-term effects after all.

(unless the composition of AP classes changes for COVID or other reasons, which I guess it probably will)

Expand full comment

At the 50th percentile, 5 percentile points is 0.133 standard deviations. At the 98th percentile, that translates into less than 1 percentile point, so it would push you down to the 97th percentile.

Expand full comment

You seem to be making some rather strong (unstated) assumptions about an underlying ordinal distribution, presumably a Gaussian one. What are you basing this on?

Expand full comment

Sorry, I mean a cardinal distribution underlying the ordinal one.

Expand full comment

These kinds of tests are all normally distributed are they not? The SATs are for example.

Expand full comment

SAT scores, much like intelligence-test scores, are converted to a (binned) normal scale, but they might as well be converted to any other scale. I can't see any reason to assume that an effect would be constant on that particular scale.

Expand full comment

What about the central limit theorem?

Also, suppose the underlying abilities that determine achievement test scores(primarily IQ) in reality had a very different distribution, for example such that the difference between the mean and 1 SD above the mean was not of the same magnitude as the difference between 1 SD above the mean and 2 SD above the mean.

Then you wouldn't expect normalized achievement tests scores or IQ scores to *linearly* correlate with all kinds of outcomes, would you?

Expand full comment

But there is still no reason to assume that a perturbation would move individuals by a constant in that distribution, is there? E.g if we have a binomial distribution (which is almost what we're approximating with a normal), a perturbation that switches off a particular summand for each individual will affect individuals in different parts of the distribution very differently.

[1, 2, 3, 4, 5, 6, 7] and [1, 4, 9, 16, 25, 36, 49] have a linear correlation of 0.9553 – so I would expect linear correlations for non-linear relationships, although they may underestimate the strength of the relationship.

Expand full comment

One thing covid has convinced me of is that the most important thing about much of the school system is to give young people an organized place to be and to interact with each other. For high school and university, there's some amount that involves actual learning of significant skills that can be done via online interaction, but for younger children, the online interaction is the worst of both.

I expect that things like pre-school, summer school, and other increased time in school are very helpful ways to give families more support in childcare and socialization, and that these likely have all sorts of positive effects on the future success of these children (and their parents - has anyone tested future earnings for *parents* of Head Start children, or just the children themselves?) But not too much of it is through the "education" aspect of school.

Expand full comment

How do you account for negative social interaction, such as bullying? A lot of kids generally hate the social interaction they get in school but have healthy social lives with self selected friends outside of school. Besides, this role could be fulfilled by state subsidized child care centers that make educational materials available, but don't force them on kids.

Expand full comment

I agree that much of the role could be better served by a different structure.

My thought after this year is that it's important to ensure that schools are available to all children, but that individual children and their families can decide whether any negatives outweigh the positives. I've heard in a few places recently that bullying is much less common in schools now than it was 20 or 30 years ago, but I don't know anything about that myself.

Expand full comment
founding

Are you imagining that if they are not bullied in school, they will not be bullied ever? Or that a few years of schoolyard bullying is a greater harm than a lifetime of the extracurricular sort?

"How to deal with bullies" is a social skill that has to be learned somewhere. Same with all other sorts of social interaction.

Expand full comment

Yeah exposure to a little bullying is probably a good thing it builds skills for dealing with assholes that you will inevitably experience in real life. But just like anything the poison is in the dose. I honestly think one of the best things I learned from attending very diverse (income/ethnically/culturally) MS/HS was how to not back down when someone got in your face and tried to intimidate you.

Expand full comment

I liked being around a lot of people my age in high school and college. But:

Is the interactions that take place in a school better than ones that would take place if they were working together? If a student doesn't want to interact, do you feel like they should be able to opt out?

Expand full comment

They probably should be able to opt out? I haven't thought about this side of things as much, and I would want people who have spent a lot more time working with and thinking about adolescents to be answering that question rather than me.

Expand full comment

Also, what about skipping grades, where kids are deliberately caused to miss a year's worth of material (without any gap in schooling) and that's seen as a good thing? I skipped 3rd and 7th grades. I know I missed out on some biology and European history due to skipping 7th grade; biology has remained one of my weaker points, but I think I'm fine at European history, because it comes up a lot and I'm interested in it. In 3rd grade I missed...... cursive? Honestly I have no idea what I would have learned that year; IME before 6th grade they barely even attempt to pretend that they were teaching anything in particular. (I guess I should note that even though I skipped grades I didn't skip any math, and maybe that's important.)

Expand full comment

The relevant comparison would be students of equal cognitive ability who did not skip grades. Those who did not skip grades but were just as smart, likely have different personality traits that might indicate they aren't as ambitious. This might be hard to use unless you force students to skip grades.

Expand full comment
author

I didn't think that was a relevant comparison because my impression is kids mostly skip grades because they are "too smart for" (ie have already learned all the relevant information for) their previous grade. So this doesn't prove that it's not helpful to get the information, just that if you get the information on your own that's great. Things like COVID learning loss would potentially happen to kids who have not already learned all the relevant information.

Expand full comment

Unclear whether prediction #4 is intended to be for (math AND reading) or just (math). Suggest editing wording from "...and math" to either "...in math" or "...in both reading and math".

Expand full comment

I'm torn on the value of schooling:

On the one hand, I think it's important to expose children to big topics that they'd otherwise have no exposure to. I'd have no interest in mathematics, computer science, or literature if somebody hadn't showed me why those things are interesting.

On the other hand, that didn't happen in school - I spent 4 years on the quadratic formula, a minute part of mathematics. I read about a dozen novels about the black American experience, some of which were quite good, and most of which were basically trying to copy those good ones, but none of which I personally related to. I discovered the cool stuff in college and through the internet.

Ironically, the one benefit of schooling was the one they were trying to de-emphasize while I was there - the vital skill of shutting up, sitting down, sitting still, and concentrating on boring crap for hour on end. That's what really prepares you for the working world.

Expand full comment

I'm curious. I moved to the US shortly after learning the quadratic formula, but my impression was that it formed the contents of Algebra II (and only Algebra II). How did you end up spending four years on it?

Expand full comment

In Algebra I quadratics were introduced, in Algebra II they were the main focus, in pre-calc they were reviewed (for at least a couple months), and then almost every bit of calc that I did at a high school level was finding derivatives of polynomials.

In theory, the curriculum was a lot broader. In practice, well, it's easier to test various ways of finding the roots of quadratics than it is to test just about any other thing in math.

Expand full comment

Hm. That actually feels like the system working *well* (except perhaps for the need to spend a full year of Algebra II on quadratic equations in the first place). As in, an extra month of preview and revision on either end seems like a reasonable approach, and the fact that the quadratic equation recurs as a useful concept in other math classes is a sign of a decently-put-together curriculum that hangs together. (And perhaps at some level it's a sign that the universe likes low-order differential equations; after all, in physics you end up with quadratics because of F=ma.)

Expand full comment

I mean, I don't know necessarily that I'm saying U.S. schooling is particularly bad even - I think any schooling based on metrics is probably like that. Some kids get some things too fast, others too slow. I grocked new information quickly and had low tolerance for review which is a terrible combination. That may be a personal failing, tbh.

My bigger point was 1) the point of school is to expose people to a lot of ideas and see if any light a fire under them, and 2) it failed to do that for n=1.

And the "shut up and sit down" stuff sounds like I'm being glib but I'm really not. I had tons of teachers who recognized I was a special snowflake who was stifled by a classroom setting and tried their best to lower the obvious pain it caused me.

That's not a feature of the working world, and so their kindness was not a net benefit in my life :P

Expand full comment

I'm flabbergasted. 4 years on the quadratic formula?

Expand full comment

>This one is the classic, but its pattern of effects seems kind of suspicious - a teacher strike in fifth grade is bad for boys’ (but not girls’) sixth grade math scores, but a teacher strike in sixth grade is bad for girls’ (but not boys’) sixth grade reading scores? A strike is four times worse in fifth grade than in sixth grade for reading, but about the same for writing? Possibly I’m misunderstanding this methodology, but otherwise I’m not sure what to think.

This just looks like run of the mill p-hacking.

______________

If there is some impact from teacher strikes ... couldn't a negative impact on teacher quality explain worse student outcomes better than the small period of absence? A bunch of teachers striking and getting some permanent concession about retaining bad teachers could plausibly impact every generation that goes to that school. And you've already conveniently done the research on teacher quality: https://slatestarcodex.com/2016/05/19/teachers-much-more-than-you-wanted-to-know/

A future earnings drop sounds suspicious, but a permanent drop in teacher quality would show up decades later.

Expand full comment

On the other hand, a teacher strike leading to a pay raise could lead to teachers that are more engaged, having a positive impact on teacher quality.

Expand full comment

That isn't really relevant. We aren't trying to figure out why a teacher strike led to an increase in student achievement.

Expand full comment

"And they made good laws and kept the peace and saved good trees from being unnecessarily cut down, and liberated young dwarfs and young satyrs from being sent to school..."

~ the laws of Narnia under the kings and queens in C. S. Lewis's "The Lion, the Witch, and the Wardrobe"

Expand full comment

Reminds me a bit of Arthur Jensen's summary of "Isabelle":

"The effects of maturational readiness were seen most dramatically in the famous case of Isabelle, who, from birth to age 6.5, was reared in a semidarkened attic by her deaf-mute mother, without any other social contacts (Davis 1947). When she was finally discovered by the authorities (age 6.5), she was incapable of speech, acted much like an infant, and had a Stanford-Binet mental age of 1 year 7 months. Once she was placed in a normal social environment, however, her rate of learning was far in excess of that of the average child of the same mental age. She quickly learned to talk and acquired vocabulary with phenomenal speed, in fact, at about the rate that would be expected for a child with an IQ of 300! But this incredible rate of learning lasted only until her mental age caught up with her chronological age, or maturational level, at about age 8. Within two years she had advanced from a mental age of 1 year 7 months to a mental age and level of scholastic performance on par with her 8-year-old classmates."

Source: https://sci-hub.do/https://doi.org/10.1016/1041-6080(89)90009-5, page 45

Expand full comment

I've heard that cited in arguments against early education. I can't remember who said it, but it was something like you can struggle for weeks to teach a 4-year-old multiplication, or wait until the kid is 8 or 9 and teach them in a day.

Expand full comment

Something interesting I noticed was that in The g Factor p. 113 he calls her "Isabel" but in this article he calls her "Isabelle":

Excerpt: "...the well-known 'Case of Isabel' is a classic example. From birth to age six, Isabel was totally confined to a dimly lighted attic room , where she lived alone with her deaf-m ute m other, who was her only social contact."

Expand full comment
author

I thought I had heard the opposite, about lots of feral children who missed a critical period and never picked up language or other things correctly. Am I imagining this, or are there cases on both sides? Has anyone ever compared the cases that do well and the ones that don't to see if there are differences?

Expand full comment

This review article from 1984 found that it's a mixed bag; some kids wind up learning and socializing pretty well, and others don't. Their conditions are so disparate and the sample-size so small that it's hard to make any firm conclusions: https://www.researchgate.net/profile/J-Smith/publication/232419212_Feral_and_isolated_children_Historical_review_and_analysis/links/550b2b1f0cf265693cef6bcf/Feral-and-isolated-children-Historical-review-and-analysis.pdf

Expand full comment

This is a discussion I have with my wife a lot. We have some minor disagreements, but our general feeling is that the educational value of school, meaning the facts you learn, is less important than learning to jump through the right hoops and get along with people. I guess it's a discouragingly Prussian way of thinking, but learning to be a good little citizen helps your prospects in life. It's nice if you can learn some basics along the way. I think in the higher grades it's critical to learn how to interact with your peers, as you form and break friendships and start working through romantic relationships a little bit. Those interactions happen at school, for the most part, and they're a big part of why we send the kids.

My kids had a rough year last year, including with their grades, but the worrying thing was more that they got more isolated and depressed. Maybe in the absence of formal, mandated education there would be another way for kids to really stretch their social skills. But in our system, it seems to happen at school or not really at all.

Expand full comment

The idea that most of school's value comes from kids work through social situations strikes me as plausible. But if that extends to romantic relationships as well, the implications are damning for single-sex schools, which I thought generally came out looking pretty good from all the available data.

Expand full comment

I think the ... math, and the learning to read and write, and then later the history and literature and science might be a bigger part of it. Math isn’t exactly a series of facts, but a complicated way of understanding and manipulating things and ideas. Same for reading and writing. We could just put kids in a big grassy field every day for ten years if socialization was all that’s necessary.

Expand full comment

I was homeschooled. I grew up in a social network of homeschoolers. They weren't any more (and generally were less) maladjusted to the real world than the young adults I knew who went to public school

Expand full comment

The oddest aspect of the debate around covid and schooling is that the people who most strongly defend the importance of public schooling are the ones who most want to close/limit schooling.

I would estimate that my kids (4 ranging from 4-10) have had about 3 months of schooling since March 2020 (we’ve supplemented). We live in one of the best public school districts around. The exodus to private schools has been massive. If schooling matters the oncoming inequality will be massive.

We see kids at activities outside of school regularly. The parents who are most vocal about restrictions in school don’t seem to demand the same restrictions anywhere else. Such an odd dynamic.

Expand full comment

As a teacher, I agree with this. Perhaps I'm burned out and cynical, but it seems to me that school is mostly a holding pen for children while parents go to work. Certainly if there was one theme from how parents reacted to covid, online schooling, and everything else, it was that the primary tension came from how disruptive it was to the parents' lives to have no place to send the kids during the day.

I think there's a performative need to pretend to really care about what actually goes on at school, because 1) parents treat it as a proxy measure for their validity as parents, 2) as a society and sometimes personally we spend a bunch of money on it, and 3) politically we throw around the idea that education is the solution to all of our social problems. Oh, we could have beat covid if only the population were more educated! And global warming, and poverty, and racism, and and and... As if.

(aside: I imagine Biden's Child Tax Credit thing where he sends every parent ~$3000 per year per kid no questions asked will do more to solve social problems - including educational achievement - than spending double that amount on schooling would have done. Turns out the way to solve problems is to do something about them, rather than pretend to teach the next generation to solve them and then wait 20 years to see if they do.)

But anyway, when it comes down to it, I think it's pretty obvious just from interacting with people in society that the vast majority of people aren't really learning what school is teaching, and the world seems to sort of plod along anyway. Sure, there are some edge cases of kids like me who learned a lot in school, but I feel like we would have learned a lot anyway. But for most people it won't matter much if you've "fallen behind" on the stuff you're going to forget anyway because it didn't matter in the first place whether or not you learned it - we just pretended it mattered in order to defend our individual and collective egos from having to take a hard look at what it would actually mean to be a decent parent or a decent society.

And yeah, closing any social service will hit the socially vulnerable the hardest - but again, schools were never the best way to solve social problems. If we wanted to, we could still help the socially vulnerable without giving their kids covid.

Expand full comment

In many sci-fi novels, the author can help the reader suspend disbelief about some improbable technology by using the words "quantum" or "tachyons". The same magical thinking can be enabled in politics by the word "education". Any social problem is fixable if you can just cut out the human behaviors you don't like. "Education" is the mechanism people fix on to create those changes, even though it almost never works that way.

Unrelatedly - I think the $3,000 per year per kid proposal seems like a bad idea which would have lots of unintended consequences. You get more of what you subsidize, and it's pretty difficult to find a way to subsidize *parenting*, instead of just *having a child.* A lot of the school system can probably be understood as an centralized attempt to provide at least some measure of "parenting", however crappy and at whatever expense, to every child.

Expand full comment

Gotcha, thanks for the correction. Also looks like it's been a policy for a while, recently expanded to $300/mo and with some conditions lifted. I guess what I'm saying just reduces to the same Moynihanian points people have made about the welfare state for the past 50 years.

It is interesting to me that this made its way into the COVID relief bill with no sunset clause; I wasn't aware how much long-term policy was snuck into the relief bill.

Expand full comment

> You get more of what you subsidize, and it's pretty difficult to find a way to subsidize *parenting*, instead of just *having a child.*

Giving up a second job because the child tax credit made up the difference in needed income would be a big step to better parenting. I guess we'll see what comes out of it in a couple of years.

Expand full comment

I think a lot of people are really hoping to use this to backdoor in a UBI, and get all the benefits that a UBI gives.

And they are about to find out that the UBI is not all it's cracked up to be. Lots of people in rehab completely failed as soon as they got their checks a year ago during lockdown.

Expand full comment

Some of that $3000 tax credit is going directly to private schools, tutors and homeschooling resources (in my small sample). I don't know what the fraction is across the board but I hope someone studies it later. I supported the policy.

Expand full comment

Biden set it to 3k (or 3.6k for under 5 or under 6), but Trump set it to 2k (only up to 1.4k refundable). And before that it was set to 1k, but there was also a deduction on top of that. Generally I have to remind people who swoon over how much the credit is needed and going to help people that it existed prior to Biden.

Expand full comment

Thanks for that - I think it's important to help families with kids, and depoliticizing it to whatever extent possible is probably helpful.

I'm also hopeful that this will move us forward on UBI, but we'll see.

Expand full comment

"holding pen for children"

I was expecting much more of a pushback against schools post-Covid, as people either (a) realised they didn't need schools to house their kids or (b) got angry with schools for not taking their kids. I'm not quite sure what to make of the relative lack of big changes.

It seems to me that school potentially serves three functions: (1) daycare (2) social training (3) teaching academics. The genius of school as an institution is that it serves these three needs simultaneously, and that's why schools are so popular. Like you, I think that the daycare function is much more important than many people admit.

Expand full comment

I think people have a remarkable lack of imagination when it comes to the arrangement of society with respect to schools. Here's an example: it's an uncontroversial result in social science that later school start times improve student academic, physical health, and mental health outcomes with no downside, at least for kids in middle and high school. So far I think the only thing we don't know is when diminishing returns kick in - e.g. 8 am is clearly better than 7 am, and 9am is clearly better than 8 am. We don't know if 11am is better than 10am, and if so, by how much, but only because no one has been radical enough to try to study start times that late. But as late as we've ever gone, it's been better than starting earlier.

As a parent and teacher I've advocated for later start times at every school I've worked at an also in my social circles. It's never been taken seriously. Almost no one wants to entertain the idea at all, and those who do point out that changing schedules would be inconvenient. The kids won't have time for as many after-school activities (if ASAs are in fact more important than academic outcomes, just shorten the school day), or the parents would have to leave the house for work earlier than kids leave for school (maybe I'm biased as a latchkey kid but your kid can lock the door on their way out of the house, I did this starting from grade 6). Like none of this at all even comes close to excusing the fact that we are making our kids sicker and dumber by making them get up in some cases before dawn to get to school.

And then during COVID my school moved its schedule to a 10am start time because, as an international school, we had kids from all over Eastern Europe and some of them who'd had to go back to their home countries were in a time zone two hours earlier than ours and we didn't want to make them start at 6am their time. Everyone liked starting later, everyone got more sleep, and nothing broke. But when it came time to reopen... my school just said "welp we're going back to 8am" and again wouldn't entertain the idea of maintaining a later start time even though it had worked fine and all our kids had stopped complaining about being tired all the time.

So it turns out no one cared even a little bit about the academic or social outcomes, which are proven better with a later start time. The one and only priority was making school line up perfectly with the parents' workday such that parents had a minimum of friction with getting kids to and from school. Literally nothing else mattered enough to even be worth talking about. This is one of many reasons why I have grown cynical to the breaking point with the entire modern Western educational system and virtually everyone involved in it.

It's true that some schools have been slowly moving towards slightly later start times in acknowledgement of the overwhelming evidence that they had been systematically making kids sicker and dumber with no upside, but there's been no serious effort to actually find the best start times and optimize other aspects of scheduling (e.g. recess, arts classes) to improve actual student outcomes. Instead, later start times, when implemented at all, are treated as a begrudging concession to the need to reduce the harm done by schools, rather than as part of a proactive effort to make schools actually be good for kids.

So much of what schools do is this kind of thing - paying lip service to harm reduction - and as a teacher it's impossible not to see it. I think this is one of the primary drivers of teacher burnout, but maybe that's just me.

Expand full comment

AP tests happen in early May nationwide. There's significant variance in when American high schools start (early August - early September), so some students get a full extra month of class time before the AP test (and those of us who started school in September get a full month with basically no instruction—my AP calculus class started a DnD group). If seat-hours matter, one would expect matched groups from school districts that start early to get meaningfully better AP scores than school districts that start late. I don't know whether anyone's looked into this, but it could be an interesting source of data.

The only thing I found with a very cursory google was this story, which suggests that a later start (how much later?) didn't change this school district's AP scores: https://www.argusleader.com/story/news/education/2017/08/15/late-start-date-didnt-stop-ap-students-scoring-well-exams/567997001/

Expand full comment

I think earnings is the wrong measure for educational value. It's pretty close to zero-sum - there are only so many positions, and either one person or a different person will get each one. We could chase after education systems that lead to higher earnings for their students, but that just means all the schools not implementing those systems as successfully will produce more students who wind up in *lower*-paying jobs than otherwise, as they're being outcompeted. I mean, yeah, I'd prefer my children be educated in the system that is most likely to guarantee them financial security. But as for how society at large should improve education, I'd rather focus on producing students who are better able to participate in society more generally: by being more informed voters, having better ability to discern which forms of evidence are meaningful, being more capable of expressing their views to others and understanding the views of others, etc. There is already an excess of qualified applicants in virtually every job requiring education.

Expand full comment

Why do you think earnings are zero-sum? Do you imagine per capita earnings are the same in 2021 as in 1980? In the U.S. as in Mexico? Insofar as earnings are a measure of productivity, increased earnings mean the society is becoming more productive.

Expand full comment

No. I should clarify - I don't think earnings overall are zero-sum. If you can raise average earnings globally, that's non-zero-sum. It's more that I think that when we use earnings as a measure of comparative educational effectiveness, we're often measuring something other than *total* earnings/productivity increase. More like a localized earnings increase, which I suspect is offset by other earning decreases among the segment of the population whose education made them relatively less competitive.

Our economy can only handle so many people in "good" jobs at any given point in time. We can't all be doctors and lawyers and financiers. I think that often an earnings gain attributed to an educational intervention is actually just getting the benefitting students slotted into the "good" jobs at a higher frequency than those without - but those jobs don't actually gain much of anything by the very marginal difference between choosing one candidate over the next best candidate, and thus don't actually gain in total productivity.

Expand full comment

Like, they have to pick one candidate or another, and they're going to have to use some criteria to pick between them, even if it's not actually a useful or predictive criteria on the whole. It's just by necessity that they only pick one of the two candidates, both of whom would in reality end up performing equally.

Expand full comment

That seems wrong on three levels.

1) If the quality of the candidate is improved, the candidate they select adds more value to the company, which allows the company to grow and hire more people. Generally having better candidates and then sorting them into the appropriate positions raises the water line.

2) If a candidate is higher quality, they won't just get the job, they'll get raises and promotions. It's about more than just the immediate conflict for the job.

3) If everyone was as at minimum as competent as a moderately competent doctor/lawyer/financier, then sure we'd have to go through some temporary growing pains to figure out how to staff jobs that require less competence, but also many of those jobs would get a lot more efficient, very quickly, and there would be a lot more good jobs. If everyone was at most as competent as the median (insert entry-level job here), then we wouldn't have any doctors at all.

Now if it is just credentialism, then sure, it's mostly zero-sum (ignoring any sort of positive gain by allowing the organization to sort candidates more effectively). Maybe that's your argument, but if so, why do you think a system that can't educate to improve competence would be able to educate to create good citizens?

Expand full comment

Of course I think certain systems can educate to improve competence better than others. I just don't think increased earnings is a good measure of education system effectiveness because it's hard to know whether those increased earnings are due to positive-sum competence improvements or due to zero-sum (credentialist, e.g.) improvements. If it's 100% due to worthless credentialism, that will still show up as increased earnings. So, as you can probably tell, I am usually skeptical of the competence gains explanations in the absence of evidence specifically pointing to that.

Expand full comment

This just isn't the case. I have worked in high tech (Semiconductor Design) for 30 years. At every job I have worked at, at any time, independent of the economy, our hiring position was to hire-any-superstar. Basically, the best employees will earn you more money than you have to pay them. In fact, they can accomplish so much that you need to hire additional support staff.

This is how Apple and Google and thousands of other tech companies got rich. Talent creates products which generates wealth. There is no "cap" on available high paying jobs.

Expand full comment

Do you really believe that tweaking education systems converts otherwise-regular people into the superstars you're talking about? My sense is that people like that are pretty much born, not created, and that the particulars of formal education have little effect on them becoming who they become. I doubt the earnings increases you see in the studies that do find them are a result of producing more such superstars, and are instead driven by everyday marginal competitiveness gains of the higher-earning group relative to the lesser-earning group. My view is mainly theoretical at the moment, and I could be convinced it's wrong by empirical evidence.

Expand full comment

For superstars, yes, actually. Putting them at say MIT or caltech with all the other superstars, big research labs, and the smartest professors and experts does in fact massively benefit them. And that does then lead to earnings increases. Even though Ramanujan was a genius, and figured so much out on his own (__bur with the help of a library, and with arithmetic taught in school__), he began making many more real contributions at a university when around many other genius mathematicians.

Expand full comment

I don't think this really relates to the point I was trying to make. I don't think "tweaks" to education systems create more Ramanujans. Ramanujan was not an "otherwise-regular [person]". He was clearly naturally endowed with extraordinary intelligence - a born superstar, just like I said. In fact, he in particular is an example of someone whom a slightly better education system simply could *not* create out of an otherwise-regular person.

Expand full comment

I had a long argument with Robert Frank on my blog having to do with this issue:

https://daviddfriedman.blogspot.com/search?q=Robert+Frank

(Includes some exchanges on other topics)

Expand full comment

> There is already an excess of qualified applicants in virtually every job requiring education.

This is an extremely false statement. In jobs that require extreme math or science or engineering or coding talent and technical skill and intelligence, there is an extreme lack of qualified applicants relative to every potential job. Hence, supply and demand, ML engineers and really good code guys making piles and piles of money. Every tech startup and company is hiring, and at high salaries.

Expand full comment

There is a grave shortage of good teachers. I can tell you that for a fact.

Expand full comment

When I first looked at the post I missed the word "Missing," which made the subject even more interesting.

A couple of observations from an unschooling parent:

1. Our kids were initially in a very small private school run on unschooling lines. One year some of the kids wanted to learn math. They started at the beginning of arithmetic, ended in algebra. It was a small mixed group of kids of varying ages and talents.

2. In college, our unschooled daughter got along better with faculty and staff than with fellow students. She was struck by the fact that when a class she liked was canceled for a day, the other students were happy. She was bothered by the fact that she was being asked to write papers which would ever be read by only one person, and only because it was his job to read and grade them.

Oberlin, where she spent her first two years, has a one month winter term during which you can do any educational project a professor will sign off on, not necessarily on campus. Her second year she came home and translated an Italian recipe collection from the early 15th century. It's now webbed. That was the sort of thing she thought she ought to be doing, given her educational background.[http://www.daviddfriedman.com/Medieval/Cookbooks/Due_Libre_B/Due_Libre_B.html]

Expand full comment

I, too, have been bothered immensely by the fact I had (and have) to write papers to be read by only one person, and one that considers it drudgery (and has no interest in having me rewrite it better). The only thing worse than that was having my papers read to the class, who were resentful and/or bored by it. Other schoolwork is not dissimilar: nobody needs me to calculate this or that, define that term or summarize that theory. Why should I put my best effort into something that only serves to raise expectations for my next meaningless exercise?

In my spare time, I map for OpenStreetMap (mostly via StreetComplete) and proofread for Project Gutenberg, and found that I do so most often when I am in dire need of doing some tangible useful thing that others can see, and that someone should be doing. These are the quick fixes when longer term efforts have been fruitless for a while. We all want to be useful, for our efforts to have meaning, and that doesn't just start when we enter the workforce (where this isn't given either, often enough, but at least someone has use for us, enough to pay us, then). School doesn't harness that need, and students are painfully aware that even attempts to offer some self-efficacy (mostly via art projects where kid's art is shown in public places) are basically occupational therapy, more about making them feel good then having a real use.

Expand full comment
author

Have you written about your family's schooling/unschooling experiences somewhere? I plan to have a kid within a few years (and some friends who also respect you a lot have preschool-age kids now) and would be really interested in hearing how you did things.

Expand full comment

Mostly on my blog, but also the final chapter in the third edition of _The Machinery of Freedom_.

Expand full comment

I have read them years ago!

DF has 2 posts on homeschooling. The theory and the practice. I've sent them to so many people! Superb concise posts.

Expand full comment

Scott—candid stories from grown unschoolers can be found on grownunschoolers.com, a site I helped create. I've also got some good stories accumulated in my 2022 book, Why Are You Still Sending Your Kids to School?

Expand full comment

> my own experience with these standardized tests is that eg my school’s social studies courses would move leisurely through the Native American and colonial era, get distracted talking about How Bad Slavery Was, and then a week before the standardized test go into “ohmigod we’re supposed to be at the 1950s now fuck fuck fuck Grant Hayes Garfield Arthur Cleveland Harrison Cleveland McKinley World War One World War Two labor unions Japanese internment ok good luck!” mode

This is hilariously accurate to my experience as well.

Expand full comment

Yeah, even in my day it was "oh heck we're two weeks from the end of term and there's a huge chunk of the curriculum we haven't even touched yet, okay it's cram cram cram from here on out".

Expand full comment
author

How far did they get in Irish history classes? There's so much of it!

Expand full comment

Ah well, in my time at secondary school which was *mumblemumble* years ago, the modern history part of Irish history only went up as far as the 50s (Séan Lemass and our first 'economic boom' which, like all our economic booms, was short-lived).

I'm sure since then the curriculum has at least caught up to the 60s and the Civil Rights Movement in the North 😀 But modern history isn't that important, once we've passed the elements of the Easter Rising, the War of Independence, and the Civil War (all in the 20s).

It's a bit like Dara O Briain's routine here (3rd to 6th minutes): "Ah, he left a lot out" https://www.youtube.com/watch?v=MqPzDWwuZn8

The main parts are the Golden Age (pre-Norman Invasion, Brian Boru and the Battle of Clontarf the most important part here), The Norman Invasion, and then a list of "stuff that happened next door that affected us" (e.g. Henry VIII, the plantations under Elizabeth and James) and "uprisings that didn't succeed", THE FAMINE/AN GÓRTA MÓR, until we get to the Easter Rising ("technically did not succeed but was a moral victory"), the War of Independence ("holy crap, it worked!") and then immediately into the Civil War (because this is Ireland, of course we turned on each other the minute we kicked the Brits out).

The European/World history part of the history curriculum is the remainder of it.

I remember for the Intermediate Certificate examination (now renamed the Junior Certificate, national state examination for 15 year olds) we had four history textbooks - 2 for Irish history (named "Conquest and Colonisation" so you get the drift there) and 2 for European/World history. The night before the history exam, I decided to revise by reading all four textbooks through and it turned out lucky that I did so, because one question on the paper was about something I had read the night before!

Sample timetable for this year's Junior Cert: https://www.examinations.ie/misc-doc/BI-EX-38587703.pdf

It tended to be all subjects, not just History, I remember it very strongly in Maths: "okay, end of the academic year is in three weeks, we have a lot of the curriculum left to plough through, so we are absolutely going to *gallop* forwards and if you fall behind, tough luck, we haven't time to pause and go through it again for you".

Expand full comment

Same here for middle school! In the last week of eighth grade, we covered WW1 to the "present," but despite the teacher's best attempts, we only made it to the Vietnam war. I'm a millennial, so there was plenty of history between the Vietnam war and the then-present we didn't cover. My high school teachers did a better job pacing, perhaps because I generally took AP history in high school, and the teachers were presumably under pressure to have high average AP scores.

Expand full comment

Why is it considered so very terrible if a kid has to repeat a year of school? Okay, so they'll be 19 instead of 18 when they go to university- so what? Over here kids will repeat the Leaving Cert if they didn't get enough points for the course they wanted.

I can see the problem would be if an entire class needs to repeat, because now you have two classes of the same year. But if everyone is repeating a year, that will sort itself out, surely? This kind of hair-tearing panic about "their life will be ruined!!!" doesn't seem justified, since it's an unusual event due to the pandemic and every child who was kept out of school, not just your kid, is in the same boat.

And frankly, if your kid is going up against the kids of people who can easily afford cram schools and grinds and six extracurriculars calculated to get them into The Right College, they're not competing on a level playing field anyway so a year more or less is not that much of a difference.

Expand full comment

An extra year in school can really matter for people who pursue medicine or PhDs, especially if they're women. Under a typical timeline, you'd get a bachelor's degree at 22 and an MD at 26, and then you'd have something like 3-7 years on residency. So depending on your choice of specialty, you might be 33 by the time you could practice medicine fully. If you want to have two or more children and don't want to deal with a baby during the stressful years of residency, you're already on a tight timeline. An extra year in school makes the time pressure even tighter. I think the numbers work out similarly for academia when you consider getting a master's degree, a PhD, and then doing a post-doc or two, especially if you want to establish yourself as a professor by publishing a strong body of research for a few years before having kids. And both sets of numbers are computed assuming you don't take any time off from school or change your path from your initial plan when starting undergrad.

Expand full comment

It sounds like you're not American, so perhaps doctors can go straight to medical school without a bachelor's in your country. But I think the number of years required to establish oneself as a professor are pretty similar worldwide.

Expand full comment

Very good point!

Expand full comment

My son went to a serious Montessori school from age 3 to 11. He finished all their math and science material early, went through their grammar and history and then Latin. Maybe 3 hours a day is the longest work day they had.

He socialized with the other kids, and that's all the school really provided. Some of the peers were real bullies, but he seemed happy. He was not a troublemaker so the teachers completely ignored him.

When he turned 8, we suddenly thought he ought to do something more structured. There was a class Stanford University offered online, for gifted kids. He did their set theory class with great relish...maybe 20 minutes daily.

We also took him to weekend (once a month) outreach programs at our local university - specially in math, a little in computer science (they met rarely) and he went through the Cambridge Latin book on his own...

All that matters is motivation, was our vague philosophy. That can come from not forcing kids to do anything. Get poor quality stuff out of their environment. Maybe no screens. (The school seemed pretty zealous about that). With screens available, kids seek those bells and whistles all the time. We were pretty serious about ourselves having little to zero screens.

He refused to work on his handwriting. We did not see a way to fix that. Maybe a traditional school would have forced him to.

His elementary years spent in this fashion were actually pretty amazing intellectually. And laid a great foundation. He had fun. He made up his own set of meanings for various arithmetic operators and we had to help him unlearn (happened quite easily). He also designed menus for an imaginary hotel business for months.

I think our biggest issue at this non-school, was bullying.

Because they didn't force him to do anything, he retained his curiosity and motivation.

He lost interest in Latin though! But found other healthy passions like math, physics, computer science...

This is a data point of just one, but I think benign neglect of actual "work" atleast in the early years, can be very healthy.

It can go wrong too, I guess.

We were pretty confident this was better than a traditional system with grades and awards and homework, for these early years.

A pandemic would be a fun opportunity to implement something like this within a social bubble.

Expand full comment

> He refused to work on his handwriting. We did not see a way to fix that. Maybe a traditional school would have forced him to.

Maybe a pen pal? Might be interesting for a kid to learn about another country/culture/etc. but it would only work if they can read his letters!

Expand full comment

Great ideas. Or see his parents write! We never did. I guess he didn't see the point and didn't know to tell us that was why he did not want to work on it.

But he is 19 now. We are past that!

I should have added, since SA brings up testing, he took a standardized test called ISEE after 5th grade and scored in the 99th percentile in all sections including, much to our surprise, the written essay. My husband sat down with him to convince him to use the conventional definitions of +, -, ×, ÷ etc.

He did very well in a competitive prep middle school, always scoring over an average of 95%, from 6th to 8th grade. And then a magnet school, with a 4.44/4.0 GPA despite bring somewhat firgetful and occasionally not turning in a homework. And did very well in 5 standardized tests in high school (ACT, 4 SAT subject tests, 11 AP tests).

So, along these measures, there was no penalty for bypassing a traditional education all through elementary and doing random things. I was always surprised when people understood his handwriting though. To get these grades and scores they must have. Actually I asked a teacher in 8th grade about it. She said these days all kids have terrible handwriting, so they had become experts at reading it.

Expand full comment
author

Handwriting was my bane for most of my school years too. I don't think I "refused to work on it", exactly, but I do think it might have been a fine motor skills issue where my nervous system just wasn't developed enough. I banged my head against it for a few years and then it just seemed to get better on its own, which is great for the three things a year I still need to hand-write.

How is your son doing now?

Expand full comment

That might have been it.

His teacher gave him a triangular pyramidal structure about an inch long with a cylindrical hole to fit a pencil. This helped get a better grip on the pencil. It is a Montessori tool, possibly. It helped improve things a bit. They insisted on cursive too.

His handwriting - we don't get to see it much anymore. It is still pretty bad, maybe worse than ever. He says he types everything in college. Even math homework.

My husband also has terrible handwriting. Maybe this is hereditary.

Expand full comment

>Maybe no screens. (The school seemed pretty zealous about that). With screens available, kids seek those bells and whistles all the time. We were pretty serious about ourselves having little to zero screens.

This caught my attention along with the info that your son is 19. Would you be able to expand further on your no-screen/minimal screen environment in the early 2000s and how that's changed until now? My understanding is that 'screens' didn't really start being super interesting until around the iPhone but didn't really take off until early 2010's. I feel like it was a lot easier to avoid screens in the early 2000-2010, slightly harder from 2010-2015 and in recent years pretty much impossible! Especially non-interactive screens like TV vs phones/iPads now

Expand full comment

I agree it was a lot easier in his younger years to avoid screens than it is now with young kids of comparable age. Nowadays, they've become a normal part of education even in the elementary years.

When my son was 3, I read this book called "The Plug-in drug". It was written 30 years earlier but seemed very relevant to me even then. So we got rid of our TV. Just announced it did not work. It was gone for the next 11 years!

I really liked the argument in this book that keeping away screens to the extent possible, is a gift to your kids.

I think it helped him develop wide interests and be interested in conversations with people.

I felt that many of his peers were so enamored by the bells and whistles in video games and TV shows, that they expected that TYPE of stimulation in all things, to stay engaged.

When he was 6, he went to a Robotics camp. I still didn't buy him a laptop. When he was 7, I bought him the "Python programming for kids" book and let him use my laptop once in a while, to type up programs. I could see it became the most interesting thing very quickly, and so I took it away again completely for a few years. The kind of strange hold it had was a bit scary.

His middle school expected home work that involved googling. I still didn't buy him a phone or laptop. We delayed that until 9th grade. He used my laptop.

Residential summer camps used phones to keep track of kids. In fact, my son went to a residential computer science summer camp in 7th grade where the smartphone was a requirement for students, even though they supplied laptops to them for the actual work there. (We gave him my old phone just for the 3 weeks).

It helped a lot that his middle school confiscated any smartphone and so no one had one.

So he went to an elementary abd middle school that was anti smartphone.

Social life in high school involved a Facebook account. I hated that so much but nothing could be done anymore. By this time he was solid in his work habits, desire to do something important, and maturity. The rule was the smartphone was not allowed outside the kitchen. Every argument in high school, pretty much, involved him violating that!! All parents I know report this to me now, that it is mainly what they argue about.

His magnet high school gave him tons of mostly interesting work, he was involved in hard academic competitions, etc, so he was busy.

His work in high school involved the computer a LOT. Kids were up late discussing group projects. But I convinced him to use the computer not smartphone, for all that. Somehow, that helped him keep that communication efficient.

Since the phone goes everywhere with you. It is a lot worse than even a laptop.

I STILL worry about the smartphone and how strange, addictive and harmful it is. It interferes with healthy family life by being a constant distraction for us all. But even in hindsight, I think I was correct to recognize it as a bad thing!

Screens are hard to avoid but the smartphone is the worst kind of screen. It is getting harder and harder to avoid. I worry about kids growing up now, in this regard. Jonathan Haidt does good research on smartphones and teens, btw. I just "knew" what he researches, intuitively.

Expand full comment

Thank you for sharing this.

Expand full comment

Yes, huge thank you for the write-up! Some great ideas here

Expand full comment

Sure. I left out an important point.

Don't buy a laptop even, if avoidable. But a computer that is large and cannot be easily moved.

And set it up in an area where parents come and go often, unpredictably. Otherwise even the best-intentioned kids waste a ton of time going off on tangents.

The whole screen thing is awful. It creates such bad work habits, among other things.

When they're younger, you might want to set up parent control software and be more tech-savvy than they are, just to protect them.

Expand full comment

We should find out what successful middle age adults remember from school. Anything most such adults don't remember should be considered non-vital to teach kids.

Expand full comment

I believe you had Bryan Caplan on your podcast to discuss this, so you already know but it seems like middle age adults remember very little from school. It would be difficult to determine what information was reinforced along the way. For example, if I had not taken an interest in the structure of DNA recently, I would not have remembered the bases used in DNA. If you tested me, I would remember the bases of DNA but that cannot be attributed to my education. I actually think that a very great deal of the information that is "remembered" would've just been reinforced at a later time.

The usefulness of the information should be evaluated it as well. If 20% of the knowledge remains, then that 20% should be evaluated. They should ask "How often do you use this fact?"

Expand full comment

Here you are evaluating education via facts, when a big part (as mentioned in other comments) is skills, instead. School taught me to sit in front of an empty paper, organize my thoughts and write. It taught me to sit still through boredom, frustration, inner turmoil, to piece together a conversation retrospectively after tuning back in, to phrase questions in a way likely to produce a satisfactory answer, to keep a schedule ... you get the point.

It's not the facts themselves, it's exposure to facts. It's not what is taught, it's submitting to teaching.

I think you'll also find that what information is "vital" depends entirely on the career, and teaching only what is useful for the most successful will probably fail the ones in "lesser" occupations.

Expand full comment

Few professional adults write papers. Working helps students to do those other things and for some, it's less frustrating and has less turmoil. A job teaches you to submit to an employer, which is more relevant than submitting to a teacher which most won't have after 18. Why not just have the kids work?

Expand full comment

Intention often is to provide as many kids as possible a variety of skills so that they have more freedom to choose a career of their liking compared to whatever skills they learn in the one job their parents pick for them.

Like, most people don't know or need Gaussian elimination. Having some intuition about linear system of equations is useful for those who choose to pursue anything mathy.

Expand full comment

I think that's the justification, but I disagree that the current system actually does that. Mostly I see the current system pushing kids through stuff that is college prep work, regardless of aptitude or desire. If the goal was "broad exposure to choose a career they'd like" I think we're doing it wrong.

Expand full comment

"It's not what is taught, it's submitting to teaching."

I have to argue back about that. If you are teaching pupils that France went to the moon in 1896, that does make a huge difference. "Submitting to teaching" in that case is doing nothing except "uncritically swallow what an authority figure tells you, even if it contradicts your common sense or what seems plausible".

This is, after all, half the argument over CRT teaching in schools! Is it merely "teaching students about racism and the history of slavery" or "blaming everything bad on white people and never admitting they ever did anything good"?

Expand full comment

I phrased it so negatively on purpose, but it can also mean things like "listening to someone explaining a thing" or "spend time on a topic of someone else's choosing", and I think those are pretty essential skills.

I have to agree on the France to moon thing, but at the same time, it doesn't matter much whether 19th century France gets covered in history class or not (the general teaching of history, of foreign countries' history, of non-Western history, those matter, but the exact curriculum? I don't think so.) "Teaching true things" and "teaching these specific true things" aren't the same.

Expand full comment

"listening to someone explaining a thing" or "spend time on a topic of someone else's choosing"

Of all the people I've met in the last decade, I'd guess that less than 5% of them have these skills. 100% of them went to school.

Expand full comment

I disagree! What we learned then was necessary to what came next. We are also learning how to learn then, building a work ethic, etc.

It is impossible to start from where we are now and try to guess the unnecessary parts in our education. Things could be pretty interconnected too, in unexpected ways.

Expand full comment

Facts are training data that (hopefully) improve mental models in ways that persist after facts are forgotten.

In teaching, I have mostly stopped caring about facts for their own sake. I share facts to demonstrate ideas and ways of thinking. I share facts to provide context for the analysis I model. Some of those facts will stick, and some of them won't. The real curriculum is me.

A good school, to my thinking, is filled with wise teachers who are empowered to mentor students in how they approach projects and problems instead of "covering" prescribed standards and facts.

Expand full comment

Dr. Daniel Cunningham, a cognitive science professor who studies K-12 education has some great insights on all this. He says we de-emphasize memorization too much. His entire book has brilliant analogies. The book is, "Why do students not like school".

Expand full comment

Do you mean Daniel Willingham, Why don't students like school? (https://www.amazon.com/dp/B092TSZHCC/)

Expand full comment

I agree that mere lists of names and dates are not much good, you need to be able to absorb what you are being taught, think about it, and come up with ways of connecting that knowledge to other areas in a coherent and useful way that will generate insight.

But too much "ways of thinking" on its own, unmoored by "what are the facts here?" is just as bad. Think of all the online discourse that is "how I feel, what I think, this is terrible!" which is not tethered by any facts at all, or at most is the pop culture version of things.

If you know twenty ways of arguing over Christopher Columbus' choice of breakfast cereal, but don't know that he didn't in fact eat breakfast cereal, then you have not learned anything except maybe how to shout the loudest.

Expand full comment

Maybe school just teaches and reinforces important life skills:

1. Schedules, deadlines and organization.

2. Socialization.

3. Discipline.

The actual subjects and knowledge is maybe just window dressing, since the stuff that's relevant you'll learn anyway in due course. The above skills drive the economic engine of the world. The kids who drop out fail and succeed less perhaps fail to appreciate some of these skills, and so don't end up finding a good spot in the economic engine.

Expand full comment

The type of scheduling, organization and deadlines, socialization and discipline that are best fit for working in the economy or the type that is common in an actual job. Why not just let children work?

Expand full comment

We do: Children can work 40-hour weeks during the summer starting at 14 years old. Few of them do -- even the 16-19 year-old group mostly isn't employed over the summer -- so probably starting younger or loosening rules for during the school year would not add a lot of workers. It's part a supply issue and part a demand issue. If we did not have compulsory education for kids, I do not think most of them would work, in part because kids in general are not very productive and in part because the kinds of kids who would drop out quite young are probably particularly unproductive.

Expand full comment

We should let them even if not a lot would do it. The fact that you have to go back to school makes stable employment at 40 hours a week only a temporary thing.

Expand full comment

It's only stable employment because it's temporary. Places take on kids working over the summer or for after-school jobs because they won't have to pay them full wages, they don't need or want them for anything too responsible, and it's cheap grunt work. Make it into a 'proper job' and you won't get employers wanting to hire on 16 year olds, or else be prepared to have everywhere employing 16 year olds, so if your food is cooked wrongly or you can't get your problem sorted out, the 16 year old who is working that job for three weeks won't know what to do, can't help you, and isn't mature or experienced enough to come up with a different solution.

What you will get with young workers who are not working part-time/temporary jobs is a labour force of cheap, inexperienced workers whose selling-point is that they are disposable. For some jobs, yes, putting a young worker alongside an older, experienced worker means they can learn but for those jobs to be any good, we call them apprenticeships. But where you replace your older, experienced workers with the cheap youngsters is not going to provide good service for anyone.

We take kids in on work experience here where I work, and they get the kind of make-work stuff like shredding or photocopying or similar because you can't give them real work to do and this is just about getting a taste of what working life is like, anyway. The kids doing college/post-secondary school vocational courses who come on work experience are more useful but still need (by regulations as well as by simple prudence) to be supervised by a staff member, again because they can't be allowed work on their own. It helps them see what the reality of a job in this area would be like, and if they're suited for it. Then they go back and finish their education and get a qualification for the job.

Modern working practices have changed so that there isn't the route from "messenger boy to vice-president" anymore, you don't start in a job at twelve and work your way up the ranks of the company.

You'll get cheap manual labour and unskilled labour with a workforce of fourteen to eighteen year olds. And yeah, you'll get some training on the job for them - maybe someone starts washing up in a kitchen when they're fourteen and gets on the job training as a cook and moves up to do that as they get older.

But when jobs can be done by sixteen year olds, then wages will be at a sixteen year old level, and that is not a living wage. Any one older than sixteen doing those jobs will be on those wages, because 'why pay more, it's a kid's job!' and you are going to have a permanent underclass of low-wage labour. A bit like the necessity of illegal immigrant labour to do farm work in America as it stands, because paying the wages that Americans would expect is too much for the farmers.

So sure, let fourteen year olds work full-time jobs, but you are now going to have them living at home permanently because they will never be able to move on to jobs with decent wages. Because you are saying "these are the kids too stupid for white-collar work, they're not academically able, they're getting nothing out of school, let them work". And once you establish a labour market where fourteen year olds can work the same jobs as adults, then the wages will sink to "pay a fourteen year old still living at home being supported by their parents" level and stay there. What employer is going to pay full whack to someone in their twenties when they can get a cheap fourteen year old instead?

Expand full comment

I agree that younger workers are frequently not particularly useful workers because of the degree of supervision that they need, because they are not good at making decisions independently. When it comes to white-collar work, even my college student interns were not really on net useful when it came to professional tasks (as opposed to something like running errands.) For older teenagers, the only employers I know who consistently get genuinely skilled work out of people that age are militaries, which invest heavily in assessment, training and supervision. But for them, there's a major advantage to youth for other reasons that aren't the case for most employers, so they have a reason for doing it.

Expand full comment

See, this is the bit that gets me: "the stuff that's relevant, you'll learn anyway in due course".

How will you learn this? Who will teach you? Will you teach yourself? Learn 'on the job'? On the job learning can give you "do this by rote" but no understanding of what it is you are doing, so when the routine breaks down you are stuck.

How do children learn to read and write and do basic maths? There seems to be the assumption here that schools do nothing at all, that parents will teach the kids how to read or the kids will somehow pick it up out of the air (maybe by watching Sesame Street on TV?).

I could read before I started school at the age of four and a half (a bit late, school age was four in my day and kindergartens/pre-school and so on were not common; you started school proper at four - the minimum earliest age - unlike today where you have pre-school etc. and go on to 'big school' at four).

However, going to school gave me so much more opportunity to learn, to access sources of information, to be taught things my parents couldn't teach me or didn't think to teach me. Socialisation was of course part of that, and since I was living out the country with no kids of our age around that was important, (but it didn't take very well with me since I never have learned how to be around people).

But I didn't go to school to be socialised. I went to school to learn, and I did learn, and if I had been taken out of school at age twelve or fifteen I would not have 'picked it up anyway'.

I do think there's a bias here in the commentariat about coming from backgrounds that have access to (1) resources (2) parents who are themselves educated (3) other people, including children of your own age, around (4) living in cities or towns or other urban areas where there are places to go and things to do that will give parents and children opportunities to learn outside of school.

"School is only good for meeting other kids and signalling" is something that may be true for *you* but is not true for everyone.

Expand full comment

I believe that schooling is used primarily for signaling purposes (~80%). The best book on this that I know of is Bryan Caplan's The Case Against Education. He believes education is a signal of conscientiousness, intelligence and conformity.

A major issue with education is the effect of the problem of forgetting or information retention. Students retain very little information. It is not surprising that students can actually catch up quickly because all the other students forget most of the old information anyway. What matters is how you stand in relation to other students for signaling purposes because you will forget a great deal of what you learned in school. Even if you remember it for the rest of your life, very little of that information is of use for the students in their personal or professional lives.

The entire process is wasteful and harmful because it is not quite zero sum but there is not a ton to be gained. We should legalize child labor and allow students and parents to make the decision. Here are the top careers in America according to indeed: Cashier, Food preparation worker, Janitor, Bartender, Server, Retail Sales Associate, Stocking Associate, Laborer, Customer Service representative, Office Clerk. A great deal of students will grow up to be in these jobs and not use history, biology, chemistry or even basic math. If you are going to force children to do something for their own best interest, you need to demonstrate that it's in their best interest beyond a reasonable doubt. Using coercion against someone in their so-called best interest when it is dubious or clearly not in their best interest is very bad. Furthermore, the cost of keeping children out of the labor force is ~$10,000. So it costs a great deal to do something that is not in many students best interest.

There is a straw man that child labor means children would be in coal mines but in reality children could probably work simple tasks that are not cognitively demanding. The really smart kids can stay in school because they will eventually have jobs that require technical knowledge. Those children could be compensated the cost of educating them and have that money invested for them. After years of experience, you could have 18 year olds who have developed a great deal of specialized skills and have demonstrated competence in an alternative way.

Expand full comment

I appreciate your argument and I will look for Caplan's book. Some of those jobs, though, are the ones I think most likely to be done by machines - not even necessarily AI - just automation. Stocking, most clerk roles, cashiering - so much can be done with a combination of online ordering and automation. The path of birth-school-cashier could be changed to birth-work-cashier but I think more likely it will be birth-school-unemployment or birth-school-UBI (if it's adopted). Birth-apprenticeship-job that still exists might be a better path. There are massive reserves of untapped artistic, mechanical and social talent which school obscures.

Expand full comment

People will go into the jobs available. After AI takes over those tasks, other low skill labor tasks will become more common. I think that schools should train for apprenticeship but my position on schools now is based on what they currently do.

Expand full comment

To clarify: I meant to say if a student leave school before 18, they would receive the monetary value of that schooling (~$10,000 per year) either immediately or as an investment.

Expand full comment

If you really think a cashier or bar tender doesn't use basic maths, I think you have no idea what you are talking about. And those are not careers; they're jobs. A very different thing.

If you're working with food, for example, even if you are a high school drop-out you will be expected to comply with basic hygiene requirements so as to avoid giving the customers food poisoning. No, you don't have to be highly educated or very smart, but you do need certain traits such as conscientiousness and you do need to be able to understand about infection, cleanliness and the like.

And it seems it's not a job that people are very happy in:

https://www.careerexplorer.com/careers/food-preparation-worker/satisfaction/

You are just reinforcing what parents and society in general think: if you want any kind of good job, much less a career, stay in school and get an education. Otherwise you are going to work low-wage jobs in poor conditions for the rest of your life.

Expand full comment

The reason why those jobs are the most common is because there are bottom rung people who need jobs. Indeed, most of those jobs are things that are either presently being automated away or are things we can do without.

In a society without bottom rung people, we would probably automate away most of those jobs, or they'd become much more scarce (having other people cook food for you would be expensive if the dumbest person in society had an IQ of 120 and expected a salary to match).

The reason why these jobs exist is because there are bottom rung people who need jobs so we can pay them very little to do them.

And as for your list: Chemistry, biology, physics, mathematics, statistics, economics? They're necessary for being a useful citizen in society. You basically cannot make meaningful choices about political stuff without it, and if you have to make choices about stuff like healthcare, having a grounding in this stuff is vital.

You literally cannot understand most things of importance in greater society without this stuff. People who are antivax and hold other foolish beliefs are ignorant of these things at a frequency far above chance.

Expand full comment

If society changes massively and the most common jobs disappear and everyone requires skills, I might modify my position but I am talking about the current situation as is. I anticipate other low skilled jobs becoming popular if all that stuff is automated anyway.

The reason the jobs exist is because they need to be done. The existence of the people doesn't entail the existence of the jobs.

"You basically cannot make meaningful choices about political stuff without it..."

Would you advocate for people to demonstrate competency in these areas before having the right to vote?

Expand full comment

The reason why we barred literacy tests and similar things in the US is because they were used for the purpose of excluding black people specifically from voting while having exceptions for white people; the history of abuses associated with these tests is what resulted in their ban.

I take no issue with the notion of requiring people to pass a competency test to vote. This would also solve the issue of "How old do you need to be to vote?" quite neatly; people who can pass the test are capable of voting, those who aren't yet ready cannot.

I don't think that it is plausible to get such a requirement passed in the United States for obvious political reasons.

"The reason the jobs exist is because they need to be done."

We have mostly replaced cashiers at a number of local grocery stores; the overwhelming majority of people at this point go through self checkout. I have been to restaurants where they replaced most of the wait staff with apps; you ordered electronically rather than through people because this reduced the staffing load, with staff being there only to bring food out to people. And in the end, thanks to things like pre-prepared frozen food and gas station food, restaurants really don't *have* to exist or be all that common. Costco proves you can have far fewer people engaged in stocking activities, and of course, Amazon does away with traditional shopping for many things entirely.

There's lots of jobs that could be done away with or made significantly more efficient and thus require fewer people if it was too expensive to employ people to do them. Indeed, rising wages, mechanization, and automation have historically eliminated many factory and agricultural jobs. When it becomes cheaper to automate than employ people, we tend to do so.

Expand full comment

Here is a viewpoint.

It is possible that some (maybe majority of) people who need to pick up something they didn't previously learn can pick it up when it becomes important to them. It can also make a mess of any planned curricula that builds on previously learned material, which results in repeating things some of the students know it for the benefit of those who don't know, for one reason or another. Naturally if most of everything gets repeated at some point, altogether missing a year or two does not hurt much. But wouldn't it be nice and efficient if everyone who comes to class would know the prerequisites, and on each class one would keep moving to new, exciting, more advanced things?

Expand full comment

Could anyone here recommend a good source on how much young children benefit from time spent by parents developing early academic/school readiness skills, skills such as letters and the names of letters and numbers, counting, reading out loud, holding a pencil, etc.? Everything I've seen confounds kid interest/IQ/socio-economic status with parental effort, but parent effort is the variable that I'm interested in because it is the variable that I control. I was hoping that Emily Oster's new book would cover this, but she pretty much entirely skips the preschool years.

Expand full comment

Short term benefit or long term? Are you looking for if they'll be ahead of their classmates briefly or if they will earn higher income later in life or do better in college?

Expand full comment

So schools are really bad at teaching.

What would an effective schooling look like for kids who actually really want to learn?

Expand full comment

There would not be 1 year of biology/chemistry/etc. but 12 years with each semester building on the knowledge of earlier semesters. Exam topics would retest over all past coursework, not just recently. You can graduate when you demonstrate competency in all the desired areas but you have to complete a final exam with all the information you ever learned on it.

Expand full comment

A lot of comments are suggesting that schooling is important for socialization. For those who feel this way, what do you think will happen without this type of socialization?

Homeschooled kids socialize with friends or when they go to church or play little league sports. If children worked, they would socialize with their coworkers. Is this not enough or the wrong type of socialization?

Many people who read this blog are introverted or a little odd socially. Would it be beneficial to use taxpayer money to help them socialize better? I think people would probably say no to this. But it seems reasonable that if it is a good thing to do at age 10, 12, 14 and 18, it should be a good thing at 20, 25, 30, etc.

Expand full comment

School is important for socialization! Boy howdy I am just *so* grateful for the bullying and abuse I received from not only students but also teachers and administrators. How else would I be prepared for modern adult life?

Honestly of everything I experienced in my childhood, the social environment of school was the one that was the *least* like adult life in every possible way.

Expand full comment

I also think there are a lot of negative or unnatural forms of socialization. This experience is probably very different for people according to their personality type. Clustering people by age cohort so that everyone is the same age and is instructed by a teacher that is a great deal older is a very unusual environment. To the extent that it adapts you socially, it would seem to be a maladaptation. Where else follows this structure? (real question)

Expand full comment

In school you are socializing with people all of whom are about your age and, in some sense, your direct competitors — for grades, but also social status. That is a very unnatural social environment.

Expand full comment

I think having children do things together with other children is what makes them get friends in the first place, usually (didn't work for me, but I am certified weird). More than that, I think school serves as a place where you meet a broad array of kids. It gives you an idea what is normal, and what sort of kids exist in the world. Self-selection will distort that (private schools, homeschooling etc do too).

"Am I normal?" is a stigmatized question in a society that values individuality above all, but it is an important one. It helps you find our where you are relative to others: what are you good at? What are you bad at? What can you expect from others, how likely is it that a random kid/person has this or that trait? Identity can only develop in company, and hoo boy does an individualist society care about identities. Whatever your identity is within your homeschooling group, it'll probably be "the homeschooled kid" when you leave that bubble. There's also the comparison of teachers and other kids' parents to one's own: it's the place you find out not every parent hits their kids, or has a big house and garden, or lets them watch TV all day, or goes to church. You're confronted with all sorts of ways of living through others at school (less so in the US with private schools, homeschooling being legal, neighbourhoods being extremely divided by class).

I don't think you need to spend every day sitting at a desk being told you can't go to the toilet to have that, and a lot of it can be done in smaller and selective environments. But there's another meaning of "socialization": "the process of learning to behave in a way that is acceptable to society" (Oxford). Society. Not your church, your football club or the friends your parents approve of. And the norms and opinions and customs of the wider society help you assimilate into it on your own, free you from the control of your parents. I think this the main argument for schools, actually.

We can talk about the smart and privileged and nice kids of smart and privileged and nice people all day, but those are not the one that need school as a gateway to self-determination.

I say all this as someone who was definitely harmed by school and would probably be a better, happier, healthier, more productive and better adjusted person without it.

Expand full comment

This is a good argument, but it only works to the extent schools fulfill this function. I think many of them don't. As you admit up top, private schools definitely don't do this.

Expand full comment

And what is the "wider society"? To what extent should parents who may not like the messages prevalent in that wider society be allowed to make decisions about what their children should be exposed to, whether through homeschooling or private schools or even just deciding what sort of media/internet access/etc. their children should have access to?

Expand full comment

That is a matter of balance, but I am coming down hard in favor of parents not being able to opt out of outside influences for their children to the degree that unmonitored homeschooling allows. "Wider society" is vague on purpose, but usually it's the country/state that is in charge of curricula and schools.

Two thought experiments, let's assume for a second that our only options are complete homeschooling with no oversight and compulsory attendance of public schools: 1. a dissident family in Nazi Germany. 2. a fundamentalist anti-gay family with a gay child in a modern European city. Both disagree with the things their child would learn at school, the kids it would meet there, and both have an interest in isolating it from those influences. If we don't want to descend into "school is good when it teaches the right values and bad when it teaches the wrong values" then our answer must be the same for both.

I think parents should have the right to teach their children what they please, as long as the children also go to school and learn what "the outside world" believes, outside of the control of the parents. Marketplace of ideas, if you will. Take school away, and the kid is basically a hostage to their parents and their parents' community, unprepared for going their own way.

Real life is less black and white and there's middle ground, obviously. In practice, I think schooling beyond grade school should be optional, but some form of getting away from your parents, being around random kids, regularly - I think this is necessary.

Expand full comment

I do think it's a tricky question, and your hypotheticals get to the heart of the matter. I think even someone who supports homeschooling or unschooling would agree that there's a point at which homeschooling could be so bad that it's essentially child neglect. I brought up the "wider society" point because for some families the "wider society" would be an Amish community or a Hasidic village. Those societies clearly have different values from the US as a whole. To take a broader example (and, admittedly, to veer into culture war territory), many people (or at least politicians) in Hungary clearly have different views on how children should be raised (pertaining to the recent hubbub about the Hungarian law about exposing children to LGBT material) from politicians in the Netherlands and much of the rest of the EU. If you have public schools, the decisions about what is taught in them may be made at various levels (local school boards, states/other equivalents in federal systems, national governments, even EU decrees or public pressure). I would much rather live in a world where parents have a relatively free hand to raise their children than one in which politicians acting on behalf of the "wider society" make the decisions.

Expand full comment

I think the Amish community or Hasidic village would be too narrow to consider "wider society" here - but I have to admit I'm largely ignorant of these matters as a German, where curricula are set at the state level and even private schools with a religious focus or philosophical slant have to adhere to them.

I'd argue both the dissident in Nazi Germany and the LGBT Hungarian profit from exposure to their surroundings, though. To learn how to speak and act in a way that is acceptable to the majority, to learn what they believe and what will cause backlash is important to fight against it, too. If most people are against you in some way, knowing how to stay in or move back into the closet is a useful skill. I don't think full-time assimilation is required for this (and indeed, it's not hard to see how it is damaging, too). I've seen enough examples of people remaining hostile towards the values taught in school, parroting their parents' objections to it, to be confident that parental influence is still strong in the face of compulsory school, though.

If I can spitball for a bit, imagine a fully modular education, where kids can take some subjects, all subjects, accelerated or decelerated classes according to their needs (with minimal requirements as to what order to take them in, or to their number). Imagine homeschooling for most subjects but sending your kids to school for math, imagine taking fewer classes for a stressed or ill or slow or just not very motivated kid, more or faster ones for an ambitious one, and allowing alternatives to regular school. In such a scenario the only things I'd mandate would probably be one course per year/semester in a public school up to age 14 - it might even be an extracurricular, as long as some really basic stuff is taught somehow, somewhere, by someone qualified (civics and health come to mind). As to who decides what a kid takes and where, two out of three - child, parents, teachers - should work?

Expand full comment

"If we don't want to descend into 'school is good when it teaches the right values and bad when it teaches the wrong values' then our answer must be the same for both."

Why is this a descent? Can't we say that Nazi schools are bad, modern European schools are good, and our ethical principles ought to acknowledge the difference? I'm not sure why there should be a universal principle governing whether parents should be allowed to opt children out of school. Or why that principle can't be "parents should be morally obliged to allow outside influences on their children, provided those influences themselves meet a basic minimum standard of morality."

I don't understand the tendency towards trying to design principles or institutions or laws that are completely agnostic towards object questions.

Expand full comment

Homeschool socialization varies enormously; there are the ex IBLP/Bill Gothard/Pearl/cult homeschoolers who were socialized extremely oddly and have trouble adapting, and plenty of other homeschoolers who were socialized decently. I mainly participated in church youth group and 4-H clubs several times a week for most of a decade, and that worked decently well.

The current cultural climate is not very tolerant of people saying their weird counter-zeitgeist opinions in public, and parents probably have a duty to explain why reticence is often the better strategy. Schools teach this more through reinforcement learning, but it might work just to tell the young adult that when they go out into the world. There can be a steeper learning curve for homeschoolers trying to work in somewhat oppressive environments.

As to whether it makes sense to spend public money on adult socialization, it might depend on the specifics of what that looks like, and the extent to which it works. My main objection is that there's a pretty good chance the Department of Friendships would organize lame activities where nobody actually made any friends, but the administrators made a nice living for themselves and everyone played along and pretended to be interested. As has been known to happen in schooling.

Expand full comment

Something like this actually existed in East Germany, in the form of the Jungpioniere and FDJ (Freie Deutsche Jugend) (nominally voluntary but in practice membership was necessary for getting into university or your preferred line of work), and for adults the Arbeiterbrigaden. In part that went like you said - indoctrination mostly (but not exclusively) being received as something to pay lip service to and roll your eyes at - but the activities weren't actually lame (clubs, vacations, concerts) and they also did a lot of things people in this thread lament schools not doing (practical skills, useful work, interest-directed learning). I don't have the spoons to write the essay the topic demands though.

I should mention that one of the things people lament most after the German reunion (apart from, you know, the unemployment and poverty and mishandling of Eastern assets) is the loss of social cohesion. This stuff worked.

Expand full comment

I mean, the activities college organizes for young adults are... lectures and homework? They seem to work. (Obviously I exaggerate, but I do think that if that were all that was provided, it would work.)

Expand full comment

"Would it be beneficial to use taxpayer money to help them socialize better? I think people would probably say no to this. But it seems reasonable that if it is a good thing to do at age 10, 12, 14 and 18, it should be a good thing at 20, 25, 30, etc."

You may or may not be glad to learn that Social Prescribing/Community Referral is now a thing in Ireland:

https://www.hse.ie/eng/about/who/healthwellbeing/our-priority-programmes/mental-health-and-wellbeing/social-prescribing/

"Social prescribing recognises that health is heavily determined by social factors such as poverty, isolation and loneliness. Social prescribing offers GPs and other health professionals a means of referring people to a range of non-clinical community supports which can have significant benefits for their overall health and wellbeing.

Social prescribing generally involves three key components

- A referral from a healthcare professional,

- Consultation with a link worker

- An agreed referral to a local community activity. Examples include; art, cookery, meditation, GAA, men’s sheds, music, drama, walking groups and many more.

Social prescribing can also enable and support people to access health services such as Smoking Cessation, Self-management support programmes and others."

Were I the kind of person who wanted social support, I could go to my doctor and talk about this. Because I'm not, wild horses would not drag me to a weekly group-therapy/hobbies and meetings style event where I'd have to (pretend to) talk to people.

Expand full comment

This is related to my interests, as a BSA Scout, Girl Scout, Young Mudder, Library, and Sunday School volunteer, and parent of two kids with a fair number of home-school friends. Yes, it is vital (though many schools fail terribly at this job), and yes, we should continue to use taxpayer money on it. Socializing doesn't mean "get along wit your peers". It means "learn how to function in all sorts of environments." School presents a number of novel environments that parents can't provide because learning to function without your parents can't be provided with your parents. Learning to deal with beurocratic, stratified organizations that say they care about you but don't, staffed with people who probaby do care about you (but maybe not) but are beholden to the org - well, that's a thing basically everyone is going to have to deal with for the rest of their lives.

This all comes from schooling. If you think public funds shouldn't go to schooling (I do, in theory but very much not in practice in the US in 2021) then yeah I think public funds should go to it.

Expand full comment

School isn't a normal environment. After school, people won't be in it for the rest of their lives and likely not anything too similar. Why not get children socialized/accustomed to the workplace which is the environment they will be in for the rest of their life?

Expand full comment

I missed most of school, for various reasons, and never caught up on any of it. Now I have a son who has different problems, having developed speech very late, causing him to be held back a couple of years, and ultimately attending a private school where he is given much more attention by the teachers. My experiences with school systems in different countries agree with the article, but I would add that they are also intrinsically unfair. Where you live determines the school you go to, the college you go to, the job you wind up doing - or where you live means you don't get to go to college at all. Schools are a system of networks which unfairly sort you based upon arbitrary factors. The ones at the top of the pile will make sure that you get the grades you need to continue, barring an utter lack of participation in your studies.

Expand full comment

Speaking of which, I would like to use this article as a reading comprehension exercise for my 13 year old kid. Do you have a reading comprehension question appendix?

Expand full comment
author

Just go on Twitter, and if your kid agrees with any of the takes there, fail him.

Expand full comment

Math & language instruction differ from each other, and over time, in systematic ways that I think are relevant here.

From a US perspective, grade-level math (e.g., 4th grade math) is much more repetitive than subject-level math (e.g., trigonometry). In my personal case, I was moving 2-3 "grades" of math per year of elementary, but once I got into the subject sequence (algebra 1 -> geometry -> algebra 2 -> precalc -> calc) I moved at the same pace as everybody else, just years younger. Missing a substantial fraction (no pun intended) of grade-level math is more readily recoverable because what follows doesn't depend on what came before to anything like the same degree. I missed one day of calc, when Reimann sums were covered, and struggled for weeks after (I've still never really grokked them, despite advancing much further in math).

For language arts, there's no apparent dependency at all, just repeated practice. At no level does it matter whether or not you read a particular work in a previous class, and teachers seemed to start from scratch on parts of speech &c every year regardless. I would expect missing this instruction at any point to be as recoverable as missing grade-level math.

Expand full comment

If you are interested in an anecdote:

I did not go to high school (well, attended for two or three months) and now I have a PhD from a very good university. Not receiving any formal education between the ages of 16 and 23 does not seem to have affected my ability to do college (and later grad school) level work.

Expand full comment

I wonder what effect school has on the ability to write Chinese characters. It seems to have been large in the Sinosphere. The papers on the matter are all probably in Chinese, though, and I don't read Chinese.

Expand full comment

https://www.harvardmagazine.com/2010/11/kindergarten-matters

This study seems to suggest that even though the effects in terms of school performance may not last beyond a few years, there is an effect that returns in adulthood, translating directly to increased income.

There might be something special about kindergarten age, or class size, and this effect may not apply to older students, but it does suggest that the quality of schooling may affect more than just school performance, and thus looking at school performance is insufficient.

Expand full comment

Many interviews with homeschoolers/unschoolers/altschoolers and their parents here for those who want to hear from people who skipped most or all of conventional schooling and have had great lives (including an interview with John Deming, father of Laura Deming, who never attended school but was admitted to MIT at 14, dropped out for a Thiel Fellowship at 16, and is now an anti-aging VC),

https://whenschoolsnotworking.com/episodes/

Expand full comment
founding

I fear that this may be a case of looking for one's lost keys under the streetlight. The aspects of education and human development that are easiest to measure and do statistics on, like reading and math and ability to name presidents in order, may be the part where formal education has the least marginal relevance.

You mention the possibility, and one study in support of same, that gaps in formal education might result in increased dropout rate. I'd generalize that to executive function, conscientiousness, and diligence generally. Probably someone like John Stuart Mill is going to grow up to be a quite diligent chap, but not everybody gets to be the son of James Mill. Averaged across the range of human possibility, there may be a substantial impact from formal education here - particularly when we're dealing with the offspring of lliM semaJ. But we're not talking about that here, because we don't have the data readily available.

And then there's the bit about how it is So Very Important that children go to a Proper School so that they can learn social skills; the homeschooled kids will all be doomed to a life of nerdish dorkitude. Which I've always been skeptical of, because of the obvious counterexamples and because every school I've ever been to "taught" social skills the same way being thrown in a lake teaches swimming.

Except, people who are thrown in a lake are *somewhat* more likely to learn to swim, than people who live in a desert. And we're (hopefully!) coming out of a year and a half when not just the schools were closed, but most of the other venues for childhood social interaction. Also, unlike e.g. math or reading, in childhood social interaction, being left behind in the development of social skills results in being excluded from opportunities to develop those skills, so less likely to be a rapid catch-up period at the end.

But, again, we're not talking about that because where's the data? Look, here's some shiny standardized test scores in math and reading.

Expand full comment

Thesis: "adolescence", the cultural phenomenon, is a pure artifact of the school system, in which young people with no responsibilities are forcibly confined with each other, prevented from interacting with the broader society or their own families, and given nothing to do. This alienates them from their families and causes them to develop "strange" (accurate, but only for them) ideas about how society functions.

My second-hand read of other people's impression of homeschooling is that the academic effects are nothing to remark on one way or the other (as noted in this essay), but homeschooled children display a striking tendency to fit into their parents' world without problems that ordinarily-schooled children do not.

Expand full comment

Yeah, I think a lot of times when you leave kids alone in peer groups for too long they end up creating miniature Lord of the Flies style societies whereas people who have a lot of socialization across age groups end up better-socialized. Schools should try to figure out more ways to account for that.

Expand full comment

Adolescence is a recent cultural environment. We continue to delay giving young people the challenges of everyday life and the consequence is that they are unprepared. This unpreparedness is then used as a justification for continuing the infantilization.

Expand full comment

An alarming result of this is the large fraction of teenagers today who hate adults. Google "okay boomer"; everything you find will be hate speech against adults. This has been going on since at least the 1960s, when Jack Weinberg said, "We don't trust anybody over 30."

Expand full comment

I'm homeschooling 6 kids, with the oldest two in college now, having started when they were13-14. So far they're doing well, better GPA than school educated me at an equally good college. This is in spite of a total of 6-10 hours per week of instruction before college. They all proceed at their own pace, but I predict the rest of them will follow their elder siblings in starting college early and finishing in 4-5 years.

This tells me that smart kids don't get much at all from school. I remember it mostly as busywork and a waste of time, and I'm happy that I get a chance to spare my children that misery.

Expand full comment

The year I started high school, the school I went to (GWHS in Denver) instituted a policy that any student with 11+ unexcused absences in a class would automatically receive an F in that class. I'm sure they had a mandate to fight truancy or whatever but... the sheer self-fulfilling prophecy of it all was obvious to us even as 14-year-olds.

Have I mentioned lately that school is a prison for children, and that education is not its purpose nor has it ever been?

Expand full comment

I really need to precommit to arguing against several views expressed here. My main points will be that dropping 5% percentile from missing a year of school is actually a big deal, that lower grade levels are more important for attendance than upper grade levels, and that measuring the benefits of school for different classes of people reveals how much of a cost absence is, and that metrics by category are the ones one should use for determining whether to go to school or not, as opposed to this extremely broad brush that Scott is using.

Expand full comment

"They should do whatever makes them, their child, and their family happiest and safest, without losing too much sleep over the educational consequences."

I think it would be great to see your take on the risk of COVID for young kids and their parents. If you ask your average Democrat voter, they'd probably tell you COVID is going to kill your child and getting infected is the worst thing that could happen to them. If you ask a Republican voter, they'd say they don't give a shit about COVID. Who's right and who's wrong?

Expand full comment

I question whether learning is relevant to your original question.

Going to an ivy league school isn't important because they teach better--they don't. I've taken classes at possibly a dozen different colleges, from a local community college to Johns Hopkins (and an online course at Stanford); and there is a strong negative correlation between university reputation and the quality of teaching. (For instance, the very best teacher was at the local community college, and two of the worst were at Stanford and Johns Hopkins). The higher reputation a school has, the less importance the professors attach to their classes (and the less importance their teaching proficiency was in their being hired as professors). (It's something like the phenomenon by which, if you go to see the most-famous surgeon in a specialty, you may get the worst care, because you'll sit in the waiting room for 3 hours before the famous surgeon's physician's assistant interviews you, and then the famous surgeon will spend 1 to 2 minutes reading the PA's notes before deciding what to do. Again, personal experience.)

Going to an ivy league school is important because every job search committee at every academic and research institution takes the stack of resumes and pulls out all the Harvards, Yales, Stanfords, and MITs, and only looks at the rest of the stack if none of those people accept an offer. Judging by the results, every grant committee, every venture capitalist, and every Nobel prize committee considers the college someone attended as literally the most-important thing about them.

Similarly, the purpose of going to a private school is probably not to learn, but to put that school's name onto an application to a "good" university. Not one of the "best" universities considers objective measures of learning such as SAT scores, so how could learning help in any case? The only question to ask is whether having attended that good private school /in and of itself/ increases their chances of getting into a "good" university.

Expand full comment

"Going to an ivy league school is important because every job search committee at every academic and research institution takes the stack of resumes and pulls out all the Harvards, Yales, Stanfords, and MITs, and only looks at the rest of the stack if none of those people accept an offer."

Why would they do this, if it isn't a good way to select candidates? Surely hiring is so essential to a company's success that if this wasn't a good way to filter candidates, the companies that don't hire this way will outcompete the ones that do and drive them to bankruptcy.

Expand full comment

I work for a large corporation, and I am afraid to say actual corporate practice is incredibly inefficient, almost no one knows how to hire for performance with any kind of consistency.

What actually happens is ass-covering: in the same vein as "no one gets fired for hiring IBM", and no money manager gets fired for buying investment grade bonds, no HR committee gets fired for putting ivy League / Oxbridge resumes at the top of the pile. At least at my company, the goal of the hiring system is legibility (being able to explain why you hired someone) rather than performance.

Expand full comment

At least a few things prevent this sort of idealized notion of perfect market competition from happening.

First, established companies may have plenty of moats other than good hiring practices. This can be anything from regulatory capture to patents to institutional knowledge.

Second, some companies might even have pretty decent internal training, and if admissions committees for these elite universities are doing any sort of halfway good filtering by selecting students good at learning, then hiring from there can make sense even if the signal is weak, especially at scale where it is difficult and time-consuming to evaluate candidates on a deep, individual basis.

Third, it's not clear the incentives are there for people founding companies or hiring for existing companies to worry about this kind of thing. If you're a hiring manager and your company goes out of business, what do you care? Your chances of getting hired at the same or better salary by whatever company replaces them is nearly 100%, even if your bad hiring did in the last company. From the founder's perspective, I think it would be very difficult to approach investors with a pitch like "I'm going to offer exactly the services and products offered by this other giant company that already does it, but use a more sane hiring practice." Investors expect you have some sort of better product idea, not you're promising to hire better people who will figure out the product for you later.

Expand full comment

I don't know. Every time I've been involved in hiring or training an ivy-league candidate, it went badly. One accepted our offer, but retracted it before starting work because someone gave him a better offer (I hear this is very common with ivy grads). One worked with me for 3 weeks and /then/ quit for a better offer, so that not only did I waste 3 weeks training her and helping her understand the program code, but we had to start the candidate search all over. Another came and stayed, and I worked with him for 2 years, but he just never got very good at any part of the job. On top of all this, they want more money than other candidates. If I ran a company, I'd put the ivy-leaguers at the bottom of the stack.

Expand full comment

Doing a quick Google search, all of the top universities normally require SATs. A few seem to have temporarily suspended that policy due to the pandemic, but it doesn't seem to be permanent.

The SAT is basically a disguised IQ test with some academic requirements, though. It's not actually primarily about learning. They probably could just give people IQ tests and they'd get mostly the same results.

Expand full comment

TD, it appears you're correct. Yale, Harvard, and Columbia normally require SAT, though MIT has decided to permanently eliminate standardized tests from consideration. Thanks for correcting me; I was so confident about that misinformation that I didn't bother to check.

Expand full comment

I learned English as a second language. I don't remember much about tenses or prepositional clauses, and I definitely don't remember all of the millions of words I must have heard or read while practicing. Nevertheless, I can now read, write, speak, and understand English mostly correctly most of the time. I only vaguely remember the plot of Lord of the Flies, but if someone alludes to it in an article, I'd know what they were referring to and understand the argument they were trying to make. As a teenager, I learned how to ride a bike. I don't remember any of the hundreds of motions I did while learning to ride a bike, but I can now ride bikes without thinking about it.

It takes practice to be good at anything. Just because you don't remember every detail about the practice, doesn't mean the practice was useless, or that there's a way to get the same result without practice. School drills into you a framework for understanding the world by forcing you to practice, in the form of understanding the details. Over time, the details fade from memory, but if the schooling was any good, the framework remains.

Expand full comment

I've commented below (in reply to an earlier comment) but that was specifically to do with the schooling.

Here, I want to take just a second to comment on the very last sentence of Scott's essay on earnings being affected by COVID.

I obviously don't know how 2019 graduates will fare against 2022 graduates 10 years from now. However I do believe that this data is available for the GFC and other previous economic crises and the evidence is fairly compelling : graduating during a recession really screw your earnings up for a long while (this is why, iirc, your birth year has a huge impact on your earnings... if 21-23 years after you're born, you're in the midst of an economic downturn, you're going to feel it).

https://siepr.stanford.edu/research/publications/recession-graduates-effects-unlucky

https://www.nber.org/digest/nov06/career-effects-graduating-recession

Expand full comment

But at present, due to federal and state subsidies of excess unemployment compensation, companies are having a very hard time hiring enough employees to operate, and I've seen quite a few increase their hourly wages as a result. So I'd have thought it would increase your earnings at present. I don't know if this applies to salaried rather than hourly wage earners.

An important question is why your salary is so "sticky"--why it follows you from place to place for the rest of your life. I think it's very important for employees to collectively refuse to tell prospective employers what they earned at their previous position. They have no legitimate use for that knowledge; its only possible use is to try to squeeze you harder. I've found that about half of potential employers will auto-reject you if you refuse to tell them this information. This fraction is increasing now with online application forms, which often won't submit the form unless that is filled in; but on the bright side, it prevents me from wasting time interviewing with employers who will demand that information.

Expand full comment

I don't think the subsidies to unemployment compensation are the only factor behind present day labor shortages. But, that point aside, I really like your comment about sticky wages and the reason past salary follows you around. I don't think it's just a matter of anchoring (though that's clearly a big part) but it is also likely that recession damages career trajectories i.e. you wanted to work in finance but you end up in middle office/admin instead of investment banking/sales...

Expand full comment

Stuart Ritchie has a paper from 2018 that has this to say:

We meta-analyzed three categories of quasiexperimental studies of educational effects on intelligence: those estimating education-intelligence associations after controlling for earlier intelligence, those using compulsory schooling policy changes as instrumental variables, and those using regression-discontinuity designs on school-entry age cutoffs. Across 142 effect sizes from 42 data sets involving over 600,000 participants, we found consistent evidence for beneficial effects of education on cognitive abilities of approximately 1 to 5 IQ points for an additional year of education.

Sounds similar to the quasi-experimental results you cited with immigrants and Germany’s new schedule. Not sure how consistent 1-5 IQ points is with what you’ve been saying. It certainly conflicts with what we know about people like unschoolers who’ve missed a whole decade of school.

Expand full comment

How did they define "education"? I'd have thought that a study like that would have to compare several alternative classes or strata of "education", rather than "education" vs "uneducation"--because there is no such thing as "uneducation". People are always learning something. A child who skips a year of K-12 to pursue his or her own interests might easily learn more or less than one who stayed in school, depending on his or her interests.

Expand full comment

They mean formal education/schooling

Expand full comment

An interesting question that you seem to have ignored is the effect of a student's missing out on many day at a *good* school. Or on education from a good teacher in a crap school. Most schools don't teach much so the results you cite are predictable and entirely in line with what we know about the ineffectiveness of the average school and teacher, but they don't really say anything about this other question.

There is some evidence that most of the actual teaching is done by a rather small proportion of teachers. Do keep that in mind.

Expand full comment

There will be somebody who says better, but I think you should split 5. in your list of predictions. There a two assumptions in there.

1) Low performing kids profit from learning "expected values and behaviors".

2) High performing kids suffer from the schol environment.

Expand full comment

One thing I’d consider regarding the primary school versus secondary school aspect is whether the subject matter difference matters. Like, if you miss a year of primary school where they teach multiplication, you can compensate easily. Multiplication comes up a lot in normal life, your parents are probably proficient multipliers, etc. Whereas if you miss a year of secondary school where they teach trigonometry, the rest of your life isn’t going to give you nearly as many opportunities to learn it anyway. That could contribute to primary school absences causing less of a long term effect than secondary school absences.

Expand full comment

There's a recent paper using data from the Netherlands that seems to be of pretty high quality compared to some of the others mentioned above:

https://www.ncbi.nlm.nih.gov/pmc/articles/PMC8092566/

I'm still trying to decide what to make of their results. The authors appear to favor the "learning loss is a big problem" explanation, but a "teaching to the test works well" explanation also seems to fit, at least at first glance.

Expand full comment

Hello, English policy expert specialising in education here. Fascinating blog as always. Some comments!

In England we have a National Reference Test - a fixed set of questions sat by a representative sample of 15/16 year olds each year. This will enable us to measure lost learning and test your hypotheses, and is probably as close as we'll get to the idea in your penultimate paragraph.

When the body politic discussed the response to "lost learning", our education experts wanted to lengthen the school day by half an hour but our Treasury wasn't happy with the evidence from it. Our designated "what works centre" for education has an evidence review (including links to various analyses in the full study). Their view is that a longer school day can see pupils make up to an additional two months progress, but very dependent on how the time is used. This is a finding in a lot of education research that will probably not surprise you - how you use time/spend money/implementing proven programmes with fidelity turns out to be very very important.

https://educationendowmentfoundation.org.uk/evidence-summaries/teaching-learning-toolkit/extending-school-time/

Expand full comment

https://astralcodexten.substack.com/p/kids-can-recover-from-missing-even

I have many things to say, about standardized testing and scientificity, about what school is about, and about privilege.

While I was reading, the tidbits about Gaussian elimination and dates of Civil War events stuck me: “when was this? the 1960s? how old are you?!?” Learning math techniques without the context of problem-solving and, even worse, lists of dates and events instead of sociological trends, was already long deprecated when I was a student myself in the late 1980s and the 1990s.

Then I realized it is probably a US thing, and I strongly think it must be related to standardized testing.

The idea, I think, is that standardized tests are best because they are more scientific. But are they, really?

There is more to science than using jargon to disguise everything as math. We always say that science is about making predictions that can be tested by experiment. This is true, but it is not the whole picture. Testable predictions are necessary for good science, it will never be good science if it is contradicted by experiment or untestable in the first place, but it is not sufficient.

For good science, we need impressive predictions framed in a convincing explanation.

This is where I make the case for a somewhat controversial position: science is not just a compendium of techniques and methods, it is also a form of art: being impressed by predictions, being convinced by an explanation, they are emotions, and triggering emotions is the realm of art.

And this is something that is missing in many recent scientific disciplines. Take economy, one of my pet peeves. Economists will talk about utility functions, saying they are complex objects meant to encode all the richness of people's feeling about the resources available to them. But when it comes down to doing the math, it is too complicated, so they just take utility = price and run the models. The models are not wrong, but they fail at convincing that they match reality, that they are relevant.

A similar phenomenon exists in computer science when it comes to proving that programs are valid. They can do very strict semi-automated proofs that the navigation system of a plane, for example, correctly implements a specification. But it only moves the problem: does the specification really mean I am safe in that plane? In principle, the specification should be simpler than the implementation, and therefore more understandable, more convincing. But in the end, there has to be a human doing the understanding, being convinced.

And the same goes in social and cognitive sciences. There are many experiments in social and cognitive sciences, each proving — assuming they can be replicated — a tidbit about human behavior. If we look at many of them, we certainly see patterns emerge: these results are not completely random, they are the manifestations of deeper phenomenons. But what these phenomenons are, there is still no theory, no explanation. And therefore, social and cognitive are not convincing, not yet.

That brings me back to standardized tests. By eliminating subjectivity in the evaluation, they have the veneer of science, they pose as objective measurements. But the subjectivity is not gone, it never can be. With standardized tests, all the subjectivity has been pushed into that simple question: what are these tests measuring?

I am talking, in essence, of Goodhart's law, but it is more fundamental than that. Until we have the frame of a theory, until we have a scientific language to express our ideas in a rigorous way and act as a bridge between our intuition and objective observations, we cannot design experiments and measurements capable of guiding us towards understanding and conviction.

Which is why I believe, as of now, standardized testing as practiced in the US is inferior to what other countries do. I know especially well what we do here in France (full disclosure: I teach math in a public high-school in one of the poorest suburbs of Paris): the examination exercises are much more subject to the subjectivity of teachers, both when it comes to designing the exercises and evaluating their difficulty, and when it comes to correcting them and grading students, even in math, but I believe they are much closer to the informal ideal of testing what we would like to test.

What I am saying is that the nascent sciences of education are still very far from being able to compete with the intuition of experienced teachers.

And that brings me to another side of the issue: what is school really for? I have skimmed the comments, and I have seen quite a few takes on this. I will give mine, focussing mostly on the latest part of school, from 12 to 18 years old approximately.

The obvious answer: school is about the knowledge and skills that kids gain, is clearly wrong. Acing the test about Gaussian elimination does not mean you will be able to do it until the end of your life, or even two years from now unless you keep practicing, and it is not why the test is administered.

There is an example I use on the first day of every year with my students: considering there is no jumping rope in a boxing match, why are boxers endlessly training with one?

To that question, the students always know the answer: to build the muscles they need.

And this is, in my opinion, the greatest role of school: to build the muscles of the mind at a time where it is most plastic.

By learning Gaussian elimination, you did not just learn a math technique, you trained to follow a set of rules with utmost application, and that will help you in life when it comes to, for example, filling your taxes. Well, maybe not that much, since taxes are usually less logical than they should.

(This is why Gaussian elimination is not that good an exercise, and we do not do much of it here. I teach it, but not as a core topic. Usually, it goes “now that you've toiled moving x, y and z around so many fractions and found the correct coordinates for the intersection of these geometric figures, let me show you how you could have done it twice as fast and without fractions.” (Thanks to the metric system, students here do not need to be as fluent with fractions as Americans.) Also, we certainly do not call it “Gaussian elimination” to the students' face.)

I think the kind of math we do in France in high school is a better example, or at lease one I can explain better. It is mostly centered around demonstration: we have a situation, maybe a function, possibly modeling some kind of real-world problem, and students have to prove properties of this function: is it increasing? is it always positive? What it trains is the ability to think of every tiny detail that can ruin the proof: can I divide by x, or may x be 0? can I use the intersection of these lines, or may they be parallel? It may help them become good mathematicians later, but it may also help become better detectives or lawyers or security experts, because a hole in a mathematical demonstration is the same thing as a loophole in a law or a flaw in a security system.

In the end, to use words that are trending right now, tests about a specific topic are only a proof-of-work for this training.

But I think there is another very important side to school, and I have seen it mentioned a few times in the comments.

I have a friend who toiled into getting their car driving license for utilitarian reasons. And then, on a whip, they started on the motorbike license, and surprisingly they liked it, a lot; now they are making many rides just for the fun of it.

You cannot find your life passion if you never try it.

And this is why school, with its wide array of mandatory topics, is so important.

I suspect many people who end up commenting on a rationalist blog come from a background, starting with family, where knowledge was valued, where trying new things for the fun of experience was part of life, and possibly where parents and other mentors could introduce these things to them. It is not wealth, it is not a network, but it is a kind of privilege too.

And I can assure you, it is not true for everybody.

My students literally live a stone's throw from one of the greatest cultural center of the world, but most of them, left to their own devices, would never go to a museum or an exposition.

School is very important to broaden the horizons of kids, and it is very easy, from the top of the hill that is the life of an adult reading a rationalist blog, to forget that so many kids start with very narrow horizons.

Expand full comment

Perhaps we should remember the very large number of kids around the world whose schooling was severely interrupted 1939-1945? My father was too old and was off fighting, but my mother, and both my in-laws had their schooling disrupted to varying degrees with bombings and evacuations. No one regards the kids who should have been at school in those years as 'damaged' and my relatives all turned out well career-wise.

Expand full comment

I have a friend whose grandma's schooling was disrupted due to WW2. As a consequence, she didn't finish secondary school. I don't know how her life would've turned out otherwise, but I did find a study on this matter I haven't fully read yet. It's called "The Long-Run Educational Cost of World War II", and it's by Ichino and Winter-Ebmer. It claims that Germans and Austrians who were ten years old during the war earned less due to educational disruptions.

Expand full comment

Hard to distinguish that from other WWII stuff, like being bombed, having inadequate nutrition, seeing people you love die, etc etc.

Expand full comment

I taught physics and chemistry in UK secondary schools for 15 years, and every year the majority of students entered Year 11 (of 11) knowing next to nothing of either subject. Many would only start cramming for exams at Christmas, or later. Most achieved respectable grades, albeit from teachers frantically force-feeding them with facts and coaching the language of exams. If schooling worked properly, there would be no need for this annual pantomime. This year would have been little different than any previous year, except it didn't happen. Exams were abandoned in favour of ongoing 'teacher assessment'. Hence, little last minute learning and the best results ever.

Expand full comment

A bit of personal experience:

We live in China, where learning to write is a much bigger part of the curriculum, because it's much harder. My 10 year old did grades 1&2 in an international school, where Chinese teaching was minimal, then switched to a Chinese public school. He's starting grade 5 now and still struggling to keep up, because his slow writing slows him down in every class.

I should point out that this is not an endorsement of the teaching that Chinese schools do. They mostly teach kids by being very negative to the kids who do badly to incentivize parents to provide/get whatever help the kids need. But that function - the enforcing very clear standards function - seemed valuable enough to us that we made the transfer to Chinese schools when the international school was not providing it well enough.

My story is complicated by a bunch of individual factors, of course - my kid is particularly bad at writing (the fine motor side of it), so where another kid might have caught up by now, he hasn't. But it did make me wonder if there are certain crucial skills that schools might be delivering. Because I agree with Scott's thesis for the most part, but this writing thing is really kicking our ass.

Expand full comment

One further reflection: I think Scott's "we forget most of what we get taught in school" is not such an important point as he suggests. Because in that process, you learn something massively important: that you can pick up skills as needed. We never need long division after school, but we remember perfectly well that we learned it in 5th grade or whatever, and so we know we can learn it again. All of that learned-and-forgotten stuff does two things: (1) it maps out the sphere of knowledge, so we are aware that there's stuff beyond the things that we use in our day-to-day; (2) it gives us experience of learning stuff as required, so we have confidence to learn new stuff later.

Expand full comment

All of that can be done in non-school settings. A kid who gets seriously involved in almost anything will find there are skills he needs and can pick up. And reading will tell him there is a sphere of knowledge out there. Having the sphere of knowledge he is exposed to being the curriculum of his school means that it is the same sphere of knowledge all the kids he knows are exposed to, except to the extent that some of them are learning things out of school. That's a narrower view of the world than he can get from his own interests plus those of his friends, parents, other adults he interacts with, the internet, ... .

It isn't as if he gets an extra eight hours a day to do school and homework. That's coming out of time when he could be reading books or arguing politics with his friends or playing games, board games or computer games, that require thinking in order to win, or ...

Expand full comment

In theory you could be right. Perhaps I will be proved wrong. But I have a much more pessimistic view of kids' ability to get stuff done. I've watched my own kids - and my adult self - waste days just goofing off. And I've seen very high achievement through perseverance, and very low achievement because of lack of practice, despite reasonable ability. This is in maths, music, literature, sports, cooking... any area of life.

One of the pieces of advice I've been given in my professional career is to join large organisations, because their structure and their resources will allow you to leverage your skills much better. I've found it to be at least half-true. It'd be odd if the same weren't true of children.

Expand full comment

I remember hearing about some kids who were unschooled. They did what a lot of parents fear: played video games, all day, every day, for months. Eventually, they decided that they wanted to make video games and learned programming and game design. Got really into history, too. And once they started learning they learned really quickly because they had the internal motivation that's lacking when we force kids to learn stuff.

Expand full comment

During the pandemic a lot of kids sat at home bored out of their minds, wasting away on their laptops, melting into twitter and facebook, eating bonbons, and getting fat. They missed badmouthing their teachers and playing jokes on their friends, having tween relationships, and playing sports. No chess club, robotics, jazz band, or lacrosse. Many parents saw their kids wasting away so they ponied up the big bucks for private schools that stayed open. One of my elementary-age daughters got into Finnish hobbyhorsing through school. The school does expose kids to things they would not see at home. None of this is measured on tests...

Expand full comment

For me it's the exposure to different things, different topics (not necessarily extra-curricular) that has felt one of the most valuable things about school for our two. That and the social side (no sufficiently local homeschool groups here) and the sense of belonging to a community.

We ignored 80% of the work that came home during school closures although we did do informal semi-educational things based on their interests and I did limit screen time. Both my boys were at the top of their classes in the main subjects when they returned (one already was before the lockdown, the other wasn't - I had stifle a laugh when his teacher congratulated us on our home-schooling!) so really don't feel that they suffered academically. Anecdotal of course and realise that our circumstances are different to many people's. They're 8 and 10, so perhaps would have been different at other ages too.

But nonetheless, definitely feels like it has been good for them to be back in school even though they coped very well with the closures and I was never worried about them.

Expand full comment

When schools closed I think a lot of parents realized their kids could do an entire days' worth of school work in a few hours. No commuting or lining up, and an individualized curriculum means homeschooling is a more efficient use of time. I get the feeling homeschool parents like to believe they are giving their kids more freedom, but a small homeschool group seems socially stifling. At school my daughters choose their friends from a sizeable group of kids. No adult is deciding who they are going to play with, and then they come home and tell me who they want to have a playdate with. In the homeschool groups I was part of it was always the adults who decided who was going to meet with whom, and where.

Expand full comment

I'm surprised nobody brought up schools as civic religion. That is, societies used to have something like the catechism and mass as a default canon that everyone was expected to know and share in. We don't do that anymore (I mean, many people do, but we don't define our society by that anymore), and I think school tries to fill in that gap, so that instead of not knowing your Old Testament prophets you would not know e.g. your US presidents. (My apologies if I misrepresented what is involved in the catechism.) It's not objectively essential for most people to know the list of US presidents, or the list of British kings, or the list of Chinese emperors; but knowing or at least pretending to know it is a marker of belonging to that culture.

To offer a silly US example, it is surely not imperative for anything to have played Oregon Trail as a kid, but most of my generation did, and not having done so marks me as an outsider. Obviously I can fix this particular deficit, but it's one of rather many; I come from a different school canon. Modern, liberal, nerd, US, society is very permissive of it, but I imagine that's less true in other social circles.

----

I think a common thread in many objections to Scott's post is "yes you can fill in any one thing you missed, but it doesn't follow that you can miss all the things simultaneously and expect to fill them all in". Stable systems tend to have some redundancy by design; the question would seem to be exactly how much of it you need in education, and whether we have too much of it.

Expand full comment

There are scenarios where schooling would make a big difference, especially in 12th grade, but even before that. If a student wants to get into a difficult field, like advanced math, they will need to build each year on their previous knowledge. If they lack the previous knowledge of, let's say, 9th grade math, then they may struggle more with 10th, 11th, 12th, and then college. Additionally, as you mentioned, if you are trying to use the [at that point fresh] knowledge in order to accomplish a task such as college admissions, then getting that information is vital during a time sensitive period.

I think this is directly supported by your first section. What information do we hold onto? The parts that we use. If we have to artificially "use" that information to keep it relevant (as with foreign exchange programs to learn a language or mid level math to learn advanced), then we depend on those programs. There were probably only a tiny fraction of foreign exchange students learning languages over the last 18+ months. That likely will have a huge impact on their ability with that language. Maybe they would have lost that ability over time anyway, but if they wanted to go to college to study Spanish, that immediate loss would be huge in terms of college applications and ability in the program.

Expand full comment

I think it's true that missing 9th-grade math puts you at a disadvantage in future grades, but that's largely because of the way school works. If you had a private tutor or homeschooling parent, they wouldn't just barrel forward with concepts you aren't equipped to understand. They'd work with you to solve the problem, figure out what's tripping you up, and then review the concept you're missing.

To launch on a tangent, it's really hard to disentangle the effects of schooling from the effects of natural brain development that happens as children age. Do sixth graders understand more math than fifth graders because they already did fifth-grade math or because they're a year older? And complicating that, it seems like brain development happens in fits and starts at different ages for different kids. We notice it with height when a short 11-year-old has a growth spurt and becomes a lanky 12-year-old. But it's harder to see when an immature-for-his-age 11-year-old has a spurt of brain development and becomes a mature-for-his-age 12-year-old. And too often we might say, "I guess he just really had a great teacher that year" when rapid development at age 11 may have been cooked into his genes from the start.

We put every kid on a treadmill that goes the same speed for everyone, and if you're slow in one grade and fast in the next, too bad, you already fell off the treadmill.

Expand full comment

In terms of your first paragraph, that reflects on a conundrum. The resources required of society for everyone to have a private tutor or homeschooling parent is massive, far greater than K-12 public school. It's very unlikely to be possible, even if every family was stable enough to provide education at home. Secondly, even though one on one instruction would be much better at handling it, there are some concepts that require the intervening instruction. You can't do calculus without mid level math, including concepts like algebra and geometry. So sure, a private tutor could teach the 9th grade math and then move on to harder concepts instead of just trying to "barrel forward," but missing a year of 9th grade instruction is still missing the year.

For lower levels, I tend to think that age has a lot more to do with it than "did you complete 5th grade math" but it's not a bright line. Some kids are really smart and can conceptually understand math easier than other kids. Some kids need a lot of practice and repetition so that memorization can help make up for conceptual struggles. If I know that 6x8=48, but my brain struggles with what that really means, then I can move through the math curriculum still, just at a slower pace. If my brain really groks the concept, I can zip through that in no time, not bothering to stop and memorize the multiplication table. More advanced subjects, going back to algebra -> calculus for instance, works a bit differently. Unless you're smart enough to derive those concepts naturally, you're going to need to learn them in some kind of order in order to move on.

Expand full comment

1) This post is very interesting and important (and I mostly agree), but I must note that "Steelmanning The Case Against Missing School" has to include a thorough critique of the methodologies of the studies you started with (Benezen, New Orleans, Potonchik, cancer etc.), as critical as you were with the following studies. Else, why should we believe you that one set of studies is less problematic than the other?

You seem to treat the first set as truth, and then interpret the other studies starting from that assumption.

2) Re: homework and other pedagogical tools. I'm ~familiar with studies regarding their efficacy, but also with personal experiences and those of teachers who insist that these tools are useful and are there for a reason (and these are intelligent teachers who actually actively optimize for their students' learning success, and remove things they deem unnecessary). While the discrepancy may be mostly (or even all) teacher's cognitive bias, I think there may be an effect similar to the discrepancy between global clinical trials of psychiatric drugs and psychiatrists' personal experience prescribing them - i.e. the human interaction provides subtle information and differences between people that global experiments miss.

Specifically about homework, though, it seems to me that class work is better than or equal to home work, but some work is better than no work at all, and teachers are logistically more limited in how much class work they can assign (limited time that has to be dedicated to frontal teaching). Idealistic experiments often compare homework to equivalent classwork but teacher's day-to-day is choosing between homework and no work.

Expand full comment

Education differs dramatically in the US between elementary school and middle school. In elementary school, all the students tend to be lumped together in a single class with a single teacher and you're taught all the subjects together.

Once you get out of elementary school, you start getting specialized classes that teach particular topics.

What I think is really going on:

1) Most of elementary school is really about making sure all the kids have the basics and like, know how to read and write, add and subtract and multiply and divide. Everything else is setting the groundwork for further repetition and reinforcement in later schooling, but you can pick this up elsewhere. Your parents can teach you all of these things, so the number of students in first world countries who get screwed here is basically the children whose families are totally illiterate and who can't pick it up otherwise, which is heavily skewed towards disadvantaged minorities.

2) Middle school and high school have you learn in tracks, where you build on previous knowledge. In reality, though, liberal arts mostly don't actually build up in this way; they're mostly about reinforcement. As such, what actually matters is learning math and science. If your parents can teach you this stuff, you're fine; if they can't, though, you're probably screwed if you miss out, because you end up permanently behind.

3) Schools generally teach as fast as the bottom few students learn, rather than as fast as the top few students learn. As such, it will mainly impact the bottom students because the rate of learning is so slow, and they can speed it up and just screw over a few students at the bottom.

As such, I'm not sure if elementary school studies are even useful, and it's possible that a lot of these effects are just "people catch up students who are behind, who can catch up because the speed of teaching is so slow".

There's some other issues with your analysis as well.

For instance, you're assuming that all kinds of absences are created equal. But what if they're not?

In the case of a medical absence, there was some big issue and the teachers know about it and they go out of their way for Little Timmy the Cancer Patient to catch up. In the case of snow days, everyone was gone, so time to catch everyone up! Disaster? Everyone was gone, catch them up.

The only way you lose out here is if you are graduating close enough to the disaster that it is a problem. So people who graduated from 12th grade last year will lose out more because they never had the chance to get caught up, while people who missed, say, 7th grade, they have a lot of years to catch up in.

But if you're just randomly gone a bunch of days, they don't catch you up. You're just missing out on that content entirely. So this hurts you a lot more, because the teachers don't fix things for you.

As such, I'd expect that random absences will matter a lot more than big ones like medical issues or disasters, so these probably aren't actually directly comparable. And I'd expect that later gaps matter more than earlier ones - missing two years of high school will matter a lot more than missing first and second grade.

If students actually were taught stuff as fast as they could learn it, I suspect that absences would screw students a lot more. But they don't.

Expand full comment

Someone may have said this elsewhere in the comments, but "missing school" for extended periods isn't really an option. You're not allowed to not school your kids. You can homeschool or unschool, but that's also currently a worse-than-usual option with stuff like libraries and homeschooling cooperatives not operating normally.

Younger kids are lonely and bored after a year of playing with Legos while their parents tried to work. You could take time away from work to homeschool (and some families do manage to homeschool while working full time). You could hire a nanny or tutor to do stuff with them. But you can't just do nothing.

But yes, during the extended freakout among parents in my child's first grade, I kept chanting "In Finland they don't start academics until age 7."

Expand full comment

A poster told me that all I really need to know I learned in kindergarten. Sucks for anyone who was in Kindergarten this last year, otherwise chill out.

Expand full comment

Kind of feels like all those "we should have scientific cost benefit analysis of shutdown" types spanked pretty hard.

I approach this from a slightly different angle. When the UK shutdown, there was a brief scare that domestic violence would spike and initially there was some surge in calls to the hotlines, but pretty fast calls returned to normal levels even though people were stuck at home with each other more. What happened?

Pretty simple, IMO, if you are the type of man who can't avoid beating his wife unless you have some 'out' to go 8 hours most weekdays, what will happen if all in the sudden you have to be home almost all the time because of a shutdown? Well what happens is I'm sure intutition is right, in some cases violence will happen when it wouldn't have happened before. Other cases, though, people said something like "since going out half the day is no longer an option, I either have to fix the situation or exit it now". Families that would have remained in a borderline situation either broke up or did what they needed to do to resolve their issues. The hotline reverted to baseline because violence that would have otherwise not happened was offset by violence that did not happen but otherwise would if the status quo had continued.

In other words, cost-benefit does not correctly account for compensation.

Expand full comment

So we have the conclusion for parents, which seems to be "don't worry too much".

What about the conclusion for teachers?

If you were a middle school teacher (in a very good school), what would you do differently having that information?

Expand full comment

Teacher's don't have all that much freedom but if I ran a school, I'd be very skeptical of test scores as a measure of quality. Perhaps we need some type of benchmark test that indicates which students have passed or not passed some critical benchmark in various subjects.

Expand full comment

I like the way learn-to-code websites do it. They test you on all the skills they teach and they allow infinite do-overs. The measure of your success isn't the ratio of right answers to wrong answers but the total amount of right answers.

Of course, this only works because the 'grading' is done by a computer and costs basically nothing. But in a world where GPT-12 can grade any essay instantly, we could move to that system.

Expand full comment

Is this, though, the purpose of school? Perhaps the purpose is to keep the kids safe and encourage them if they take up some topic (like coding) but not necessarily to push it until certain milestone points.

Expand full comment
founding

In education, this is largely referred to as mastery based learning (or mastery based grading). At the end of the year, instead a bunch of grades that show how well you learned topics 1 to 10, you get a list of which topics you mastered. At a school I attended that did this, at the beginning of the year you took a pretest on all of the topics from the prior year, then the topics for the current year. You only had to take lessons on the topics you hadn't mastered, and you stayed on that topic until you reached that level (which was getting a 90% or higher on the test). It worked well for both getting ahead quickly and making progress without gaps.

Expand full comment

A note on the hours of schooling vs. PISA math scores: I've read that in certain Asian countries that do well on the PISA (i.e. Korea), students don't spend a lot of hours in school, but they do spend a lot of hours in after school tutoring. Not sure how much that confounds this metric, but I'd suspect in places where students spend more hours in school (one-on-20/30) parents will be less inclined to send their kids to additional tutoring (often 1:1). Not that class size reductions have a strong argument for improving performance, but at a certain point along that curve you should expect some benefit.

Expand full comment

>And I think that’s the whole point. We learn lots of things in school. Then we forget everything except the things that our interests, jobs, and society give us constant exposure/practice to. If I lived in Spain, I would remember Spanish; if I worked in math, I would remember what Gaussian Elimination was. I think a lot of the stuff you’re exposed to and interested in, a sufficiently curious child would learn anyway; the stuff you’re not goes in one ear and out the other, hopefully spending just enough time in between to let you pass the standardized test.

The argument "because we forget much of what we learn in school, school isn't important" proves too much. I forget the content of most books I read years later. Is it not important to read books? No, I say -- those books affected me and had some small impact on my character and thinking. I have forgotten a lot of math since college, but learning the mathematics was still valuable because it taught me how to think mathematically.

Same applies to sports. Most kids won't end up playing professional sports, so should they not bother playing while young? I actually think that competition is important -- kids learn what they can and cannot do.

I think kids need to have some kind of structured learning. Something where they get to compete with themselves at learning something, something that ideally challenges them. Even if they forget all of it, they will hopefully remember the process of learning. Supposedly Yeats said "Education is not the filling of a pail, but the lighting of a fire." At its best, school inspires curiosity and a desire to learn more.

Expand full comment

The main reason to read a book is to enjoy the book while you're reading it. I wouldn't read an unpleasant book in the hope that it would have some positive impact on my character years later. Particularly if that book took over 10,000 hours of my childhood to read!

Expand full comment

I read books for enjoyment too, but I don't think that's the only value of books. I am not suggesting that everyone must read unpleasant books, just that for some activities, there are benefits beyond the short term one..

Expand full comment

I have long maintained that schools have a clear favoritism to girls, who make the honor roll twice as often and high honors three times as often. Boys drop out are far higher rates. That's some patriarchy you got there. Schools are designed by women for girls. (As a side note, I think much early feminist anger sprang from being told that absolutely killing it at school was the key, doing that, and then finding that the out-of-school world played by different rules. Betrayal. Unfair.) As conscientiousness is the Big Five trait where girls have a demonstrably greater strength, it is therefore likely that conscientiousness is one of the main things being taught. I was intrigued that you mentioned it in passing.

That is a very good thing to teach, and if schools taught nothing else they would still have some value. But it's not the only thing. I have five sons, three adopted, two of those foreign, and our experience with public school, private school and home school is varied. I am now convinced, a couple of hundred thousand dollars later, that it's mostly genetics for the academic side, but school has value for the "adapting to life with all its unfairness and randomness" aspect.

Expand full comment

Sorry if someone already sent this, haven't time to read all comments, but I'd be optimistic about enough between country variability in shool closures for comparisons vs. that country's results in previous years to be made. See https://twitter.com/SilverVVulpes/status/1401866064691183616

Expand full comment

I feel like it's possible the teacher strikes and Bad Education outcomes are both caused by the same thing rather than one causing the other

Expand full comment

So do I take

"A kid who misses all of 9th grade will ... be less than 5 percentile points behind in reading (60%)"

to be the same as

"40% of kids who miss all of 9th grade will ... be at least 5 percentile points behind in reading"

or am I misinterpreting you? Phrasing it my way strikes me as somewhat more concerning.

Expand full comment

the percentages in parentheses are confidence intervals. Scott is saying "I think it's more likely than not, but not overwhelmingly likely (i.e., 60% chance), that the average kid who misses all of 9th grade will be less than 5 percentile points behind."

Expand full comment

Yeah, I get that. He's saying he would make a bet against 3-2 odds that...something. That a given kid will be less than 5 points behind? That all kids will be less than 5% behind? That only the 5% of kids that are ignored in a 95-percent confidence trial will be?

Given the wide range in kids' capabilities, this kind of assertion seems to me to assume more uniformity that really pertains. There are probably more than a few kids would would do *better* skipping ninth grade.

If all we can learn from the confidence intervals is stuff about Scott's betting proclivities, I'm not sure they are interesting enough to bother mentioning, especially when they are as close to 50% as this. We agree, don't we, that a 50% confidence tells us nothing at all, right, except that Scott will register mild surprise no matter what happens?

Expand full comment

Because this is the internet, I can’t tell whether you’re in fact unclear about things or rhetorically deploying such language, so I apologize if below I’m simply restating things you already know or advancing arguments you have already considered (and/or rejected).

That said, my read of Scott’s probabilities is informed by his use of “on average” in item 1. So I read him to be saying that he would bet, with 3-2 odds, that whatever the true distribution of the impact of missing 9th grade, the average of that distribution would be less than 5 percentile points.

The “winning conditions” of this bet would cover an infinite set of distributions, of course. It would include distributions where every kid is impacted not at all. It would include distributions where every kid is impacted exactly -4.998 percentile points. (Whatever that would mean; percentiles are funny and I’m not sure this is possible.) It would include normal distributions where the mean & median impact is, say, -4, but the variance is wide enough that about 16% of students are hit by -10 or more. It would include bimodal distributions where over 90% of kids experience no impact and nearly 10% of kids get hit with a whopping -50 percentile point penalty. Etc.

It’s not clear to me that there’s a principled way to decide what the distribution of distributions “should” be, so you can’t convert Scott’s bet into a statement of confidence about the prospects of an individual student.

If your point is “a mean of ~ -5 percentile points with a wide variance seems like a pretty serious impact, and Scott’s confidence that we’re even that safe is relatively low, so these numbers don’t seem to align with the overall nonchalant tone of the essay”, then that’s fair enough. Although if you’re expecting a wide variance and a mean of less than -5, then there are lots of kids out there who would be better off in school for having skipped school—which is its own fairly radical conclusion.

And for your last point, you write “We agree, don't we, that a 50% confidence tells us nothing at all”.

This is apparently hotly debated on this and other forums. But I’m strongly opposed to that idea. 50% confidence tells us stuff all the time.

50% confidence that a coin will return heads expresses a conviction that the coin is fair. 50% confidence that the economy will outperform the Fed estimates expresses a conviction that the Fed is accurate, though not necessarily precise. 50% confidence that Madagascar will have the world’s largest GDP in 2031 represents a remarkable anticipation of a Madagascar economic boom. 50% confidence that I, personally, will win Wimbledon this year tells you that I think I’m fucking fantastic at tennis. And so on.

Now, 50% confidence can be maliciously deployed to avoid responsibility for predictions. If I predict in January of 2020 that Kanye West will win the presidential election, you might laugh at me in November. But if I predict that he will win with 50% confidence, I can respond to your laughter by shrugging and saying “I never thought it was more likely than not.”

This is a real social problem but from a math perspective it’s bullshit. My confidence in that possibility was too high—even if it’s harder to show that I overestimated it.

Now it’s also true that, 50% confidence on a summary statistic across an unstated and undefined distribution of distributions is a pretty squishy thing—especially if it’s more or less aligned with your current thinking. But I think it’s still useful, depending on your pre-existing model of school.

Consider two toy models of school.

In one, each year the students learn a new song. The song is simple and easy to remember (say, “Old McDonald Had a Farm” in 1st grade, “Miss Mary Mack” in 2nd, etc.) but it is strictly taboo to sing or learn them outside of school. At the end of high school students are tested on their ability to perform each song & graded on musicality, precision, etc. Not knowing even a single song is grounds for disqualification.

In this scenario, missing one full year of school would be disastrous. It would be associated with an average -50 percentile point drop, as poor, average, and excellent students alike plummet to the very bottom.

In the other model of school, school does literally nothing for kids. Maybe the activities in school have no net educational benefit over what kids do outside of school. Maybe the tests at the end of school merely measure the physical development of the maturing brain. Whatever.

In this scenario, missing one full year of school would be meaningless. It would be associated with an average 0 percentile point drop. Maybe some kids would benefit and others would lose out, but there’d be no way to know in advance. The question of whether or not to go to school would be answered with a shrug.

Scott’s 60% confidence in -5 percentile points or less implies that he believes the real world is much closer to the second model than the first. If you’re at about the same place yourself, it’s not an interesting remark. But if you’re much closer to the first model—if you’re convinced that every year of school imparts valuable knowledge and the gap is nigh-impossible to make up—then it’s a robust challenge to your thinking.

All that said, of course these are numbers just pulled out of OP’s ass, so, you know, not a lot of precision there. It’s more just pointing at the general shape of the research.

Expand full comment

You make a lot of legit points. I take offense at none of it and I wasn’t trying to score rhetorical points. But it still leaves a bad taste in my mouth. Yes, “50% confidence that a coin will return heads expresses a conviction that the coin is fair.” But (a) there is a much stronger way to state that conviction, and (b) if we analogize the coin flip to Scott’s assertion, it means he’s saying that 60% of the time we will find that the degradation (of students, or groups of students, or something) is less than 5 percentile but 40% of the time we will find that it’s more — and I don’t think that’s what he meant either.

I’m not a big proponent of public schools and otherwise have no particular dog in this fight, so I’m probably just wasting everyone’s time here.

Expand full comment

THANK YOU!! I keep telling my libertarian friends that you cannot for decades complain that the public educational system does a LOUSY job educating children (25% functional illiteracy at high school graduation, for example) and SIMULTANEOUSLY cry out that if we can't get the kids into these same schools their education will be horribly and negatively affected.

Expand full comment

For the last several years, we've been tutoring a boy who missed several years of regular schooling because his alcoholic mother melted down and he was caught in the middle of a cross-state custody battle. He has made up a lot of ground, but he has also been attending a fairly good school and had two well educated tutors helping him.

His biggest problem was in mathematics. The problem with math is that so much of it is cumulative. Even when you are taking advanced analysis in graduate school, assuming you bother to do so, you will still need first grade addition and subtraction. He was in middle school and taking algebra, but we spent a lot of time going over stuff he had missed like the multiplication table, prime numbers and fractions. It is easy to see how someone could miss a chunk of the math curriculum and never recover, not because the material was impossible to learn, but because there was mechanism to teach it.

When I was a kid, I somehow missed whatever my school taught about multiplication and division. My parents, both college educated, made me flash cards, taught me the various algorithms and got me up to speed in a few weeks. If I had not had educated parents or if no one had caught the gap, I could have foundered.

More recently, my niece had a similar problem. She had never learned how to read critically, that is, analyzing what was written and making appropriate inferences. She did terribly on her SATs, so we wound up tutoring her. We gave her a big dose of 19th and early 20th century literature and got her up to speed.

So, a lot depends on what happens when school starts up again. Is the kid simply flunked out, dumped into some awful remedial course, taught at an appropriate level, or given personalized tutoring? Is the problem that led to school absence remedied or merely ameliorated? I think the people most worried about their children missing school due to COVID are the ones whose children are least likely to be affected. Odds are their parents will do the necessary or see it done. It's the children whose parents don't have the resources, the background, the culture or the priorities that are likely to have problems.

Expand full comment

When I first moved to the Dominican Republic I imagined myself holding classes for these poor less educated kids here. Well, that never happened.

What did happen is that I allowed them to do chores and bought them smart phones. Excellent budget phones like the Moto G and Redmi series. Not all at once. I'm certainly not rich. But I've purchased phones for about six kids now.

They are quite familiar with these phones and navigate faster than I do.

It dawned on me that being completely at ease with their smartphones probably has more to do with their future employment than having perfect grammar. The girl who didn't read and write well is quite adept with speech to text.

And "my" kids are not the only kids with phones. In fact they all seem to have them!

Expand full comment

Well, "Kids Can Recover From Missing Even Quite A Lot Of School", so, OK.

I'd agree with that, however, (at least) two questions arise; "Why?" And "What exactly is a lot?"

The "kids can recover" is based on future grades/marks, or subsequent earnings, and is presented as "catching up".

Here's three things; chronological age, developmental age and school age.

Chronological age is determined by your birth date (a statement of the bleeding obvious if ever there was one). Developmental age is a sequence of milestones passed at a chronological age.

So, crawling, walking, talking, not wetting the bed. That sort of stuff, for infants.

Chronological age is not necessarily the greatest specific predictor of developmental age. It roughly works, but can be really rather vague - "by about 18 or 24 months".

Note that the difference there is six months, being a third of 18 months, a quarter of 24.

School age (or school year, grade for the Yanks) is an artifact. The underlying assumption being that chronological age is a good predictor for developmental age, continuously, for the entire duration of formal education, for some large majority of children.

Testing, exams, take place at specific times during the school year(s). Notionally, they're measures of information, knowledge, facts received and retained, and abstractions successfully manipulated. Scott says this - that test scores, marks, are partially dependent on when the test is taken, and differences in ranking between children is due to the mis-match between the three ages.

So, just wait. For some children, delay delivering the information, for some period, and they magically catch-up with others as they get older and reach an equivalent developmental age.

And the system does, kind of, have at least one way of handling this, within each specific school year, or measurement period. Revision.

Finish elements of the curriculum early enough, to spend time going over the information again, ahead of the test. Some children who were struggling earlier, suddenly get it. Some of those, simply got older.

Where subjects might be interdependent, say maths and physics, you can get the same effect. Being taught something in Maths, and then having to use it in Physics two or three weeks later. Where subjects are not, English (other languages may be available) and, I dunno, Geography, you might have a bit of a problem, and rely on repetition alone during the teaching phase.

And there are birth date effects; "When you are born matters: the impact of date of birth on educational outcomes in England" (https://ifs.org.uk/wps/wp1006.pdf) or "Season of Birth Effects and the Role of Emotional Intelligence: A Review of the Literature" (https://www.cambridgeassessment.org.uk/Images/109784-birthdate-effects-a-review-of-the-literature-from-1990-on.pdf).

So, in the UK, the school year runs from first week of September into about the second week on July the following year. The cut-off for birth dates are 1st September to 31 August.

This means that a child born in early August can be in the same school year, attempting to learn the same things, as a child born in early September of the previous year - with the older child having an apparent developmental advantage of nearly twelve sodding months.

At age 6, that's really quite a large number of months, between the relative chronological ages. But as they age, that relative gap shrinks, but the apparent disadvantage persists.

Best guess; the current definition school age is too long compared to chronological age, being a vague proxy for developmental age. So developmental differences persist throughout the education system.

One solution - increase the granularity of school age, by dividing into quarters. Pupil intakes take place four times per (current) school year. 1st September, plus December, February, and May.

Exams take place four times per school year, once per quarterly cohort.

If the idea that the mis-match of school age and developmental age causes a significant difference between test rankings is correct, then the range of grades awarded, results achieved, should narrow when measured at chronological age, and could be very narrow indeed at school age.

At this point, I wouldn't like to speculate about what the wider effects might be, 'cos that seems like far too much work.

As to "what's quite a lot?" - I haven't a scooby. But altering the calendar, would most likely change the answer.

Expand full comment